高校生のための数学の質問スレPART331

このエントリーをはてなブックマークに追加
1132人目の素数さん
前スレ
高校生のための数学の質問スレPART330
http://uni.2ch.net/test/read.cgi/math/1334845283/

【質問者必読!】
まず>>1-3をよく読んでね

数学@2ch掲示板用 掲示板での数学記号の書き方例と一般的な記号の使用例
http://mathmathmath.dotera.net/

・まずは教科書、参考書、web検索などで調べるようにしましょう。(特に基本的な公式など)
・問題の写し間違いには気をつけましょう。
・長い分母分子を含む分数はきちんと括弧でくくりましょう。
  (× x+1/x+2 ;  ○((x+1)/(x+2)) )
・丸文字、顔文字、その他は環境やブラウザによりうまく表示できない場合があります。
 どうしても画像を貼る場合はPCから直接見られるところに見やすい画像を貼ってください。
 ピクトはPCから見られないことがあるので避けてください。
・質問者は名前を騙られたくない場合、トリップを付けましょう。 (トリップの付け方は 名前(N)に 俺!#oretrip ←適当なトリ)
・質問者は回答者がわかるように問題を書くようにしましょう。でないと放置されることがあります。
  (変に省略するより全文書いた方がいい、また説明なく習慣的でない記号を使わないように)
・質問者は何が分からないのか、どこまで考えたのかを明記しましょう。それがない場合、放置されることがあります。
  (特に、自分でやってみたのにあわないので教えてほしい、みたいなときは必ず書くように)
・970くらいになったら次スレを立ててください。
2132人目の素数さん:2012/05/06(日) 18:11:06.64
基本的な記号の使い方は以下を参照してください。その他については>>1のサイトで。
■ 足し算/引き算/掛け算/割り算(加減乗除)
 a+b → a 足す b   (足し算)     a-b → a 引く b    (引き算)
 a*b → a 掛ける b  (掛け算)     a/b → a 割る b    (割り算)
■ 累乗 ^
 a^b     a の b乗
 a^(b+1)  a の b+1乗
 a^b + 1  (a の b乗) 足す 1
■ 括弧の使用
 a/(b + c) と a/b + c
 a/(b*c)  と a/b*c
 はそれぞれ、違う意味です。括弧を多用して、キチンと区別をつけてください。
■ 数列
 a[n] or a_(n)     → 数列aの第n項目
 a[n+1] = a[n] + 3  → 等差数列の一例
 Σ[k=1,n]a_(k)     → 数列の和
■ 積分 ( "∫"は「せきぶん」「いんてぐらる」「きごう」などで変換せよ(環境によって異なる).)
 ∫[0,1] x^2 dx = (x^3)/3|_[x=0,1]
 ∫[0,x] sin(t) dt
■ 三角関数
 (sin(x))^2 + (cos(x))^2 = 1
 cos(2x) = (cos(x))^2 - (sin(x))^2
■ ベクトル
 AB↑ a↑
 ベクトル:V=[V[1],V[2],...], |V>, V↑, vector(V)
 (混同しない場合はスカラーと同じ記号でいい.通常は縦ベクトルとして扱う.)
■行列
 (全成分表示):M=[[M[1,1],M[2,1],...],[M[1,2],M[2,2],...],...], I=[[1,0,0,...],[0,1,0,...],...]
 (行(または列ごと)に表示する. 例)M=[[1,-1],[3,2]])
■順列・組合せ
 P[n,k]=nPk, C[n.k]=nCk, H[n,k]=nHk
3132人目の素数さん:2012/05/06(日) 18:12:11.87
主な公式と記載例

(a±b)^2=a^2±2ab+b^2
(a±b)^3=a^3±3a^2b+3ab^2±b^3
a^3±b^3=(a±b)(a^2干ab+b^2)

√a*√b=√(ab)、√a/√b=√(a/b)、 √(a^2b)=a√b [a > 0、b > 0]
√((a+b)±2√(ab))=√a±√b [a > b > 0]

ax^2+bx+c=a(x-α)(x-β)=0 [a≠0、α+β=-b/a、αβ=c/a]
(α,β)=(-b±√(b^2-4ac))/2a  [2次方程式の解の公式]

a/sin(A)=b/sin(B)=c/sin(C)=2R [正弦定理]
a^2=b^2+c^2-2bccos(A)      [余弦定理]

sin(a±b)=sin(a)cos(b)±cos(a)sin(b)  [加法定理]
cos(a±b)=cos(a)cos(b)干sin(a)sin(b)

log_{a}(xy)=log_{a}(x)+log_{a}(y)
log_{a}(x/y)=log_{a}(x)-log_{a}(y)
log_{a}(x^n)=n(log_{a}(x))
log_{a}(x)=(log_{b}(x))/(log_{b}(a))  [底の変換定理]

f'(x)=lim_[h→0] (f(x+h)-f(x))/h  [微分の定義]
(f±g)'=f'±g'、(fg)'=f'g+fg'、(f/g)'=(f'g-fg')/(g^2) [和差積商の微分]
4132人目の素数さん:2012/05/06(日) 18:12:33.69
          __ノ)-'´ ̄ ̄`ー- 、_
        , '´  _. -‐'''"二ニニ=-`ヽ、
      /   /:::::; -‐''"        `ーノ
     /   /:::::/           \
     /    /::::::/          | | |  |
     |   |:::::/ /     |  | | | |  |
      |   |::/ / / |  | ||  | | ,ハ .| ,ハ|
      |   |/ / / /| ,ハノ| /|ノレ,ニ|ル' 
     |   |  | / / レ',二、レ′ ,ィイ|゙/   私は只の数ヲタなんかとは付き合わないわ。
.     |   \ ∠イ  ,イイ|    ,`-' |      頭が良くて数学が出来てかっこいい人。それが必要条件よ。
     |     l^,人|  ` `-'     ゝ  |        さらに Ann.of Math に論文書けば十分条件にもなるわよ。
      |      ` -'\       ー'  人          一番嫌いなのは論文数を増やすためにくだらない論文を書いて
    |        /(l     __/  ヽ、           良い論文の出版を遅らせるお馬鹿な人。
     |       (:::::`‐-、__  |::::`、     ヒニニヽ、         あなたの論文が Ann of Math に accept される確率は?
    |      / `‐-、::::::::::`‐-、::::\   /,ニニ、\            それとも最近は Inv. Math. の方が上かしら?
   |      |::::::::::::::::::|` -、:::::::,ヘ ̄|'、  ヒニ二、 \
.   |      /::::::::::::::::::|::::::::\/:::O`、::\   | '、   \
   |      /:::::::::::::::::::/:::::::::::::::::::::::::::::'、::::\ノ  ヽ、  |
  |      |:::::/:::::::::/:::::::::::::::::::::::::::::::::::'、',::::'、  /:\__/‐、
  |      |/:::::::::::/::::::::::::::::::::::::::::::::::O::| '、::| く::::::::::::: ̄|
   |     /_..-'´ ̄`ー-、:::::::::::::::::::::::::::::::::::|/:/`‐'::\;;;;;;;_|
   |    |/::::::::::::::::::::::\:::::::::::::::::::::::::::::|::/::::|::::/:::::::::::/
    |   /:::::::::::::::::::::::::::::::::|:::::::::::::::::::::O::|::|::::::|:::::::::::::::/
5132人目の素数さん:2012/05/06(日) 18:13:08.45
          __ノ)-'´ ̄ ̄`ー- 、_
        , '´  _. -‐'''"二ニニ=-`ヽ、
      /   /:::::; -‐''"        `ーノ
     /   /:::::/           \
     /    /::::::/          | | |  |
     |   |:::::/ /     |  | | | |  |
      |   |::/ / / |  | ||  | | ,ハ .| ,ハ|
      |   |/ / / /| ,ハノ| /|ノレ,ニ|ル' 
     |   |  | / / レ',二、レ′ ,ィイ|゙/   私は只の数ヲタなんかとは付き合わないわ。
.     |   \ ∠イ  ,イイ|    ,`-' |      頭が良くて数学が出来てかっこいい人。それが必要条件よ。
     |     l^,人|  ` `-'     ゝ  |        さらに Ann.of Math に論文書けば十分条件にもなるわよ。
      |      ` -'\       ー'  人          一番嫌いなのは論文数を増やすためにくだらない論文を書いて
    |        /(l     __/  ヽ、           良い論文の出版を遅らせるお馬鹿な人。
     |       (:::::`‐-、__  |::::`、     ヒニニヽ、         あなたの論文が Ann of Math に accept される確率は?
    |      / `‐-、::::::::::`‐-、::::\   /,ニニ、\            それとも最近は Inv. Math. の方が上かしら?
   |      |::::::::::::::::::|` -、:::::::,ヘ ̄|'、  ヒニ二、 \
.   |      /::::::::::::::::::|::::::::\/:::O`、::\   | '、   \
   |      /:::::::::::::::::::/:::::::::::::::::::::::::::::'、::::\ノ  ヽ、  |
  |      |:::::/:::::::::/:::::::::::::::::::::::::::::::::::'、',::::'、  /:\__/‐、
  |      |/:::::::::::/::::::::::::::::::::::::::::::::::O::| '、::| く::::::::::::: ̄|
   |     /_..-'´ ̄`ー-、:::::::::::::::::::::::::::::::::::|/:/`‐'::\;;;;;;;_|
   |    |/::::::::::::::::::::::\:::::::::::::::::::::::::::::|::/::::|::::/:::::::::::/
    |   /:::::::::::::::::::::::::::::::::|:::::::::::::::::::::O::|::|::::::|:::::::::::::::/
6132人目の素数さん:2012/05/06(日) 18:13:26.88
          __ノ)-'´ ̄ ̄`ー- 、_
        , '´  _. -‐'''"二ニニ=-`ヽ、
      /   /:::::; -‐''"        `ーノ
     /   /:::::/           \
     /    /::::::/          | | |  |
     |   |:::::/ /     |  | | | |  |
      |   |::/ / / |  | ||  | | ,ハ .| ,ハ|
      |   |/ / / /| ,ハノ| /|ノレ,ニ|ル' 
     |   |  | / / レ',二、レ′ ,ィイ|゙/   私は只の数ヲタなんかとは付き合わないわ。
.     |   \ ∠イ  ,イイ|    ,`-' |      頭が良くて数学が出来てかっこいい人。それが必要条件よ。
     |     l^,人|  ` `-'     ゝ  |        さらに Ann.of Math に論文書けば十分条件にもなるわよ。
      |      ` -'\       ー'  人          一番嫌いなのは論文数を増やすためにくだらない論文を書いて
    |        /(l     __/  ヽ、           良い論文の出版を遅らせるお馬鹿な人。
     |       (:::::`‐-、__  |::::`、     ヒニニヽ、         あなたの論文が Ann of Math に accept される確率は?
    |      / `‐-、::::::::::`‐-、::::\   /,ニニ、\            それとも最近は Inv. Math. の方が上かしら?
   |      |::::::::::::::::::|` -、:::::::,ヘ ̄|'、  ヒニ二、 \
.   |      /::::::::::::::::::|::::::::\/:::O`、::\   | '、   \
   |      /:::::::::::::::::::/:::::::::::::::::::::::::::::'、::::\ノ  ヽ、  |
  |      |:::::/:::::::::/:::::::::::::::::::::::::::::::::::'、',::::'、  /:\__/‐、
  |      |/:::::::::::/::::::::::::::::::::::::::::::::::O::| '、::| く::::::::::::: ̄|
   |     /_..-'´ ̄`ー-、:::::::::::::::::::::::::::::::::::|/:/`‐'::\;;;;;;;_|
   |    |/::::::::::::::::::::::\:::::::::::::::::::::::::::::|::/::::|::::/:::::::::::/
    |   /:::::::::::::::::::::::::::::::::|:::::::::::::::::::::O::|::|::::::|:::::::::::::::/
7132人目の素数さん:2012/05/06(日) 18:24:24.15
3次関数f(x)=x^3+ax^2+bx+cがある。
任意の2次関数g(x)に対して、つねに
刀m-1,1]f(x)g(x) dx=0が成り立つとき、定数a,b,cの値を求めよ。

よろしくお願いします
8132人目の素数さん:2012/05/06(日) 18:28:30.04
∠ABC
9132人目の素数さん:2012/05/06(日) 18:29:18.99
ない
10132人目の素数さん:2012/05/06(日) 18:30:24.02
任意の自然数nに対して、不等式 10^n>n^2 が成立することを示せ 
よろしく御願いします
11132人目の素数さん:2012/05/06(日) 18:35:21.43
どこまで考えたのか詳しく
12132人目の素数さん:2012/05/06(日) 18:46:04.61
          __ノ)-'´ ̄ ̄`ー- 、_
        , '´  _. -‐'''"二ニニ=-`ヽ、
      /   /:::::; -‐''"        `ーノ
     /   /:::::/           \
     /    /::::::/          | | |  |
     |   |:::::/ /     |  | | | |  |
      |   |::/ / / |  | ||  | | ,ハ .| ,ハ|
      |   |/ / / /| ,ハノ| /|ノレ,ニ|ル' 
     |   |  | / / レ',二、レ′ ,ィイ|゙/   私は只の数ヲタなんかとは付き合わないわ。
.     |   \ ∠イ  ,イイ|    ,`-' |      頭が良くて数学が出来てかっこいい人。それが必要条件よ。
     |     l^,人|  ` `-'     ゝ  |        さらに Ann.of Math に論文書けば十分条件にもなるわよ。
      |      ` -'\       ー'  人          一番嫌いなのは論文数を増やすためにくだらない論文を書いて
    |        /(l     __/  ヽ、           良い論文の出版を遅らせるお馬鹿な人。
     |       (:::::`‐-、__  |::::`、     ヒニニヽ、         あなたの論文が Ann of Math に accept される確率は?
    |      / `‐-、::::::::::`‐-、::::\   /,ニニ、\            それとも最近は Inv. Math. の方が上かしら?
   |      |::::::::::::::::::|` -、:::::::,ヘ ̄|'、  ヒニ二、 \
.   |      /::::::::::::::::::|::::::::\/:::O`、::\   | '、   \
   |      /:::::::::::::::::::/:::::::::::::::::::::::::::::'、::::\ノ  ヽ、  |
  |      |:::::/:::::::::/:::::::::::::::::::::::::::::::::::'、',::::'、  /:\__/‐、
  |      |/:::::::::::/::::::::::::::::::::::::::::::::::O::| '、::| く::::::::::::: ̄|
   |     /_..-'´ ̄`ー-、:::::::::::::::::::::::::::::::::::|/:/`‐'::\;;;;;;;_|
   |    |/::::::::::::::::::::::\:::::::::::::::::::::::::::::|::/::::|::::/:::::::::::/
    |   /:::::::::::::::::::::::::::::::::|:::::::::::::::::::::O::|::|::::::|:::::::::::::::/
13132人目の素数さん:2012/05/06(日) 19:31:01.28
>10
帰納法
14132人目の素数さん:2012/05/06(日) 19:51:03.04
          __ノ)-'´ ̄ ̄`ー- 、_
        , '´  _. -‐'''"二ニニ=-`ヽ、
      /   /:::::; -‐''"        `ーノ
     /   /:::::/           \
     /    /::::::/          | | |  |
     |   |:::::/ /     |  | | | |  |
      |   |::/ / / |  | ||  | | ,ハ .| ,ハ|
      |   |/ / / /| ,ハノ| /|ノレ,ニ|ル' 
     |   |  | / / レ',二、レ′ ,ィイ|゙/   私は只の数ヲタなんかとは付き合わないわ。
.     |   \ ∠イ  ,イイ|    ,`-' |      頭が良くて数学が出来てかっこいい人。それが必要条件よ。
     |     l^,人|  ` `-'     ゝ  |        さらに Ann.of Math に論文書けば十分条件にもなるわよ。
      |      ` -'\       ー'  人          一番嫌いなのは論文数を増やすためにくだらない論文を書いて
    |        /(l     __/  ヽ、           良い論文の出版を遅らせるお馬鹿な人。
     |       (:::::`‐-、__  |::::`、     ヒニニヽ、         あなたの論文が Ann of Math に accept される確率は?
    |      / `‐-、::::::::::`‐-、::::\   /,ニニ、\            それとも最近は Inv. Math. の方が上かしら?
   |      |::::::::::::::::::|` -、:::::::,ヘ ̄|'、  ヒニ二、 \
.   |      /::::::::::::::::::|::::::::\/:::O`、::\   | '、   \
   |      /:::::::::::::::::::/:::::::::::::::::::::::::::::'、::::\ノ  ヽ、  |
  |      |:::::/:::::::::/:::::::::::::::::::::::::::::::::::'、',::::'、  /:\__/‐、
  |      |/:::::::::::/::::::::::::::::::::::::::::::::::O::| '、::| く::::::::::::: ̄|
   |     /_..-'´ ̄`ー-、:::::::::::::::::::::::::::::::::::|/:/`‐'::\;;;;;;;_|
   |    |/::::::::::::::::::::::\:::::::::::::::::::::::::::::|::/::::|::::/:::::::::::/
    |   /:::::::::::::::::::::::::::::::::|:::::::::::::::::::::O::|::|::::::|:::::::::::::::/
15132人目の素数さん:2012/05/06(日) 20:05:40.68
>7
とりあえず任意の2次関数を
g(x)=px^2+qx+rとおいて計算してみたのですがそこからどうすれば良いか分からずで…
16132人目の素数さん:2012/05/06(日) 20:34:11.98
>>15
刀@周回積分
∬ 二重積分
∫ いわゆる普通の高校生レベルの積分
17132人目の素数さん:2012/05/06(日) 20:37:23.50
aを実数の定数として、異なる2つの実数解をもつxの二次方程式
x^2+ax+2a^2-8=0 を考える。
このとき、
(1)x=0が1つの解で他の解が正のとき、aの値を求めよ。
(2)1つの解が負で、1つの解が正のとき、aの値の範囲を求めよ。
(3)1つの解のみ正のとき、aの値の範囲を求めよ。
(4)2つの解がともに正のとき、aの値の範囲を求めよ。

おねがいします
18132人目の素数さん:2012/05/06(日) 20:41:28.35
>>15
その結果がp,q,r各々について恒等式
19132人目の素数さん:2012/05/06(日) 20:48:45.58
>7
すいません
唐ナはなく∫でした
20132人目の素数さん:2012/05/06(日) 21:11:10.11
なんか偽者&釣り氏がいる模様
21132人目の素数さん:2012/05/06(日) 21:18:53.29
          __ノ)-'´ ̄ ̄`ー- 、_
        , '´  _. -‐'''"二ニニ=-`ヽ、
      /   /:::::; -‐''"        `ーノ
     /   /:::::/           \
     /    /::::::/          | | |  |
     |   |:::::/ /     |  | | | |  |
      |   |::/ / / |  | ||  | | ,ハ .| ,ハ|
      |   |/ / / /| ,ハノ| /|ノレ,ニ|ル' 
     |   |  | / / レ',二、レ′ ,ィイ|゙/   私は只の数ヲタなんかとは付き合わないわ。
.     |   \ ∠イ  ,イイ|    ,`-' |      頭が良くて数学が出来てかっこいい人。それが必要条件よ。
     |     l^,人|  ` `-'     ゝ  |        さらに Ann.of Math に論文書けば十分条件にもなるわよ。
      |      ` -'\       ー'  人          一番嫌いなのは論文数を増やすためにくだらない論文を書いて
    |        /(l     __/  ヽ、           良い論文の出版を遅らせるお馬鹿な人。
     |       (:::::`‐-、__  |::::`、     ヒニニヽ、         あなたの論文が Ann of Math に accept される確率は?
    |      / `‐-、::::::::::`‐-、::::\   /,ニニ、\            それとも最近は Inv. Math. の方が上かしら?
   |      |::::::::::::::::::|` -、:::::::,ヘ ̄|'、  ヒニ二、 \
.   |      /::::::::::::::::::|::::::::\/:::O`、::\   | '、   \
   |      /:::::::::::::::::::/:::::::::::::::::::::::::::::'、::::\ノ  ヽ、  |
  |      |:::::/:::::::::/:::::::::::::::::::::::::::::::::::'、',::::'、  /:\__/‐、
  |      |/:::::::::::/::::::::::::::::::::::::::::::::::O::| '、::| く::::::::::::: ̄|
   |     /_..-'´ ̄`ー-、:::::::::::::::::::::::::::::::::::|/:/`‐'::\;;;;;;;_|
   |    |/::::::::::::::::::::::\:::::::::::::::::::::::::::::|::/::::|::::/:::::::::::/
    |   /:::::::::::::::::::::::::::::::::|:::::::::::::::::::::O::|::|::::::|:::::::::::::::/
22132人目の素数さん:2012/05/06(日) 21:35:50.12
わかりません。教えてください(T_T)
1次の多項式A、BについてAをBで割った商と余りを求めよ
(1)A=2x^3+1-4x,B=3-2x+2x^2
(2)A=3x^3+2x^2+5,B=3x+5

2次の条件を満たす多項式A,Bを求めよ。
(1)Aをx^2+x-3で割ると、商が4x-1,余りが-13-5である。
(2)2x^3-3x^2+2x+8をBで割ると、商がx^2-2x+2,余りが6である。

3(1) 等式3x^2-2x-1=a(x+1)^2+b(x+1)+cがxについて恒等式であるとき、定数a,b,cの値を求めよ。
(2) 等式(k+1)x-(3k+2)y+2k+7=0がすべてのkに対して成り立つとき、定数x,yを求めよ。
(3)等式x^2+2x-1=a(x+3)+cがxについての恒等式であるように定数a,b,cを求めよ。
23132人目の素数さん:2012/05/06(日) 21:37:04.65
これもわからないです。やり方と答えを教えてください
1、次の等式を証明せよ。
(1) (a+b)(a^3+b^3)-(a^2+b^2)^2=ab(a-b)^2
(2) (a^2-b^2)(c^2-b^2)=(ac+bd)^2-(ad+bc)^2
2、a+b+c=0のとき、次の等式が成り立つことを証明せよ。

a^2-bc=b^2-ca

3、a>b,c>dのとき、次の不等式が成り立つことを証明せよ。
(1) a+c>b+d
(2)ac+bd>ad+bc
お願いします。
24132人目の素数さん:2012/05/06(日) 21:38:05.13
          __ノ)-'´ ̄ ̄`ー- 、_
        , '´  _. -‐'''"二ニニ=-`ヽ、
      /   /:::::; -‐''"        `ーノ
     /   /:::::/           \
     /    /::::::/          | | |  |
     |   |:::::/ /     |  | | | |  |
      |   |::/ / / |  | ||  | | ,ハ .| ,ハ|
      |   |/ / / /| ,ハノ| /|ノレ,ニ|ル' 
     |   |  | / / レ',二、レ′ ,ィイ|゙/   私は只の数ヲタなんかとは付き合わないわ。
.     |   \ ∠イ  ,イイ|    ,`-' |      頭が良くて数学が出来てかっこいい人。それが必要条件よ。
     |     l^,人|  ` `-'     ゝ  |        さらに Ann.of Math に論文書けば十分条件にもなるわよ。
      |      ` -'\       ー'  人          一番嫌いなのは論文数を増やすためにくだらない論文を書いて
    |        /(l     __/  ヽ、           良い論文の出版を遅らせるお馬鹿な人。
     |       (:::::`‐-、__  |::::`、     ヒニニヽ、         あなたの論文が Ann of Math に accept される確率は?
    |      / `‐-、::::::::::`‐-、::::\   /,ニニ、\            それとも最近は Inv. Math. の方が上かしら?
   |      |::::::::::::::::::|` -、:::::::,ヘ ̄|'、  ヒニ二、 \
.   |      /::::::::::::::::::|::::::::\/:::O`、::\   | '、   \
   |      /:::::::::::::::::::/:::::::::::::::::::::::::::::'、::::\ノ  ヽ、  |
  |      |:::::/:::::::::/:::::::::::::::::::::::::::::::::::'、',::::'、  /:\__/‐、
  |      |/:::::::::::/::::::::::::::::::::::::::::::::::O::| '、::| く::::::::::::: ̄|
   |     /_..-'´ ̄`ー-、:::::::::::::::::::::::::::::::::::|/:/`‐'::\;;;;;;;_|
   |    |/::::::::::::::::::::::\:::::::::::::::::::::::::::::|::/::::|::::/:::::::::::/
    |   /:::::::::::::::::::::::::::::::::|:::::::::::::::::::::O::|::|::::::|:::::::::::::::/
25132人目の素数さん:2012/05/06(日) 21:39:00.14
>>22
多項式の除法
多項式の割り算
あたりの項目を教科書や学校で使っている問題集などで調べてください
筆算を使って計算をする方法が載っていると思います
26132人目の素数さん:2012/05/06(日) 21:40:17.84
>>23
それも数2の式と証明とか恒等式とか
そういったあたりの単元に
色々載っていると思います
27132人目の素数さん:2012/05/06(日) 21:40:23.24
>25
私、数学が苦手で、教科書見ても全くわからないんです。
28132人目の素数さん:2012/05/06(日) 21:44:57.69
>>27
とりあえず筆算をしようとしてください
あとはつまずいたところで質問してください
29132人目の素数さん:2012/05/06(日) 21:47:59.35
>>27
最初の問題だけやったんですが、商2x-4 余り-6x+13になりました
合ってますか?あとその次の問題で早速詰みました
30132人目の素数さん:2012/05/06(日) 21:48:44.40
>>28
でした
31132人目の素数さん:2012/05/06(日) 21:59:57.82
>>29
違うんじゃないかな?
32132人目の素数さん:2012/05/06(日) 22:01:55.94
>>31
ほんっと数学苦手なんで教えてください(T_T)
33132人目の素数さん:2012/05/06(日) 22:02:43.38
>>32
まず筆算の式を書く
34132人目の素数さん:2012/05/06(日) 22:05:07.36
>>17です。
すみませんこの問題おねがいします・・・
35132人目の素数さん:2012/05/06(日) 22:07:32.96
  
_2x-4______
x^2-2x+3 )2x^3 -4x +1 
2x^3-4x^2+6x  
--------------------
   -4x^2+2x+1
-4x^2+8x-12
― ― ― ― ― ―
          -6x+13

36132人目の素数さん:2012/05/06(日) 22:07:48.52
(1)分数同士のわり算の商を求めるのに逆数をかける理由を文字を用いて証明せよ。

(2)数あて問題
1)1けたの好きな数を選ぶ。
2)好きな偶数を決める。
3)1)と2)を足す
4)2)と3)を足す。
5)3)と4)を足す。
6)4)と5)を足す。
7)その数の一の位で数が当てられます。
なぜ当てられるのか。説明しなさい。

1)=a、2)=2bとすると、3a+10bまではできます。
 (もしくは、1)2)をa,bとして、3a+5b, b=2m⇒3a+10m)

この後の文言をどうすればいいのか。
37132人目の素数さん:2012/05/06(日) 22:11:00.20
>>34
方程式の左辺をf(x)とでもおいて条件に合うようにf(0)とか軸を計算
38132人目の素数さん:2012/05/06(日) 22:12:06.11
>>35
(1)は
B=3-2x+2x^2
だよ?
x^2の係数は2だよ
39132人目の素数さん:2012/05/06(日) 22:14:09.97
>>17
(1)
x^2+ax+2a^2-8=0
にx=0放り込んでaを定めて
その上でそのaに対するxの方程式の解を調べて。

(2)
f(x)=x^2+ax+2a^2-8
とした時
f(0)<0となるaの範囲が求める答え

(3)
意味解らん・・

(4)
判別式D≧0
y=f(x)で
軸の方程式x=-a/2>0
f(0)>0

この3つの条件からaの範囲を定める
40132人目の素数さん:2012/05/06(日) 22:21:29.54
>>22
最初の2問については>>25の指摘の通り教科書で「整式の除法」を学習すること。
注意)割られる式も割る式も降べきの順にすること

[2]「次の条件を満たす多項式A,Bを求めよ。
(1)Aをx^2+x-3で割ると、商が4x-1,余りが-13-5である。
(2)2x^3-3x^2+2x+8をBで割ると、商がx^2-2x+2,余りが6である。

↑についても教科書にあるハなので↓を利用
(割られる式)=(商)×(割る式)+(余り)

ところで(1)の余りは「-13-5」ではなくて「-13x-5」とかでないか?

>>29
(1)A=2x^3+1-4x,B=3-2x+2x^2 は
A=2x^3 +0x^2 -4x +1, B=2x^2-2x+3 としてやり直せ。 
(商)は「x+1」になると思うけどオレは計算嫌いだし苦手。

検討を祈る!
41132人目の素数さん:2012/05/06(日) 22:22:28.33
>>36
10bの1の位は0
つまり
3a+10bの1のくらいは3aの1の位
3aは1けたの数を3倍したもののどれか
そしてそれらの1の位の数はすべて異なる
42132人目の素数さん:2012/05/06(日) 22:24:43.47
>>39
わたしも(3)の意味がわからないんです・・・
(2)とどう違うのでしょうか
4340 :2012/05/06(日) 22:25:22.09
x)↑についても教科書にあるハなので↓を利用
○)↑についても教科書にあるハズなので↓を利用

おまけに誤字、脱字も多いときたもんだ。
44132人目の素数さん:2012/05/06(日) 22:26:58.33
>>40 
何言ってんのかなぁ・・・と思ったら問題の写し間違いでしたすいません
A=2x^3+1-4x,B=3-2x+2x^2 →A=2x^3+1-4x,B=3-2x+x^2
45132人目の素数さん:2012/05/06(日) 22:27:07.10
46132人目の素数さん:2012/05/06(日) 22:30:15.83
>>39>>42
(3)1つの解のみ正のとき、aの値の範囲を求めよ。
もう一つの解は負の時もあれば0(ゼロ)の時もあるって意味じゃね?
47132人目の素数さん:2012/05/06(日) 22:30:22.70
>>42
(3)
一つの解が正でもう一つの解が負又は0
(1)と(2)の両方あわせたaの範囲だろう
48132人目の素数さん:2012/05/06(日) 22:31:21.40
49132人目の素数さん:2012/05/06(日) 22:36:35.50
>>48
ありがとうございます。
その次の問題も途中まで解いたのですが
_x^2______
3x+5)3x^3+2x^2+5
3x^2+5x^2

で詰みました
50132人目の素数さん:2012/05/06(日) 22:38:21.75
>>49
3xとx^2かけたら3x^3だ
51132人目の素数さん:2012/05/06(日) 22:40:02.38
>>50
もう頭悪すぎて泣きそうです(T_T)
52132人目の素数さん:2012/05/06(日) 22:42:46.58
>>46
では、
(1)の答えが、a=±2
(2)の答えが、-2<a<2
になったので、
(3)の答えは、-2≦a≦2 となるということでしょうか?
53132人目の素数さん:2012/05/06(日) 22:43:45.83
>>44
(商)は正解かも、で余りは「4x-11」かな。イイ線まで行ってる、あと一息。
答え付きの教科書準拠の問題集を買って正解になるまで繰り返し学習すべし。
教科書の出版社に電話して聞くと良い。

検討を祈る!
パソコンで十分理解できるまで教えるのは難易度が高いので。
54132人目の素数さん:2012/05/06(日) 22:47:02.44
>>52
当方、計算嫌いだし苦手なのでその辺りヨロシク。

検討を祈る! 寝るからね、お休みなさいzzz
55132人目の素数さん:2012/05/06(日) 22:49:42.43
56132人目の素数さん:2012/05/06(日) 22:49:58.68
>>52
やってる事は正しいけど
(1)の答えはa=±2なの?
a=-2を元の2次方程式に入れたときの解と
a=+2を元の2次方程式に入れたときの解を
もう一度確認した方がいい。
57132人目の素数さん:2012/05/06(日) 22:53:30.88
>>52
(1)は問題文をよく読め
(2)は合ってると思う
(3)はそれだと2つの解が負と0の場合も含まれる
58132人目の素数さん:2012/05/06(日) 22:54:24.79
問題文を書き間違っているんだろ
59132人目の素数さん:2012/05/06(日) 23:00:00.45
>>55
ありがとうございます(T_T)
60132人目の素数さん:2012/05/06(日) 23:06:42.97
2次の条件を満たす多項式A,Bを求めよ。
(1)Aをx^2+x-3で割ると、商が4x-1,余りが-13-5である。
(2)2x^3-3x^2+2x+8をBで割ると、商がx^2-2x+2,余りが6である。
>>22の問題に未だ手こづってます。終わりそうにありません(T_T)
61132人目の素数さん:2012/05/06(日) 23:11:46.08
>>60
(1)
AをPで割った商がQ, 余りがR
A=PQ+R
62132人目の素数さん:2012/05/06(日) 23:16:25.90
>>61
(1)A=(x^2+x-3)(4x-1)-13x-5
=4x^3+3x^2-26x-5
合ってますか
63132人目の素数さん:2012/05/06(日) 23:17:42.34
>>60
宿題をGWの最後までほっとくなよ
サービスだ

(1)
A=(x^2+x-3)*(4x-1) -13x-5
= 4x^3+4x^2-12x -x^2-x+3 -13x-5
= 4x^3 + 3x^2 - 26x - 2
(あまりが-13-5がわからん-13x-5で計算した)

(2)
2x^3-3x^2+2x+8 = B*(x^2-2x+2) + 6
B*(x^2-2x+2) = 2x^3-3x^2+2x+2
= 2x^3-4x^2+4x + x^2-2x+2
= 2x*(x^2-2x+2) + (x^2-2x+2)
= (2x+1)*(x^2-2x+2)

B=2x+1
64132人目の素数さん:2012/05/06(日) 23:21:38.60
>>62
惜しいですね
65132人目の素数さん:2012/05/06(日) 23:24:16.43
>>63
ほっといていません!!ただ、わからなくて困っていて・・・。
ありがとうございます!
>>64
はい・・・
66132人目の素数さん:2012/05/06(日) 23:26:13.35
3(1) 等式3x^2-2x-1=a(x+1)^2+b(x+1)+cがxについて恒等式であるとき、定数a,b,cの値を求めよ。
(2) 等式(k+1)x-(3k+2)y+2k+7=0がすべてのkに対して成り立つとき、定数x,yを求めよ。
(3)等式x^2+2x-1=a(x+3)+cがxについての恒等式であるように定数a,b,cを求めよ。
今ここらへんやってます・・・ 難しい・・・
67132人目の素数さん:2012/05/06(日) 23:29:57.28
>>66
(1)
まあ工夫のしようはあるがここは実直に右辺を展開して整理
両辺の係数を比較
なんで係数を比較するのかは教科書で恒等式のところをチェック
68132人目の素数さん:2012/05/06(日) 23:30:44.59
>>66
3(1)
3x^2-2x-1=a(x+1)^2+b(x+1)+c
a(x+1)^2+b(x+1)+c
= 3x^2-2x-1
= 3x^2+6x+3 -8x-4
= 3(x+1)^2 -8x-8 + 4
= 3(x+1)^2 -8(x+1) + 4
係数比較して
a=3,b=-8,c=4

3(2)
(k+1)x-(3k+2)y+2k+7=0
kについて整理して
k(x-3y+2) + x-2y+7 = 0
kに関わらず成り立つ条件は
x-3y+2=0
x-2y+7=0
これを解いて
x=-13
y=-5

3(3)
問題を正確に書け
69132人目の素数さん:2012/05/06(日) 23:32:07.05
>>67
比較するとしか書いてなくてなんで比較するのは書いてないです(T_T)
訳わからないです。詰みました(T_T)
70132人目の素数さん:2012/05/06(日) 23:33:46.95
途中抜けてました。すいません・・・
(3)等式x^2+2x-1=a(x+3)+b(x+3)+cがxについての恒等式であるように定数a,b,cを求めよ。
71132人目の素数さん:2012/05/06(日) 23:33:47.54
>>69
それが恒等式となるための条件だからです
72132人目の素数さん:2012/05/06(日) 23:34:21.19
>>70
もういちどかいた式をよく読み直そう
73132人目の素数さん:2012/05/06(日) 23:37:30.41
>>72
(3)等式x^2+2x-1=a(x+3)^2+b(x+3)+cがxについての恒等式であるように定数a,b,cを求めよ。
これで多分合ってます。こんなに親切にしてくれてるのに何度もすいません (T_T)
74132人目の素数さん:2012/05/06(日) 23:39:21.85
マルチすんなよ
75132人目の素数さん:2012/05/06(日) 23:41:17.58
>>74
?
マルチってなんですか?
76132人目の素数さん:2012/05/06(日) 23:43:37.00
su(3)ってなんですか
77132人目の素数さん:2012/05/06(日) 23:45:27.60
>>76
3次ユニタリ行列の群だろ?
78132人目の素数さん:2012/05/06(日) 23:46:06.75
>>75
79132人目の素数さん:2012/05/07(月) 00:17:42.86
          __ノ)-'´ ̄ ̄`ー- 、_
        , '´  _. -‐'''"二ニニ=-`ヽ、
      /   /:::::; -‐''"        `ーノ
     /   /:::::/           \
     /    /::::::/          | | |  |
     |   |:::::/ /     |  | | | |  |
      |   |::/ / / |  | ||  | | ,ハ .| ,ハ|
      |   |/ / / /| ,ハノ| /|ノレ,ニ|ル' 
     |   |  | / / レ',二、レ′ ,ィイ|゙/   私は只の数ヲタなんかとは付き合わないわ。
.     |   \ ∠イ  ,イイ|    ,`-' |      頭が良くて数学が出来てかっこいい人。それが必要条件よ。
     |     l^,人|  ` `-'     ゝ  |        さらに Ann.of Math に論文書けば十分条件にもなるわよ。
      |      ` -'\       ー'  人          一番嫌いなのは論文数を増やすためにくだらない論文を書いて
    |        /(l     __/  ヽ、           良い論文の出版を遅らせるお馬鹿な人。
     |       (:::::`‐-、__  |::::`、     ヒニニヽ、         あなたの論文が Ann of Math に accept される確率は?
    |      / `‐-、::::::::::`‐-、::::\   /,ニニ、\            それとも最近は Inv. Math. の方が上かしら?
   |      |::::::::::::::::::|` -、:::::::,ヘ ̄|'、  ヒニ二、 \
.   |      /::::::::::::::::::|::::::::\/:::O`、::\   | '、   \
   |      /:::::::::::::::::::/:::::::::::::::::::::::::::::'、::::\ノ  ヽ、  |
  |      |:::::/:::::::::/:::::::::::::::::::::::::::::::::::'、',::::'、  /:\__/‐、
  |      |/:::::::::::/::::::::::::::::::::::::::::::::::O::| '、::| く::::::::::::: ̄|
   |     /_..-'´ ̄`ー-、:::::::::::::::::::::::::::::::::::|/:/`‐'::\;;;;;;;_|
   |    |/::::::::::::::::::::::\:::::::::::::::::::::::::::::|::/::::|::::/:::::::::::/
    |   /:::::::::::::::::::::::::::::::::|:::::::::::::::::::::O::|::|::::::|:::::::::::::::/
80132人目の素数さん:2012/05/07(月) 00:24:47.78
(3)等式x^2+2x-1=a(x+3)^2+b(x+3)+cがxについての恒等式であるように定数a,b,cを求めよ。
わかる人いませんかー
81132人目の素数さん:2012/05/07(月) 00:29:56.08
>>73
もうヒントはやらんぞ
x^2+2x-1=a(x+3)^2+b(x+3)+c
= ax^2 + (6a+b)x + 9a+3b+c
xの次数が同じ各項を係数比較して
a=1
6a+b=2
9a+3b+c=-1
この3元1次方程式を解く

又は
a(x+3)^2+b(x+3)+c = x^2+2x-1
= x^2+6x+9 -4x-10
= (x+3)^2 -4x-12 + 2
= (x+3)^2 -4(x+3) + 2
係数比較して
a=1
b=-4
c=2

または
a(x+3)^2+b(x+3)+c = x^2+2x-1
においてxがどのような数字であっても成り立つから
x=-4,-3,-2を代入して
a-b+c=-25
c=2
a+b+c=-1
この3元1次方程式を解く

好きな方法を選べ
82132人目の素数さん:2012/05/07(月) 00:30:29.11
>>80
(1)が理解できたら(3)は計算量が増えただけ
83132人目の素数さん:2012/05/07(月) 00:32:49.07
>>82
応用問題なんですかね?教科書にパターン載ってなかったです。
84132人目の素数さん:2012/05/07(月) 00:34:31.91
>>81
ありがとうございます。難しいですね・・・
85132人目の素数さん:2012/05/07(月) 00:35:17.47
応用じゃなくて計算量が増えただけ
3桁同士の足し算が5桁同士の足し算になったようなもの
86132人目の素数さん:2012/05/07(月) 00:37:15.26
いままでやってこなかったからできないんだろ?
それとも自分は特別だから何もせずともできるようになると思ってた?
87132人目の素数さん:2012/05/07(月) 00:39:12.44
>>85
わかりやすい説明ですね、さすがです。
でもそういうのを応用って言うんじゃないんですか?
まぁ・・・納得です。。。
88132人目の素数さん:2012/05/07(月) 00:40:37.11
よく進級できたな
2次関数とかほとんどできなかっただろ?
89132人目の素数さん:2012/05/07(月) 00:40:42.16
>>86
なんというか元の頭が悪くて、計算ミスしまくりで途中からわけワカメになるんですよね。
許してください。
90132人目の素数さん:2012/05/07(月) 00:41:13.73
許します
91132人目の素数さん:2012/05/07(月) 00:41:47.14
>>88
成績上位者でした。
2次関数めっちゃ得意でした(過去形
92132人目の素数さん:2012/05/07(月) 00:45:14.00
もう宿題も終わりだろ(あっても忘れましたと言え)
寝て明日に備えな
93132人目の素数さん:2012/05/07(月) 00:46:32.33
>>92
いや、まだあるんで自力でやります。
94132人目の素数さん:2012/05/07(月) 00:48:45.68
一応問題載せときます。

1、次の等式を証明せよ。
(1) (a+b)(a^3+b^3)-(a^2+b^2)^2=ab(a-b)^2
(2) (a^2-b^2)(c^2-b^2)=(ac+bd)^2-(ad+bc)^2
2、a+b+c=0のとき、次の等式が成り立つことを証明せよ。

a^2-bc=b^2-ca

3、a>b,c>dのとき、次の不等式が成り立つことを証明せよ。
(1) a+c>b+d
(2)ac+bd>ad+bc
95132人目の素数さん:2012/05/07(月) 00:49:29.39
文系でちゃんと進学を考えていて数学が必要なら黄チャートをちゃんとやりましょう
白チャートに逃げたりせず粘りつづけましょう
96132人目の素数さん:2012/05/07(月) 00:52:47.22
>>95
文系で、数学を受験科目として使う気は全くないです。
黄色チャートどころか白チャートも難しいと思うレベルです。

97132人目の素数さん:2012/05/07(月) 00:52:55.99
>>94
1(1)展開展開
1(2)展開展開
2
c=-a-bを
代入代入
3(1)左辺-右辺
3(2)左辺-右辺

よかった簡単で
98132人目の素数さん:2012/05/07(月) 00:54:44.01
>>96
なら赤がおすすめ
99132人目の素数さん:2012/05/07(月) 00:56:55.78
>>98
お金の無駄なんでやめときます^^
100132人目の素数さん:2012/05/07(月) 01:03:59.06
>>97
簡単ですね、でも、何がなんだかわからないです。
101132人目の素数さん:2012/05/07(月) 01:06:34.47
展開すら出来ないとは
102132人目の素数さん:2012/05/07(月) 01:08:40.89
>>101
めんどくさがり屋ですいません。自力でやります。
103132人目の素数さん:2012/05/07(月) 01:27:59.67
1+1/2^2+1/3^2+...+1/n^2 の値って求められまっか?nは自然数です
104132人目の素数さん:2012/05/07(月) 01:28:41.21
          __ノ)-'´ ̄ ̄`ー- 、_
        , '´  _. -‐'''"二ニニ=-`ヽ、
      /   /:::::; -‐''"        `ーノ
     /   /:::::/           \
     /    /::::::/          | | |  |
     |   |:::::/ /     |  | | | |  |
      |   |::/ / / |  | ||  | | ,ハ .| ,ハ|
      |   |/ / / /| ,ハノ| /|ノレ,ニ|ル' 
     |   |  | / / レ',二、レ′ ,ィイ|゙/   私は只の数ヲタなんかとは付き合わないわ。
.     |   \ ∠イ  ,イイ|    ,`-' |      頭が良くて数学が出来てかっこいい人。それが必要条件よ。
     |     l^,人|  ` `-'     ゝ  |        さらに Ann.of Math に論文書けば十分条件にもなるわよ。
      |      ` -'\       ー'  人          一番嫌いなのは論文数を増やすためにくだらない論文を書いて
    |        /(l     __/  ヽ、           良い論文の出版を遅らせるお馬鹿な人。
     |       (:::::`‐-、__  |::::`、     ヒニニヽ、         あなたの論文が Ann of Math に accept される確率は?
    |      / `‐-、::::::::::`‐-、::::\   /,ニニ、\            それとも最近は Inv. Math. の方が上かしら?
   |      |::::::::::::::::::|` -、:::::::,ヘ ̄|'、  ヒニ二、 \
.   |      /::::::::::::::::::|::::::::\/:::O`、::\   | '、   \
   |      /:::::::::::::::::::/:::::::::::::::::::::::::::::'、::::\ノ  ヽ、  |
  |      |:::::/:::::::::/:::::::::::::::::::::::::::::::::::'、',::::'、  /:\__/‐、
  |      |/:::::::::::/::::::::::::::::::::::::::::::::::O::| '、::| く::::::::::::: ̄|
   |     /_..-'´ ̄`ー-、:::::::::::::::::::::::::::::::::::|/:/`‐'::\;;;;;;;_|
   |    |/::::::::::::::::::::::\:::::::::::::::::::::::::::::|::/::::|::::/:::::::::::/
    |   /:::::::::::::::::::::::::::::::::|:::::::::::::::::::::O::|::|::::::|:::::::::::::::/
105132人目の素数さん:2012/05/07(月) 01:28:49.58
>>103
求められるけど
高校生にはちょっとだけ大変
106132人目の素数さん:2012/05/07(月) 01:33:54.92
あー終わった
合ってるかどうか分からないんですけど・・・
107132人目の素数さん:2012/05/07(月) 01:34:28.68
2
証明したい式より
左辺-右辺=a^2-bc-b^2+ca
=(a+b)(a-b)+c(a-b)
=(a-b)(a+b+c)
仮定よりa+b+c=0なので
(a-b)(a+b+c)=0
よって左辺=右辺

3
(1)仮定よりa-b>0、c-d>0
よってa-b+c-d>
したがってa+c>b+d

(2)左辺-右辺=ac+bd-ad-bc
=a(c-d))-b(c-d)
=(a-b)(c-d)
仮定よりa-b>0、c-d>0なので
(a-b)(c-d)>0
108132人目の素数さん:2012/05/07(月) 01:36:28.76
>>107
いいです
109132人目の素数さん:2012/05/07(月) 01:38:52.14
>>105
となると
(さっきの式)≦2-1/n を数学的帰納法で示すのが目的なのですが、
両辺の差をとったやり方ではできませんかね?
110132人目の素数さん:2012/05/07(月) 01:40:15.39
>>108
ただめんどくさかっただけなんで^^
すいませんね。答え合せして頂いてありがとうございました。
111132人目の素数さん:2012/05/07(月) 01:41:29.36
区分求積法のときのような感じでいけるんじゃないの?
112132人目の素数さん:2012/05/07(月) 01:45:08.26
          __ノ)-'´ ̄ ̄`ー- 、_
        , '´  _. -‐'''"二ニニ=-`ヽ、
      /   /:::::; -‐''"        `ーノ
     /   /:::::/           \
     /    /::::::/          | | |  |
     |   |:::::/ /     |  | | | |  |
      |   |::/ / / |  | ||  | | ,ハ .| ,ハ|
      |   |/ / / /| ,ハノ| /|ノレ,ニ|ル' 
     |   |  | / / レ',二、レ′ ,ィイ|゙/   私は只の数ヲタなんかとは付き合わないわ。
.     |   \ ∠イ  ,イイ|    ,`-' |      頭が良くて数学が出来てかっこいい人。それが必要条件よ。
     |     l^,人|  ` `-'     ゝ  |        さらに Ann.of Math に論文書けば十分条件にもなるわよ。
      |      ` -'\       ー'  人          一番嫌いなのは論文数を増やすためにくだらない論文を書いて
    |        /(l     __/  ヽ、           良い論文の出版を遅らせるお馬鹿な人。
     |       (:::::`‐-、__  |::::`、     ヒニニヽ、         あなたの論文が Ann of Math に accept される確率は?
    |      / `‐-、::::::::::`‐-、::::\   /,ニニ、\            それとも最近は Inv. Math. の方が上かしら?
   |      |::::::::::::::::::|` -、:::::::,ヘ ̄|'、  ヒニ二、 \
.   |      /::::::::::::::::::|::::::::\/:::O`、::\   | '、   \
   |      /:::::::::::::::::::/:::::::::::::::::::::::::::::'、::::\ノ  ヽ、  |
  |      |:::::/:::::::::/:::::::::::::::::::::::::::::::::::'、',::::'、  /:\__/‐、
  |      |/:::::::::::/::::::::::::::::::::::::::::::::::O::| '、::| く::::::::::::: ̄|
   |     /_..-'´ ̄`ー-、:::::::::::::::::::::::::::::::::::|/:/`‐'::\;;;;;;;_|
   |    |/::::::::::::::::::::::\:::::::::::::::::::::::::::::|::/::::|::::/:::::::::::/
    |   /:::::::::::::::::::::::::::::::::|:::::::::::::::::::::O::|::|::::::|:::::::::::::::/
113132人目の素数さん:2012/05/07(月) 02:09:25.11
>>109
数学的帰納法で示すってことは、(さっきの式)=a(n)、(右辺)=b(n) とおいて
・a(1)≦b(1) を示す
・a(n)≦b(n) を仮定して a(n+1)≦b(n+1) を示す
をそのまんまやればいいんは?
114132人目の素数さん:2012/05/07(月) 02:10:04.32
>>109の問題
Σ[k=1,n]1/k^2 が求まらなくても答え出るのですか?
115132人目の素数さん:2012/05/07(月) 02:11:01.98
>>113 そ れ か /(^o^)\ナンテコッタイ
116132人目の素数さん:2012/05/07(月) 02:20:53.65
n = k+1のとき
1+1/2^2+...+1/k^2 + 1/(k+1)^2 ≦ 2-1/k + 1/(k+1)^2
これで右辺を 2-1/k+1 にすればええんですよね?
まとまる気がしないのですが何か間違ってますかね;
117132人目の素数さん:2012/05/07(月) 02:29:54.90
だから
左辺<1+∫[1, n] 1/x^2 dx =1+1-1/n
だっていってんだろうが
118132人目の素数さん:2012/05/07(月) 02:40:18.55
>>117
だから〜言われてもIDないから誰が何言ってるのか分からんですわすみません。

Σ[k=1,n]1/k^2 と ∫[1, n] 1/x^2 dx って同値なんですか?
119132人目の素数さん:2012/05/07(月) 02:45:58.92
>>36
a/b÷c/d=(a/b)/(c/d)
分母分子にbdかけて
=ad/cb=(ab)*(d/c)
120132人目の素数さん:2012/05/07(月) 02:48:18.38
>>118
階段状の幅1の長方形群
これで分からないなら教科書や参考書調べてください
121132人目の素数さん:2012/05/07(月) 02:48:35.90
数3のほうね
122132人目の素数さん:2012/05/07(月) 02:54:01.66
>>116
>>113で a(n+1)-a(n)≦b(n+1)-b(n) が示せれば
>・a(n)≦b(n) を仮定して a(n+1)≦b(n+1) を示す
はできる。>>117は数学的帰納法を使ってないと言われるかも。
123132人目の素数さん:2012/05/07(月) 02:56:40.47
>>120
区分求積法ですよね?概念はおおよそ把握したと思います
シグマよかインテグラルを使うとあっという間ですね。

感謝感謝。ご協力いただいた皆様ありがとうございました!
124132人目の素数さん:2012/05/07(月) 02:58:32.92
○○ですよね?は誤解フラグ
125132人目の素数さん:2012/05/07(月) 03:01:43.42
>>111
みて気づいてない時点で分かってないんじゃ?
126132人目の素数さん:2012/05/07(月) 03:02:35.58
このプリントを全部やってくれるような神様がいらしたらお願いします
http://i.imgur.com/wDN53.jpg
http://i.imgur.com/SUqj1.jpg

どうか助けてください
127132人目の素数さん:2012/05/07(月) 03:05:56.51
まだ1年も余裕あるじゃん
128132人目の素数さん:2012/05/07(月) 03:13:16.83
2-1/k + 1/(k+1)^2<2-1/k +1/k(k+1)
<2-1/(k+1)
129132人目の素数さん:2012/05/07(月) 05:43:44.59
>>126
そういうのはヒマな人が多い土曜の夜に貼らないと。
130132人目の素数さん:2012/05/07(月) 08:50:24.29
          __ノ)-'´ ̄ ̄`ー- 、_
        , '´  _. -‐'''"二ニニ=-`ヽ、
      /   /:::::; -‐''"        `ーノ
     /   /:::::/           \
     /    /::::::/          | | |  |
     |   |:::::/ /     |  | | | |  |
      |   |::/ / / |  | ||  | | ,ハ .| ,ハ|
      |   |/ / / /| ,ハノ| /|ノレ,ニ|ル' 
     |   |  | / / レ',二、レ′ ,ィイ|゙/   私は只の数ヲタなんかとは付き合わないわ。
.     |   \ ∠イ  ,イイ|    ,`-' |      頭が良くて数学が出来てかっこいい人。それが必要条件よ。
     |     l^,人|  ` `-'     ゝ  |        さらに Ann.of Math に論文書けば十分条件にもなるわよ。
      |      ` -'\       ー'  人          一番嫌いなのは論文数を増やすためにくだらない論文を書いて
    |        /(l     __/  ヽ、           良い論文の出版を遅らせるお馬鹿な人。
     |       (:::::`‐-、__  |::::`、     ヒニニヽ、         あなたの論文が Ann of Math に accept される確率は?
    |      / `‐-、::::::::::`‐-、::::\   /,ニニ、\            それとも最近は Inv. Math. の方が上かしら?
   |      |::::::::::::::::::|` -、:::::::,ヘ ̄|'、  ヒニ二、 \
.   |      /::::::::::::::::::|::::::::\/:::O`、::\   | '、   \
   |      /:::::::::::::::::::/:::::::::::::::::::::::::::::'、::::\ノ  ヽ、  |
  |      |:::::/:::::::::/:::::::::::::::::::::::::::::::::::'、',::::'、  /:\__/‐、
  |      |/:::::::::::/::::::::::::::::::::::::::::::::::O::| '、::| く::::::::::::: ̄|
   |     /_..-'´ ̄`ー-、:::::::::::::::::::::::::::::::::::|/:/`‐'::\;;;;;;;_|
   |    |/::::::::::::::::::::::\:::::::::::::::::::::::::::::|::/::::|::::/:::::::::::/
    |   /:::::::::::::::::::::::::::::::::|:::::::::::::::::::::O::|::|::::::|:::::::::::::::/
131132人目の素数さん:2012/05/07(月) 09:59:48.28
>>129
へー土曜の夜だとやってくれる神様が現れるのかな?

自分の子供でも無い限り昼間だって全部はやらない。
132132人目の素数さん:2012/05/07(月) 13:43:05.74
(X1-X0) / sqrt(X0*X1)^2
これを変形すると
(1/X0) - (1/X1)
になるらしいんですが計算過程がわからないです
133132人目の素数さん:2012/05/07(月) 14:24:16.75
X0>0、X1>0、X0>X1 とかの条件ないなら絶対値も考えないとアカンだろ。
最近は√a^2 =|a|・・・怖い、ってコピーなくなったのかな。

まぁ良いけど、その辺り端折って大雑把に計算するから
sqrt(X0*X1)^2 は2乗してルートなのでほぼ逆演算 で、X0*X1 が出てくる
sqrt(X0*X1)^2 = X0*X1

すなわち 与式=(X1-X0) / X0*X1 = (X1 / (X0*X1)) -(X0 /( X0*X1))
約分して (1/X0) - (1/X1) になる場合もあるかも。
絶対値も考えて、もう少し緻密な計算をしてくれ、検討を祈る!
134132人目の素数さん:2012/05/07(月) 16:18:51.76
自分は習った事がないのですが、素数か否かの判定法で
「ある数値の各桁の和を3で割って、割り切れなかったら素数、割り切れたら素数」
というものがあるそうですが
この素数判定方法の名称などあれば、教えていただけませんか?
135132人目の素数さん:2012/05/07(月) 16:37:37.04
15は素数
136132人目の素数さん:2012/05/07(月) 16:41:00.66
すみません、割り切れなかったら素数、割り切れたら非素数、です。
137132人目の素数さん:2012/05/07(月) 16:42:47.61
>>136
それは3で割り切れるかどうかだけの判定
138132人目の素数さん:2012/05/07(月) 16:43:42.72
8は3で割り切れないから素数か…
139132人目の素数さん:2012/05/07(月) 16:47:55.10
2桁以上で成り立つようです。
140132人目の素数さん:2012/05/07(月) 16:58:38.36
偶数と5の倍数を除けば、成り立つな
141132人目の素数さん:2012/05/07(月) 17:10:40.82
>>139
49は素数?
142132人目の素数さん:2012/05/07(月) 17:19:13.06
偶数と5の倍数と冪乗を除けば成り立つな
143132人目の素数さん:2012/05/07(月) 17:21:59.73
77、 91
144132人目の素数さん:2012/05/07(月) 18:16:40.04
jyg
145132人目の素数さん:2012/05/07(月) 19:18:43.36
いくつかの林檎を何人かの子供に分けたい。今、子供1人に三個ずつとすると全体で8個あまり、子供1人に5個ずつとすると、最後に1人だけ、5個に足りないという。子供の人数と林檎の数を求めよ

不等式ですが、式がうまく立てれません
146132人目の素数さん:2012/05/07(月) 19:23:19.03
中国人に聞け
147132人目の素数さん:2012/05/07(月) 19:27:00.73
>>145
未知数に文字を割り当て、与えられた条件をその文字を使った式で表す
148132人目の素数さん:2012/05/07(月) 19:32:29.85
>>145
それは中学レベルの(連立)方程式で解けると思うけど。

子供の人数を x にして立式しなさい。

検討を祈る!・・・因みにx=5かな。
149132人目の素数さん:2012/05/07(月) 19:32:33.17
>>134-143
「アドホックな仮説」という言葉がしっくりくるやりとりだなぁ
150132人目の素数さん:2012/05/07(月) 20:55:42.28
          __ノ)-'´ ̄ ̄`ー- 、_
        , '´  _. -‐'''"二ニニ=-`ヽ、
      /   /:::::; -‐''"        `ーノ
     /   /:::::/           \
     /    /::::::/          | | |  |
     |   |:::::/ /     |  | | | |  |
      |   |::/ / / |  | ||  | | ,ハ .| ,ハ|
      |   |/ / / /| ,ハノ| /|ノレ,ニ|ル' 
     |   |  | / / レ',二、レ′ ,ィイ|゙/   私は只の数ヲタなんかとは付き合わないわ。
.     |   \ ∠イ  ,イイ|    ,`-' |      頭が良くて数学が出来てかっこいい人。それが必要条件よ。
     |     l^,人|  ` `-'     ゝ  |        さらに Ann.of Math に論文書けば十分条件にもなるわよ。
      |      ` -'\       ー'  人          一番嫌いなのは論文数を増やすためにくだらない論文を書いて
    |        /(l     __/  ヽ、           良い論文の出版を遅らせるお馬鹿な人。
     |       (:::::`‐-、__  |::::`、     ヒニニヽ、         あなたの論文が Ann of Math に accept される確率は?
    |      / `‐-、::::::::::`‐-、::::\   /,ニニ、\            それとも最近は Inv. Math. の方が上かしら?
   |      |::::::::::::::::::|` -、:::::::,ヘ ̄|'、  ヒニ二、 \
.   |      /::::::::::::::::::|::::::::\/:::O`、::\   | '、   \
   |      /:::::::::::::::::::/:::::::::::::::::::::::::::::'、::::\ノ  ヽ、  |
  |      |:::::/:::::::::/:::::::::::::::::::::::::::::::::::'、',::::'、  /:\__/‐、
  |      |/:::::::::::/::::::::::::::::::::::::::::::::::O::| '、::| く::::::::::::: ̄|
   |     /_..-'´ ̄`ー-、:::::::::::::::::::::::::::::::::::|/:/`‐'::\;;;;;;;_|
   |    |/::::::::::::::::::::::\:::::::::::::::::::::::::::::|::/::::|::::/:::::::::::/
    |   /:::::::::::::::::::::::::::::::::|:::::::::::::::::::::O::|::|::::::|:::::::::::::::/
151132人目の素数さん:2012/05/07(月) 21:49:11.09
三角形ABCで角A、B、Cが変化するときcosAcosBcosCのとり得る値の範囲をもとめよ。

cosAcosBcosC=Pとおく。
A:最小角(0<A≦Π/3)としてAを固定したとき
-1/2{(cosA)^2+cosAcos3A}≦P≦-1/2{(cosA)^2-cosA
である。

どうしてAを固定するとこの不等式が出るのかわかりません。
152132人目の素数さん:2012/05/07(月) 21:50:25.54
P≦-1/2{(cosA)^2-cosA}です

中カッコ点け忘れました。
153132人目の素数さん:2012/05/07(月) 21:50:36.21
正方行列では、交換法則が成り立たないだけで正方行列以外では普通に成り立ちますよね?
154132人目の素数さん:2012/05/07(月) 21:52:42.46
a,bを整数とする。pをa^3-b^3を満たす素数とする。このとき、p-1が6の倍数であることを示せ。

という問題がわかりません。
155132人目の素数さん:2012/05/07(月) 21:58:39.43
(a-b)(a^2+ab+b^2)
156132人目の素数さん:2012/05/07(月) 22:09:34.92
>>151
まずはcosAcosBcosCのcosBcosCに積から和の公式を当てはめればいいんじゃないかな
157132人目の素数さん:2012/05/07(月) 22:16:42.83
>>153
何の交換法則か知らないが、適当に例を作って試せばわかるんでは
158132人目の素数さん:2012/05/07(月) 22:18:43.37
          __ノ)-'´ ̄ ̄`ー- 、_
        , '´  _. -‐'''"二ニニ=-`ヽ、
      /   /:::::; -‐''"        `ーノ
     /   /:::::/           \
     /    /::::::/          | | |  |
     |   |:::::/ /     |  | | | |  |
      |   |::/ / / |  | ||  | | ,ハ .| ,ハ|
      |   |/ / / /| ,ハノ| /|ノレ,ニ|ル' 
     |   |  | / / レ',二、レ′ ,ィイ|゙/   私は只の数ヲタなんかとは付き合わないわ。
.     |   \ ∠イ  ,イイ|    ,`-' |      頭が良くて数学が出来てかっこいい人。それが必要条件よ。
     |     l^,人|  ` `-'     ゝ  |        さらに Ann.of Math に論文書けば十分条件にもなるわよ。
      |      ` -'\       ー'  人          一番嫌いなのは論文数を増やすためにくだらない論文を書いて
    |        /(l     __/  ヽ、           良い論文の出版を遅らせるお馬鹿な人。
     |       (:::::`‐-、__  |::::`、     ヒニニヽ、         あなたの論文が Ann of Math に accept される確率は?
    |      / `‐-、::::::::::`‐-、::::\   /,ニニ、\            それとも最近は Inv. Math. の方が上かしら?
   |      |::::::::::::::::::|` -、:::::::,ヘ ̄|'、  ヒニ二、 \
.   |      /::::::::::::::::::|::::::::\/:::O`、::\   | '、   \
   |      /:::::::::::::::::::/:::::::::::::::::::::::::::::'、::::\ノ  ヽ、  |
  |      |:::::/:::::::::/:::::::::::::::::::::::::::::::::::'、',::::'、  /:\__/‐、
  |      |/:::::::::::/::::::::::::::::::::::::::::::::::O::| '、::| く::::::::::::: ̄|
   |     /_..-'´ ̄`ー-、:::::::::::::::::::::::::::::::::::|/:/`‐'::\;;;;;;;_|
   |    |/::::::::::::::::::::::\:::::::::::::::::::::::::::::|::/::::|::::/:::::::::::/
    |   /:::::::::::::::::::::::::::::::::|:::::::::::::::::::::O::|::|::::::|:::::::::::::::/
159132人目の素数さん:2012/05/07(月) 22:27:42.19
>>156
ありがとうございます。
右端は示せたけど左端のcos(B-C)の下限がわからない。
A+B+C=ΠとAの値の範囲で示せるのか…
160132人目の素数さん:2012/05/07(月) 22:30:13.32
>>155
どうもです。因数分解までは気づいたけどその先が・・
161132人目の素数さん:2012/05/07(月) 22:30:46.03
定数cはc>1とする。xy平面上で点(1,c)を通る直線lと放物線C:y=x^2で囲まれる図形の面積Sを最小にするlの傾きを求めよ。また、その最小面積を求めよ。

よろしくお願いします
162132人目の素数さん:2012/05/07(月) 22:35:49.30
>>160
pをa^3-b^3を満たす素数とする
って何?
163132人目の素数さん:2012/05/07(月) 22:42:25.57
>>162
問題文「pを」→「pは」だったかも。問題文がうろ覚えですみません。
164132人目の素数さん:2012/05/07(月) 22:44:17.98
ならなおさら
p=a^3-b^3=(a-b)(a^2+ab+b^2)   (a,b;整数)
と因数分解できて
素数にならずおかしいとおもうんだ・・私
165132人目の素数さん:2012/05/07(月) 22:47:07.56
それなら、やっぱり問題がおかしいんでしょうかね。色々試したけどだめだったもので。
166132人目の素数さん:2012/05/07(月) 22:47:12.45
>>164
お前がバカだから
167132人目の素数さん:2012/05/07(月) 22:47:37.61
7=2^3-1^3
168132人目の素数さん:2012/05/07(月) 22:48:50.65
△ABCで辺BC,CA,ABを3:1に内分する点をそれぞれP,Q,Rとするとき AP↑+BQ↑+CR↑=0であることを示せ

という問題なのですが この3つのベクトルをAB↑ AC↑で表したいのですがどうやればいいのかわかりません

教えていただけませんか?
169132人目の素数さん:2012/05/07(月) 22:49:30.29
あっ。もしかして、a-bが1なのでa^2+ab+b^2=pですね。
170132人目の素数さん:2012/05/07(月) 22:57:38.90
>>154
問題間違ってないか?
a=1、b=-1のときp=2となって素数だから条件を満たすが、p-1=1で6の倍数にならない。
171132人目の素数さん:2012/05/07(月) 22:59:15.22
>>168
教科書読め
172132人目の素数さん:2012/05/07(月) 23:05:59.52
>>159
B-C=A+2B-π から cos(B-C)=-cos(A+2B)
A<π/3<B< から 3A<A+2B
でいけない?ちゃんと計算はしてないけど
173132人目の素数さん:2012/05/07(月) 23:11:35.56
>>172
違うやり方で一応示せたけど、こっちの方が簡単ですね。
ありがとうございます
174132人目の素数さん:2012/05/07(月) 23:19:15.48
x+y=5,xy=1のときx^2+y^2はいくらになるか解き方を教えてください
175132人目の素数さん:2012/05/07(月) 23:20:06.94
x^2+y^2=(x+y)^2-2xy
176132人目の素数さん:2012/05/07(月) 23:20:10.24
>>174
(x+y)^2=
177132人目の素数さん:2012/05/07(月) 23:20:43.30
教科書レベルだろ
178132人目の素数さん:2012/05/07(月) 23:22:29.71
>>171
よくわからないんです

内分点の公式を使って表したものの
なんか全然違うような気がしまして…
179132人目の素数さん:2012/05/07(月) 23:34:26.45
(x-3)^3*(x+3)^3を簡単に解く方法を教えてください
180132人目の素数さん:2012/05/07(月) 23:38:12.05
>>178
AP = AB+(3/4)BC = AB+(3/4)(-AB+AC)
BQ = BC+(3/4)CA = -AB+AC-(3/4)AC
CR = CA+(3/4)AB = -AC+(3/4)AB

公式使うのも良いけど使わなくても
導けるような方法くらいは考えた方がいい。
お小言っぽく感じたらごめんなさい。
181132人目の素数さん:2012/05/07(月) 23:40:43.70
>>179
いっそ真っ向勝負で展開して力つけたら?
182132人目の素数さん:2012/05/07(月) 23:44:23.82
>>181
真っ向勝負で解いたんだがあってるかわかんないし、
簡単にできないかなと思い・・・
183132人目の素数さん:2012/05/07(月) 23:45:19.47
>>178
中学の幾何で作図して最後にちょっろとベクトル出して終わり
184132人目の素数さん:2012/05/07(月) 23:45:25.06
>>180
公式使ったもののぐちゃぐちゃになってわけわからなくなりました

こっちの方が簡単ですね
ありがとうございました
185132人目の素数さん:2012/05/07(月) 23:45:44.57
>>182
a^3*b^3=(ab)^3
186132人目の素数さん:2012/05/07(月) 23:49:26.27
>>185
じゃあ(x-3)^3*(x+3)^3を簡単にすると
{(x-3)(x+3)}^3ということ?
187132人目の素数さん:2012/05/08(火) 00:00:15.31
>161
お願いします
188132人目の素数さん:2012/05/08(火) 00:00:49.44
>>185
解決しましたありがとうございます
189132人目の素数さん:2012/05/08(火) 00:01:16.06
>>186
(x-3)^3*(x+3)^3を簡単にすると{(x-3)(x+3)}^3になるかは知らんが
{(x-3)(x+3)}^3にすれば計算は楽になるな
190132人目の素数さん:2012/05/08(火) 00:02:19.08
>>187
傾きmとおいて
(1, c)を通る直線lを表して
計算するだけ
191132人目の素数さん:2012/05/08(火) 00:21:40.14
x=cos3tcost、y=cos3tsintからtを消去しろという問題です
いろいろいじってみたのですがうまくいきません
教えていただけませんか?
192132人目の素数さん:2012/05/08(火) 00:22:15.22
x^2+y^2=4とy=kx+4が異なる二点P,Qで交わっているときPQの中点Mの軌跡を示せという問題
M=(p,q)とおく
x^2+(kx+4)^2=4から(1+k^2)x^2+8kx+12=0
この方程式の2解をα,βとしてpを求めると
p=-4k/(1+k^2)
またMはy=kx+4上にあるのでq=kp+4
p≠0であるからk=(q-4)/p
これをpの式に代入して整理するとp^2+(q-2)^2=4

ここまでは分かったのですがkの範囲はk<-√3,√3<kなのでp,qの範囲を考えないといけない
3<k^2からpの範囲が絞れそうですが分子のkが邪魔で出来ません
ここからの進め方を教えて下さい
193132人目の素数さん:2012/05/08(火) 00:26:28.95
>161
最小面積はcを使って表していいんですかね?
194132人目の素数さん:2012/05/08(火) 00:30:04.10
√(7-2√12)の2重根号をはずして簡単にしなさい
これの解き方教えてください
195132人目の素数さん:2012/05/08(火) 00:32:09.99
>>193
おk
196132人目の素数さん:2012/05/08(火) 00:32:45.18
>>191
取り敢えず、y/x=tant ぐらいはやってみた?
197132人目の素数さん:2012/05/08(火) 00:33:46.72
>>194
教科書
198132人目の素数さん:2012/05/08(火) 00:35:29.53
>>192
ヒント
1/pの値の範囲
kが正か負かにも注意
199132人目の素数さん:2012/05/08(火) 00:38:18.68
>>196
はい
あと3倍角の公式とか適当に2乗とかもしてみたんですが
200132人目の素数さん:2012/05/08(火) 00:40:51.60
>>191
本当にtを消去する問題だったか?
結構計算大変だぞ?
201132人目の素数さん:2012/05/08(火) 00:44:24.54
>>200
はい
xとyだけで表してみろ、と
202132人目の素数さん:2012/05/08(火) 00:50:23.98
>>201
(x^2+y^2)^2=x(3x^2-y^2)
203132人目の素数さん:2012/05/08(火) 00:57:09.46
>>202
,(x^2+y^2)^2=x(x^2-3y^2)
204132人目の素数さん:2012/05/08(火) 01:05:04.05
検算してみました
>>203のほうですね
ありがとうございます

こういうのうまく導くコツとかってありますか?
205132人目の素数さん:2012/05/08(火) 01:05:15.86
>>154
できた、難しかったけど発想の転換だな、解ればナンダってレベル。

(p-1)が偶数は明らか。因数分解では行き詰まる。
あとは(p-1)が3の倍数であることを証明する。

前回は剰余の定理で証明したけど今回は他の方の証明を先に見たい。

さてと寝るとする。
206132人目の素数さん:2012/05/08(火) 01:16:48.98
>>198
出来ました!ありがとうございます
207132人目の素数さん:2012/05/08(火) 01:17:59.80
>>204
気になるなら三葉線でググるといい
数Cで媒介変数や極方程式で表されたいろいろな曲線がいくつか出てくる
多分入試で一番出てくるのはサイクロイド
数研の教科書だと図で載っていたと思う
208132人目の素数さん:2012/05/08(火) 01:21:38.96
>>205
p-1=3b(b+1)で終わりだろ
209132人目の素数さん:2012/05/08(火) 01:32:06.84
>>207
ありがとうございます
210132人目の素数さん:2012/05/08(火) 01:34:49.40
>195
ありがとうございます
211132人目の素数さん:2012/05/08(火) 03:08:46.05
>>191
x^2+y^2=(cos 3t)^2 と
1/(cos 3t)^2=((cos 3t)^2+(sin 3t)^2)/(cos 3t)^2=1+(tan 3t)^2
に気づくのがコツだろうな。
212132人目の素数さん:2012/05/08(火) 04:34:04.87
(1) 3x+8≧4x+2
(2) -(2x+1)<6x-1
(3) (2x+3)/2≦x-1

答えないから答え合わせ

お願いします
213132人目の素数さん:2012/05/08(火) 06:17:06.96
むっり〜
214132人目の素数さん:2012/05/08(火) 06:32:42.39
212ですが

自己解決しました
215132人目の素数さん:2012/05/08(火) 07:36:29.96
          __ノ)-'´ ̄ ̄`ー- 、_
        , '´  _. -‐'''"二ニニ=-`ヽ、
      /   /:::::; -‐''"        `ーノ
     /   /:::::/           \
     /    /::::::/          | | |  |
     |   |:::::/ /     |  | | | |  |
      |   |::/ / / |  | ||  | | ,ハ .| ,ハ|
      |   |/ / / /| ,ハノ| /|ノレ,ニ|ル' 
     |   |  | / / レ',二、レ′ ,ィイ|゙/   私は只の数ヲタなんかとは付き合わないわ。
.     |   \ ∠イ  ,イイ|    ,`-' |      頭が良くて数学が出来てかっこいい人。それが必要条件よ。
     |     l^,人|  ` `-'     ゝ  |        さらに Ann.of Math に論文書けば十分条件にもなるわよ。
      |      ` -'\       ー'  人          一番嫌いなのは論文数を増やすためにくだらない論文を書いて
    |        /(l     __/  ヽ、           良い論文の出版を遅らせるお馬鹿な人。
     |       (:::::`‐-、__  |::::`、     ヒニニヽ、         あなたの論文が Ann of Math に accept される確率は?
    |      / `‐-、::::::::::`‐-、::::\   /,ニニ、\            それとも最近は Inv. Math. の方が上かしら?
   |      |::::::::::::::::::|` -、:::::::,ヘ ̄|'、  ヒニ二、 \
.   |      /::::::::::::::::::|::::::::\/:::O`、::\   | '、   \
   |      /:::::::::::::::::::/:::::::::::::::::::::::::::::'、::::\ノ  ヽ、  |
  |      |:::::/:::::::::/:::::::::::::::::::::::::::::::::::'、',::::'、  /:\__/‐、
  |      |/:::::::::::/::::::::::::::::::::::::::::::::::O::| '、::| く::::::::::::: ̄|
   |     /_..-'´ ̄`ー-、:::::::::::::::::::::::::::::::::::|/:/`‐'::\;;;;;;;_|
   |    |/::::::::::::::::::::::\:::::::::::::::::::::::::::::|::/::::|::::/:::::::::::/
    |   /:::::::::::::::::::::::::::::::::|:::::::::::::::::::::O::|::|::::::|:::::::::::::::/
216132人目の素数さん:2012/05/08(火) 07:53:13.55
連休の課題なのか? 丸投げ連発だな
217132人目の素数さん:2012/05/08(火) 08:47:45.11
>>208
(a^3-b^3)と言う式からどうして 3b(b+1) になったか解りません。 
218132人目の素数さん:2012/05/08(火) 09:03:12.28
>>170はどうなったんだ?
a、bは正の整数なのか?
219132人目の素数さん:2012/05/08(火) 09:08:48.71
>>217
a、bが正の整数なら、(a-b)(a^2+ab+b^2)が素数であるためにはa-b=1。
つまり、a=b+1。これを代入。
>>205が因数分解では行き詰まると言っている理由はわからない。

a、bに正という条件がないなら>>170の言うとおり判例があって成り立たない。
220132人目の素数さん:2012/05/08(火) 10:30:30.58
>>219
レスTHX

>素数であるためにはa-b=1
なるほど。単に計算だけで因数分解して失敗してた。

>a、bに正という条件がないなら
確かにこれもそう。あるいは、ただし(p-1=1)を除くとか。

ところで尋ねたいことがある。 
この手の問題はヒントや解答を見ないで全部自力でできてるのかな?
この質問に応えてくれれば自分の証明を示す。

221132人目の素数さん:2012/05/08(火) 10:36:52.62
追伸
なるほど
a、bが正の整数との前提がなければ (a-b)(a^2+ab+b^2) で a^2+ab+b^2=1 も有りうる訳だ。
222132人目の素数さん:2012/05/08(火) 16:10:44.28
すべての正の整数整数nに対して、3^(2n-1)+2^(n+1)が
必ず7の倍数になることを証明してください
223132人目の素数さん:2012/05/08(火) 16:11:39.44
見たこと有る気がする
224132人目の素数さん:2012/05/08(火) 16:40:52.75
3^(2n-1)+2^(n+1)=3*(7+2)^(n-1)+4*2^(n-1)
225132人目の素数さん:2012/05/08(火) 17:55:32.51
>>222
「3^(2n-1)+2^(n+1) 7の倍数」でググれば答えが出てくる
余りを使うんだとさ
226132人目の素数さん:2012/05/08(火) 19:41:05.39
>>222
数学的帰納法の典型的問題と思うけど
227132人目の素数さん:2012/05/08(火) 20:09:37.34
>>222なんだが、これを解くと
n=1のとき
3^1+2^2=3+4=7
よって、n=1のときは成り立つ
n=kのとき
3^(2k-1)+2^(k+1)=7m (mは整数)・・・仮定
3^(2n-1)をA,2^(n+1)をBとおくと
n=k+1のときは、
9a+2b=7m´
=9a+2(7m-a)
=9a+14m-2a
=7a+14m
=7(2m+a)
これより
m´=2m+a
結果、題意は示された
これであってますか?
228132人目の素数さん:2012/05/08(火) 20:17:58.15
>>224から(7+2)^(n-1)を二項定理で展開すれば終わりでしょ>>222
229132人目の素数さん:2012/05/08(火) 20:20:46.42
>>227の解答ではだめなんですかね?>>228
230132人目の素数さん:2012/05/08(火) 20:26:51.57
>>229
大雑把なながれはいいんじゃね。
231132人目の素数さん:2012/05/08(火) 20:35:07.43
>>227
a, b, m´ が何か説明されてない。
232132人目の素数さん:2012/05/08(火) 20:41:31.67
>>227の式はミスでした、正しくは
n=1のとき
3^1+2^2=3+4=7
よって、n=1のときは成り立つ
n=kのとき
3^(2k-1)+2^(k+1)=7m (mは整数)・・・仮定
3^(2n-1)をa,2^(n+1)をbとおくと
n=k+1のときは、
9a+2b=7m´ (m´は整数)
=9a+2(7m-a)
=9a+14m-2a
=7a+14m
=7(2m+a)
これより
m´=2m+a
結果、7の倍数となり題意は示された
となると思います
233132人目の素数さん:2012/05/08(火) 20:51:06.19
>>232
> 9a+2b=7m´ (m´は整数)
(m´は整数) は仮定したのか、そこまでで証明したのか、その先で証明するのか
どれ?
234132人目の素数さん:2012/05/08(火) 20:56:12.87
>>233(m´は整数) は仮定して、その先で証明しました
235132人目の素数さん:2012/05/08(火) 21:08:31.90
>>234
証明することをその前に仮定しちゃいかんでしょ。それから
>3^(2n-1)をa,2^(n+1)をbとおくと
だと、n=k+1 のとき a=3^(2(k+1)-1), b=2^((k+1)+1) になるのでおかしい。
236132人目の素数さん:2012/05/08(火) 21:11:13.08
何と言うか3^(2n-1)=aのように無駄に置きかえしてるから
解りづらくなってる感じがする。
237132人目の素数さん:2012/05/08(火) 21:19:44.16
a(n)=3^(2n-1), b(n)=2^(n+1) とおいて
a(k+1)=9a(k), b(k+1)=2b(k) から
a(k+1)+b(k+1)=9a(k)+2b(k)=7a(k)+2(a(k)+b(k)) と変形して証明するのを
論理が混乱してる感じかな
238132人目の素数さん:2012/05/08(火) 21:19:51.65
>>235n=kのとき3^(2k-1)でそれをaとおき
n=k+1にすると3^(2k+1)となりそれを9*3^(2k-1)に変換し、
3^(2k-1)はaとおいたので、9aじゃないのですか?
239132人目の素数さん:2012/05/08(火) 21:20:42.25
          __ノ)-'´ ̄ ̄`ー- 、_
        , '´  _. -‐'''"二ニニ=-`ヽ、
      /   /:::::; -‐''"        `ーノ
     /   /:::::/           \
     /    /::::::/          | | |  |
     |   |:::::/ /     |  | | | |  |
      |   |::/ / / |  | ||  | | ,ハ .| ,ハ|
      |   |/ / / /| ,ハノ| /|ノレ,ニ|ル' 
     |   |  | / / レ',二、レ′ ,ィイ|゙/   私は只の数ヲタなんかとは付き合わないわ。
.     |   \ ∠イ  ,イイ|    ,`-' |      頭が良くて数学が出来てかっこいい人。それが必要条件よ。
     |     l^,人|  ` `-'     ゝ  |        さらに Ann.of Math に論文書けば十分条件にもなるわよ。
      |      ` -'\       ー'  人          一番嫌いなのは論文数を増やすためにくだらない論文を書いて
    |        /(l     __/  ヽ、           良い論文の出版を遅らせるお馬鹿な人。
     |       (:::::`‐-、__  |::::`、     ヒニニヽ、         あなたの論文が Ann of Math に accept される確率は?
    |      / `‐-、::::::::::`‐-、::::\   /,ニニ、\            それとも最近は Inv. Math. の方が上かしら?
   |      |::::::::::::::::::|` -、:::::::,ヘ ̄|'、  ヒニ二、 \
.   |      /::::::::::::::::::|::::::::\/:::O`、::\   | '、   \
   |      /:::::::::::::::::::/:::::::::::::::::::::::::::::'、::::\ノ  ヽ、  |
  |      |:::::/:::::::::/:::::::::::::::::::::::::::::::::::'、',::::'、  /:\__/‐、
  |      |/:::::::::::/::::::::::::::::::::::::::::::::::O::| '、::| く::::::::::::: ̄|
   |     /_..-'´ ̄`ー-、:::::::::::::::::::::::::::::::::::|/:/`‐'::\;;;;;;;_|
   |    |/::::::::::::::::::::::\:::::::::::::::::::::::::::::|::/::::|::::/:::::::::::/
    |   /:::::::::::::::::::::::::::::::::|:::::::::::::::::::::O::|::|::::::|:::::::::::::::/
240132人目の素数さん:2012/05/08(火) 21:27:27.62
>>238
それなら
3^(2k-1)をa,2^(k+1)をbとおくと
と書いた方が誤解を招かずにすむと思う
241132人目の素数さん:2012/05/08(火) 21:33:05.32
          __ノ)-'´ ̄ ̄`ー- 、_
        , '´  _. -‐'''"二ニニ=-`ヽ、
      /   /:::::; -‐''"        `ーノ
     /   /:::::/           \
     /    /::::::/          | | |  |
     |   |:::::/ /     |  | | | |  |
      |   |::/ / / |  | ||  | | ,ハ .| ,ハ|
      |   |/ / / /| ,ハノ| /|ノレ,ニ|ル' 
     |   |  | / / レ',二、レ′ ,ィイ|゙/   私は只の数ヲタなんかとは付き合わないわ。
.     |   \ ∠イ  ,イイ|    ,`-' |      頭が良くて数学が出来てかっこいい人。それが必要条件よ。
     |     l^,人|  ` `-'     ゝ  |        さらに Ann.of Math に論文書けば十分条件にもなるわよ。
      |      ` -'\       ー'  人          一番嫌いなのは論文数を増やすためにくだらない論文を書いて
    |        /(l     __/  ヽ、           良い論文の出版を遅らせるお馬鹿な人。
     |       (:::::`‐-、__  |::::`、     ヒニニヽ、         あなたの論文が Ann of Math に accept される確率は?
    |      / `‐-、::::::::::`‐-、::::\   /,ニニ、\            それとも最近は Inv. Math. の方が上かしら?
   |      |::::::::::::::::::|` -、:::::::,ヘ ̄|'、  ヒニ二、 \
.   |      /::::::::::::::::::|::::::::\/:::O`、::\   | '、   \
   |      /:::::::::::::::::::/:::::::::::::::::::::::::::::'、::::\ノ  ヽ、  |
  |      |:::::/:::::::::/:::::::::::::::::::::::::::::::::::'、',::::'、  /:\__/‐、
  |      |/:::::::::::/::::::::::::::::::::::::::::::::::O::| '、::| く::::::::::::: ̄|
   |     /_..-'´ ̄`ー-、:::::::::::::::::::::::::::::::::::|/:/`‐'::\;;;;;;;_|
   |    |/::::::::::::::::::::::\:::::::::::::::::::::::::::::|::/::::|::::/:::::::::::/
    |   /:::::::::::::::::::::::::::::::::|:::::::::::::::::::::O::|::|::::::|:::::::::::::::/
242132人目の素数さん:2012/05/08(火) 22:41:49.37
2乗、3乗とかってどうやってタイピングするんですか?
243132人目の素数さん:2012/05/08(火) 22:42:52.32
          __ノ)-'´ ̄ ̄`ー- 、_
        , '´  _. -‐'''"二ニニ=-`ヽ、
      /   /:::::; -‐''"        `ーノ
     /   /:::::/           \
     /    /::::::/          | | |  |
     |   |:::::/ /     |  | | | |  |
      |   |::/ / / |  | ||  | | ,ハ .| ,ハ|
      |   |/ / / /| ,ハノ| /|ノレ,ニ|ル' 
     |   |  | / / レ',二、レ′ ,ィイ|゙/   私は只の数ヲタなんかとは付き合わないわ。
.     |   \ ∠イ  ,イイ|    ,`-' |      頭が良くて数学が出来てかっこいい人。それが必要条件よ。
     |     l^,人|  ` `-'     ゝ  |        さらに Ann.of Math に論文書けば十分条件にもなるわよ。
      |      ` -'\       ー'  人          一番嫌いなのは論文数を増やすためにくだらない論文を書いて
    |        /(l     __/  ヽ、           良い論文の出版を遅らせるお馬鹿な人。
     |       (:::::`‐-、__  |::::`、     ヒニニヽ、         あなたの論文が Ann of Math に accept される確率は?
    |      / `‐-、::::::::::`‐-、::::\   /,ニニ、\            それとも最近は Inv. Math. の方が上かしら?
   |      |::::::::::::::::::|` -、:::::::,ヘ ̄|'、  ヒニ二、 \
.   |      /::::::::::::::::::|::::::::\/:::O`、::\   | '、   \
   |      /:::::::::::::::::::/:::::::::::::::::::::::::::::'、::::\ノ  ヽ、  |
  |      |:::::/:::::::::/:::::::::::::::::::::::::::::::::::'、',::::'、  /:\__/‐、
  |      |/:::::::::::/::::::::::::::::::::::::::::::::::O::| '、::| く::::::::::::: ̄|
   |     /_..-'´ ̄`ー-、:::::::::::::::::::::::::::::::::::|/:/`‐'::\;;;;;;;_|
   |    |/::::::::::::::::::::::\:::::::::::::::::::::::::::::|::/::::|::::/:::::::::::/
    |   /:::::::::::::::::::::::::::::::::|:::::::::::::::::::::O::|::|::::::|:::::::::::::::/
244132人目の素数さん:2012/05/08(火) 22:43:16.65
 = の右横
 ¥ の左横
のキー
245132人目の素数さん:2012/05/08(火) 22:43:31.34
          __ノ)-'´ ̄ ̄`ー- 、_
        , '´  _. -‐'''"二ニニ=-`ヽ、
      /   /:::::; -‐''"        `ーノ
     /   /:::::/           \
     /    /::::::/          | | |  |
     |   |:::::/ /     |  | | | |  |
      |   |::/ / / |  | ||  | | ,ハ .| ,ハ|
      |   |/ / / /| ,ハノ| /|ノレ,ニ|ル' 
     |   |  | / / レ',二、レ′ ,ィイ|゙/   私は只の数ヲタなんかとは付き合わないわ。
.     |   \ ∠イ  ,イイ|    ,`-' |      頭が良くて数学が出来てかっこいい人。それが必要条件よ。
     |     l^,人|  ` `-'     ゝ  |        さらに Ann.of Math に論文書けば十分条件にもなるわよ。
      |      ` -'\       ー'  人          一番嫌いなのは論文数を増やすためにくだらない論文を書いて
    |        /(l     __/  ヽ、           良い論文の出版を遅らせるお馬鹿な人。
     |       (:::::`‐-、__  |::::`、     ヒニニヽ、         あなたの論文が Ann of Math に accept される確率は?
    |      / `‐-、::::::::::`‐-、::::\   /,ニニ、\            それとも最近は Inv. Math. の方が上かしら?
   |      |::::::::::::::::::|` -、:::::::,ヘ ̄|'、  ヒニ二、 \
.   |      /::::::::::::::::::|::::::::\/:::O`、::\   | '、   \
   |      /:::::::::::::::::::/:::::::::::::::::::::::::::::'、::::\ノ  ヽ、  |
  |      |:::::/:::::::::/:::::::::::::::::::::::::::::::::::'、',::::'、  /:\__/‐、
  |      |/:::::::::::/::::::::::::::::::::::::::::::::::O::| '、::| く::::::::::::: ̄|
   |     /_..-'´ ̄`ー-、:::::::::::::::::::::::::::::::::::|/:/`‐'::\;;;;;;;_|
   |    |/::::::::::::::::::::::\:::::::::::::::::::::::::::::|::/::::|::::/:::::::::::/
    |   /:::::::::::::::::::::::::::::::::|:::::::::::::::::::::O::|::|::::::|:::::::::::::::/
246132人目の素数さん:2012/05/08(火) 22:47:52.49
Q:次の式はX,Yについては何次の多項式か?

@−3のxの2乗y

A:

@3


上のも問題がまったくわかりません。
なんでそうなるのですか?教えてください。
247132人目の素数さん:2012/05/08(火) 22:49:12.46
>>244 ??
248132人目の素数さん:2012/05/08(火) 22:52:31.44
>>247
キーボードの右上の方。ひらがなの「へ」のキー。
このキーを直接入力状態で打つか、半角英数に変換(F10キーが簡単)すると「^」が出る。
249132人目の素数さん:2012/05/08(火) 22:57:42.03
>>248 ありがとう、修正しました。

〜どなたか 
下記質問に答えていただけるかたいらっしゃいましたらよろしくお願いします。


Q:次の式はx,yについては何次の多項式か?

@-3x^2y

A:

@3


問題がまったくわかりません…
なぜそうなるのですか?教えてください。
250132人目の素数さん:2012/05/08(火) 23:10:41.18
          __ノ)-'´ ̄ ̄`ー- 、_
        , '´  _. -‐'''"二ニニ=-`ヽ、
      /   /:::::; -‐''"        `ーノ
     /   /:::::/           \
     /    /::::::/          | | |  |
     |   |:::::/ /     |  | | | |  |
      |   |::/ / / |  | ||  | | ,ハ .| ,ハ|
      |   |/ / / /| ,ハノ| /|ノレ,ニ|ル' 
     |   |  | / / レ',二、レ′ ,ィイ|゙/   私は只の数ヲタなんかとは付き合わないわ。
.     |   \ ∠イ  ,イイ|    ,`-' |      頭が良くて数学が出来てかっこいい人。それが必要条件よ。
     |     l^,人|  ` `-'     ゝ  |        さらに Ann.of Math に論文書けば十分条件にもなるわよ。
      |      ` -'\       ー'  人          一番嫌いなのは論文数を増やすためにくだらない論文を書いて
    |        /(l     __/  ヽ、           良い論文の出版を遅らせるお馬鹿な人。
     |       (:::::`‐-、__  |::::`、     ヒニニヽ、         あなたの論文が Ann of Math に accept される確率は?
    |      / `‐-、::::::::::`‐-、::::\   /,ニニ、\            それとも最近は Inv. Math. の方が上かしら?
   |      |::::::::::::::::::|` -、:::::::,ヘ ̄|'、  ヒニ二、 \
.   |      /::::::::::::::::::|::::::::\/:::O`、::\   | '、   \
   |      /:::::::::::::::::::/:::::::::::::::::::::::::::::'、::::\ノ  ヽ、  |
  |      |:::::/:::::::::/:::::::::::::::::::::::::::::::::::'、',::::'、  /:\__/‐、
  |      |/:::::::::::/::::::::::::::::::::::::::::::::::O::| '、::| く::::::::::::: ̄|
   |     /_..-'´ ̄`ー-、:::::::::::::::::::::::::::::::::::|/:/`‐'::\;;;;;;;_|
   |    |/::::::::::::::::::::::\:::::::::::::::::::::::::::::|::/::::|::::/:::::::::::/
    |   /:::::::::::::::::::::::::::::::::|:::::::::::::::::::::O::|::|::::::|:::::::::::::::/
251132人目の素数さん:2012/05/08(火) 23:10:57.04
>>249
xについて2次、yについて1次、合わせて3次。
そうやって数えることになってるから。
252132人目の素数さん:2012/05/08(火) 23:11:41.59
対角線論法の意味がわかりません。

対角線論法って

@ すべての実数を自然数で一対一対応できたと仮定する。
A @に従えば、実数と自然数は表にまとめれるはず。
B 実数について対角線上の数字を変えれば、その実数は表にのっておらず、@と矛盾する
C よって、実数は自然数と一対一対応しない。 

という論理であってますでしょうか?

仮に合ってるとすると、
ここから、どうやって、実数の濃度は自然数の濃度より高いという結論が導かれるのでしょうか。

また、上のAの部分が間違っているように思えるのですが。
実数も自然数も無限に観念できるのに、有限の表にまとめられると考えるのは矛盾では。
253132人目の素数さん:2012/05/08(火) 23:14:01.26
>>252
無限に広がった表だが
254132人目の素数さん:2012/05/08(火) 23:20:54.93
>>251

xの次数+yの次数=x,yの次数

という考え方でいいですか?
255132人目の素数さん:2012/05/08(火) 23:27:44.52
>>253
無限に広がる表なら、対角線の数字入れ替えたやつもどっかにのってるんじゃないですか?
256132人目の素数さん:2012/05/08(火) 23:30:55.42
>>254
いや、例えば、x+y^2とかだったら、2次だよ。
その問題の場合は、xの2次とyの1次が掛け合わされているから合わせて3次。
257132人目の素数さん:2012/05/08(火) 23:39:02.11
長さ2の定線分を直径にもつ円に内接し、その定線分にも内接するような円の中心が描く曲線と、定線分の囲む図形の面積を求めよ

解答で定円の中心から、定円と直径で囲まれる円の中心に直線を引くと、囲んでいる円との接点を通ることを使って関係式をたてています。
なぜ定円の中心から、定円と直径で囲まれる円の中心に直線を引くと、囲んでいる円との接点を通るのですか?
あと、このことは解答でいきなり利用していいのですか?
258132人目の素数さん:2012/05/08(火) 23:39:15.13
>>255
ところがドッコイ
どこにも載ってない
その無限に広がっる表てのは可算で作られたもの、
入れ替えたのは非可算はモン
だっちゅーことが判明した
259132人目の素数さん:2012/05/08(火) 23:42:57.35
>>257
それはなんでなんですかね?
後、対角線論法でわかるのは、実数は可算ではないということだけですか?
実数の濃度が自然数の濃度より高いというところまでは対角線論法だけでは証明
できないんでしょうか。
すみません頭悪い感じで。。
260132人目の素数さん:2012/05/08(火) 23:44:18.59
>>256

...わからないです。

x+y^2
はxの次数が1、yの次数が2 までは推測できます・・・
しかしそれ以降がわかりません。
掛け合わせるとはどういう意味ですか?
261132人目の素数さん:2012/05/08(火) 23:50:28.99
x*x
x*x*x
x*x*x*x*x*x
262132人目の素数さん:2012/05/08(火) 23:50:39.53
>>257
囲まれてる円に定線分とは別の接線を引けば、
扇に囲まれた円になる
そう見れば、直感的には正しい
実際に証明するとやはりただしい

263132人目の素数さん:2012/05/08(火) 23:53:11.25
>>261

ありがとう!!
264132人目の素数さん:2012/05/08(火) 23:54:20.00
入試問題などで見かけるけど無限級数の和って表現どうなんでしょうか?
級数が数列の和って意味だったと思うんですけど
265132人目の素数さん:2012/05/08(火) 23:57:05.01
誤爆です
266132人目の素数さん:2012/05/08(火) 23:58:03.53
>>261

x*y*y
 
次数が3つあるので答えは3ですね。
267すみません、次は下記がわかりません。:2012/05/08(火) 23:59:40.08
Q:次の式はx,yについては何次の多項式か?

@x^3-3ax^2+3bxy-cy^2

A:

@3

なぜ次数3になるのですか?
268132人目の素数さん:2012/05/09(水) 00:01:02.40
>>252
自然数nに対応する実数をx(n)とすると、Bで作るのはx(n)と小数点以下第n桁目が
違う実数だからどのnにも対応していない。

実数の集合⊃自然数の集合 だから 実数の濃度≧自然数の濃度 はまあ明らかで、
実数の濃度≠自然数の濃度 を証明したから 実数の濃度>自然数の濃度 ということ。
269132人目の素数さん:2012/05/09(水) 00:02:17.20
>>267
「多項式」と、その「項」って何のことだか分かる?
270132人目の素数さん:2012/05/09(水) 00:02:17.32
>>267
x, yについてなので
a, bは他の定数1とか2とか3とかと同じ扱いをす
2xは1次で
5x^2は2次
271132人目の素数さん:2012/05/09(水) 00:11:04.81
>>269

@x^3-3ax^2+3bxy-cy^2 だと

x^3、-3ax^2、3bxy、-cy^2 それぞれ4つのことですか?
272132人目の素数さん:2012/05/09(水) 00:11:08.70
>>264
何を無限級数の和と呼ぶかは教科書の中で定義されている筈だが。
273132人目の素数さん:2012/05/09(水) 00:18:18.80
>>271
その4つで一番高い次数を多項式の次数と呼ぶ
274132人目の素数さん:2012/05/09(水) 00:22:46.25
>>273

一番高い次数はx*x*x のx^3です。
x,y両方は項に含まれていませんが多項式全体にx,yが含まれている
のでOKということでいいでしょうか?
275132人目の素数さん:2012/05/09(水) 00:26:47.12
>>268
すみません対角線論法って例えば
1 = 0.1
2 = 0.11
3 = 0.111
4 = 0.1111


みたいな図を考えるわけですよね。それで
対角線上に1足したりして
0.2222
みたいな数を作り、この数は表にのってないやんみたいなこと言うわけですよね?
それで僕みたいなアホがそれはもうちょっと下の方にのってるんちゃうかと思うわけですよね。
申し訳ないですがもうちょっとわかりやすく教えてくれるとうれしいです。
276132人目の素数さん:2012/05/09(水) 00:31:01.57
>>274
答えはいいがその説明じゃちゃんと把握してなさそう
>>271の4つの項ですべてだ
除くなんてことはない
277132人目の素数さん:2012/05/09(水) 00:31:26.92
そもそも「載ってない数」の作り方が間違っとる
そこをイイカゲンに済ましてるから「もうちょっと下の方にのってるんちゃうか」なんて思うんだ
作り方を知ってさえいれば、実際に載ってないないのは明々白々
278132人目の素数さん:2012/05/09(水) 00:32:36.42
ナイス釣り
279132人目の素数さん:2012/05/09(水) 00:33:34.61
>>276

赤チャートでしごかれます。
また来ます。ありがとう〜。
280132人目の素数さん:2012/05/09(水) 00:34:22.35
無難に黒にしとけよ
問題数少ないぞ
281132人目の素数さん:2012/05/09(水) 01:38:05.54
>>275
> @ すべての実数を自然数で一対一対応できたと仮定する。
対角線論法はまずこの仮定から始まる

> 1 = 0.1
> 2 = 0.11
> 3 = 0.111
> 4 = 0.1111
> :
この仮定はその数列ですべての実数を述べられることを意味している
ここで対角線論法で作った実数Xがその数列内に存在し、その数が数列のm番目の数a(m)であるとすると、
Xのm桁目はa(m)のm桁目と異なるようにXは作られたのでX≠a(m)となり矛盾
282132人目の素数さん:2012/05/09(水) 02:31:17.56
          __ノ)-'´ ̄ ̄`ー- 、_
        , '´  _. -‐'''"二ニニ=-`ヽ、
      /   /:::::; -‐''"        `ーノ
     /   /:::::/           \
     /    /::::::/          | | |  |
     |   |:::::/ /     |  | | | |  |
      |   |::/ / / |  | ||  | | ,ハ .| ,ハ|
      |   |/ / / /| ,ハノ| /|ノレ,ニ|ル' 
     |   |  | / / レ',二、レ′ ,ィイ|゙/   私は只の数ヲタなんかとは付き合わないわ。
.     |   \ ∠イ  ,イイ|    ,`-' |      頭が良くて数学が出来てかっこいい人。それが必要条件よ。
     |     l^,人|  ` `-'     ゝ  |        さらに Ann.of Math に論文書けば十分条件にもなるわよ。
      |      ` -'\       ー'  人          一番嫌いなのは論文数を増やすためにくだらない論文を書いて
    |        /(l     __/  ヽ、           良い論文の出版を遅らせるお馬鹿な人。
     |       (:::::`‐-、__  |::::`、     ヒニニヽ、         あなたの論文が Ann of Math に accept される確率は?
    |      / `‐-、::::::::::`‐-、::::\   /,ニニ、\            それとも最近は Inv. Math. の方が上かしら?
   |      |::::::::::::::::::|` -、:::::::,ヘ ̄|'、  ヒニ二、 \
.   |      /::::::::::::::::::|::::::::\/:::O`、::\   | '、   \
   |      /:::::::::::::::::::/:::::::::::::::::::::::::::::'、::::\ノ  ヽ、  |
  |      |:::::/:::::::::/:::::::::::::::::::::::::::::::::::'、',::::'、  /:\__/‐、
  |      |/:::::::::::/::::::::::::::::::::::::::::::::::O::| '、::| く::::::::::::: ̄|
   |     /_..-'´ ̄`ー-、:::::::::::::::::::::::::::::::::::|/:/`‐'::\;;;;;;;_|
   |    |/::::::::::::::::::::::\:::::::::::::::::::::::::::::|::/::::|::::/:::::::::::/
    |   /:::::::::::::::::::::::::::::::::|:::::::::::::::::::::O::|::|::::::|:::::::::::::::/
283132人目の素数さん:2012/05/09(水) 05:01:23.68
>>257
円周の垂線は中心を通る
接してるんだからどっちの中心も通る
284132人目の素数さん:2012/05/09(水) 05:15:27.62
学校で受けた解説なんですが、

x>3→x≧3は成立
x≧3→x>3は不成立

「なんで上は成立するかっていうと、≧の記号の意味は、>または=っていう意味だから
どっちでもいいよね。だから>を選べばあきらかに成り立ってるよね。
では下は?成り立ってないよね」


285284:2012/05/09(水) 05:22:02.72
成立してるほうの解説では、≧の>だけを考えたんだなぁ。
じゃぁ不成立といわれたほうの≧も、「>または=」というふうに考え、同様に
>だけを考えれば成立するじゃんって思ったんです。
>または= なんだから って具合に。
しかしながら数直線に図示するとあきらかにマズいってことがわかります。
なんでx>3→x≧3のときは「=の可能性を考慮に入れずに」、x≧3→x>3のときは
「=の可能性を考慮に入れる」んですか?
286132人目の素数さん:2012/05/09(水) 05:33:04.98
>>285
A→B は言い換えると、Aが成り立つすべての場合においてBが成り立つという意味になる

x>3→x≧3 は、x=3の時はx>3が成り立たないから、x=3でx≧3が成り立っても立たなくてもどちらでもいい (実際は成り立つ)
x≧3→x>3 は、x=3の時はx≧3が成り立つから、x=3でx成り立たないといけない (実際は成り立たない)
287132人目の素数さん:2012/05/09(水) 05:41:49.10
xが3より大きい(x>3)ならばxは必ず3以上(x≧3)
xが3以上(x≧3)であってもxが3より大きい(x>3)とは限らない

悩む所は無いと思う
288132人目の素数さん:2012/05/09(水) 06:08:14.63
ありがとうございます。
>>286さんの、A→B は言い換えると、Aが成り立つすべての場合においてBが成り立つ
という解説と、>>287さんの解答を照らし合わせるとわかったような気がします。

A→Bのときは、Aの全ての可能性を考慮にいれるが、Bの全ては考慮に入れなくていい
ということでしょうか。
289132人目の素数さん:2012/05/09(水) 07:16:56.78
低レベルな問題なのだか、

120個のおはじきをA B Cの3人に
下の二つの条件で分けることにした。

AはBより19個多い
CはBより7個少ない

Aは何個になるのでしょうか?

頭悪すぎてすみません。誰か教えて
頂けないでしょうか。宜しくお願いします。
290132人目の素数さん:2012/05/09(水) 07:26:24.68
A=B+19
C=B−7
A+B+C=(B+19)+B+(B−7)=3B+12=120
B=108/3=36
A=B+19=55
C=B−7=29
291132人目の素数さん:2012/05/09(水) 08:02:22.31
          __ノ)-'´ ̄ ̄`ー- 、_
        , '´  _. -‐'''"二ニニ=-`ヽ、
      /   /:::::; -‐''"        `ーノ
     /   /:::::/           \
     /    /::::::/          | | |  |
     |   |:::::/ /     |  | | | |  |
      |   |::/ / / |  | ||  | | ,ハ .| ,ハ|
      |   |/ / / /| ,ハノ| /|ノレ,ニ|ル' 
     |   |  | / / レ',二、レ′ ,ィイ|゙/   私は只の数ヲタなんかとは付き合わないわ。
.     |   \ ∠イ  ,イイ|    ,`-' |      頭が良くて数学が出来てかっこいい人。それが必要条件よ。
     |     l^,人|  ` `-'     ゝ  |        さらに Ann.of Math に論文書けば十分条件にもなるわよ。
      |      ` -'\       ー'  人          一番嫌いなのは論文数を増やすためにくだらない論文を書いて
    |        /(l     __/  ヽ、           良い論文の出版を遅らせるお馬鹿な人。
     |       (:::::`‐-、__  |::::`、     ヒニニヽ、         あなたの論文が Ann of Math に accept される確率は?
    |      / `‐-、::::::::::`‐-、::::\   /,ニニ、\            それとも最近は Inv. Math. の方が上かしら?
   |      |::::::::::::::::::|` -、:::::::,ヘ ̄|'、  ヒニ二、 \
.   |      /::::::::::::::::::|::::::::\/:::O`、::\   | '、   \
   |      /:::::::::::::::::::/:::::::::::::::::::::::::::::'、::::\ノ  ヽ、  |
  |      |:::::/:::::::::/:::::::::::::::::::::::::::::::::::'、',::::'、  /:\__/‐、
  |      |/:::::::::::/::::::::::::::::::::::::::::::::::O::| '、::| く::::::::::::: ̄|
   |     /_..-'´ ̄`ー-、:::::::::::::::::::::::::::::::::::|/:/`‐'::\;;;;;;;_|
   |    |/::::::::::::::::::::::\:::::::::::::::::::::::::::::|::/::::|::::/:::::::::::/
292132人目の素数さん:2012/05/09(水) 13:19:03.96
3≦5は成り立ちません
293132人目の素数さん:2012/05/09(水) 13:39:38.34
>>288
そうだよ。
AならばBを考えているのであって、BならばAを考えているのではないのだから、
Bの中にAではない場合があっても構わない。
294132人目の素数さん:2012/05/09(水) 13:55:18.61
>>264
級数は数列の和じゃない。
数列を+をはさんで書いた物。
それを計算して和があるかどうかは分からない。
295132人目の素数さん:2012/05/09(水) 18:58:13.21
座標平面上に2直線L1:y=x、L2:y=mx(m≠1)がある
L1に関してA(a、b)と対称な点をB、L2 に関してBと対称な点をC、L1に関してCと対称な点をDとする
Aがどのような点であっても、常にAとDがL2に関して対称となるmを求めよ


という問題で、模範解答では

まずは点B、C、Dを求めて、「L2に関してAと対称な点がDと一致すればよい」
なお「その点はCの座標にてa、bを入れ代えればよい」

とありましたが、「」内の意味が理解できません
296132人目の素数さん:2012/05/09(水) 19:28:11.78
>>295
その解説はいったんほっといて、愚直に
点B、点C、点Dと順に座標を求めていったほうが
理解が早いかもしれんよ
297あのこうちやんは始皇帝だった:2012/05/09(水) 19:36:05.26

 お前たちは、定職に就くのが先決だろがあああああああ!!!!!!!!!!!!!!

 ニート・無職の、ゴミ・クズ・カスのクソガキどもがああああああ!!!!!!!!!!!!!!
298132人目の素数さん:2012/05/09(水) 19:36:09.66
A高校とB高校において、全生徒数の比は11:18、男子生徒数の比は2:3、女子生徒数の比は7:12である。
このとき、A高校での男子生徒数と女子生徒数の比はいくらか

この問題の解き方教えてください・・・
299132人目の素数さん:2012/05/09(水) 19:47:20.27
a:bとおいて連立方程式じゃないの。

300132人目の素数さん:2012/05/09(水) 19:50:15.71
a:b:18(a+b)/11とおけばいい。
301132人目の素数さん:2012/05/09(水) 19:52:55.32
11人 18人 

a人 b人とおけばよい

a+b=11

a:b:18
302132人目の素数さん:2012/05/09(水) 19:53:51.22
a3/2+b12/7=18人
a+b=11人

これとけばOK
303132人目の素数さん:2012/05/09(水) 19:54:40.43
a=4
b=7

だろ?

よって4:7が答えだ。
304132人目の素数さん:2012/05/09(水) 20:04:42.96
          __ノ)-'´ ̄ ̄`ー- 、_
        , '´  _. -‐'''"二ニニ=-`ヽ、
      /   /:::::; -‐''"        `ーノ
     /   /:::::/           \
     /    /::::::/          | | |  |
     |   |:::::/ /     |  | | | |  |
      |   |::/ / / |  | ||  | | ,ハ .| ,ハ|
      |   |/ / / /| ,ハノ| /|ノレ,ニ|ル' 
     |   |  | / / レ',二、レ′ ,ィイ|゙/   私は只の数ヲタなんかとは付き合わないわ。
.     |   \ ∠イ  ,イイ|    ,`-' |      頭が良くて数学が出来てかっこいい人。それが必要条件よ。
     |     l^,人|  ` `-'     ゝ  |        さらに Ann.of Math に論文書けば十分条件にもなるわよ。
      |      ` -'\       ー'  人          一番嫌いなのは論文数を増やすためにくだらない論文を書いて
    |        /(l     __/  ヽ、           良い論文の出版を遅らせるお馬鹿な人。
     |       (:::::`‐-、__  |::::`、     ヒニニヽ、         あなたの論文が Ann of Math に accept される確率は?
    |      / `‐-、::::::::::`‐-、::::\   /,ニニ、\            それとも最近は Inv. Math. の方が上かしら?
   |      |::::::::::::::::::|` -、:::::::,ヘ ̄|'、  ヒニ二、 \
.   |      /::::::::::::::::::|::::::::\/:::O`、::\   | '、   \
   |      /:::::::::::::::::::/:::::::::::::::::::::::::::::'、::::\ノ  ヽ、  |
  |      |:::::/:::::::::/:::::::::::::::::::::::::::::::::::'、',::::'、  /:\__/‐、
  |      |/:::::::::::/::::::::::::::::::::::::::::::::::O::| '、::| く::::::::::::: ̄|
   |     /_..-'´ ̄`ー-、:::::::::::::::::::::::::::::::::::|/:/`‐'::\;;;;;;;_|
   |    |/::::::::::::::::::::::\:::::::::::::::::::::::::::::|::/::::|::::/:::::::::::/
305132人目の素数さん:2012/05/09(水) 20:55:46.20
次の和を求める問題です。

1+1/(1+2)+1/(1+2+3)+・・・・・・1/(1+2+3+・・・・・・n)

どなたかお願いします。
306132人目の素数さん:2012/05/09(水) 21:07:27.53
2/(n(n+1)) = 2/n - 2/(n+1)
307132人目の素数さん:2012/05/09(水) 21:07:50.85
無限大飛ばすの楽しいです。
どうすればいいでしょうか?
308132人目の素数さん:2012/05/09(水) 21:11:16.00
半径aの円S[1]がある。
S[1]の四分円に内接する円S[2]をつくる。
さらにS[2]の四分円に内接する円S[3]をつくる。
このような操作を無限に続ける時、円S[1].S[2].S[3]・・・・・の面積の総和を求めよ

この問題は、
a[n]=(1+√2)a[n+1]という関係式から
a[n]=(1/1+√2)^(n-1)a
という式を求め
S[n]=π×a[n]^2
から無限級数を求める方法であってますか?

309132人目の素数さん:2012/05/09(水) 21:13:38.65
>>299-303
みなさんありがとうございます
>>302さんの式を解くと、>>303さんのいうとおりにa=4 b=7になって答えが4:7になるのもわかるのですが、
どういう考え方で>>302さんの式を立てるのでしょうか?
よろしくお願いします
310132人目の素数さん:2012/05/09(水) 21:16:43.72
>>308
あってるよ
311132人目の素数さん:2012/05/09(水) 21:21:24.62
          __ノ)-'´ ̄ ̄`ー- 、_
        , '´  _. -‐'''"二ニニ=-`ヽ、
      /   /:::::; -‐''"        `ーノ
     /   /:::::/           \
     /    /::::::/          | | |  |
     |   |:::::/ /     |  | | | |  |
      |   |::/ / / |  | ||  | | ,ハ .| ,ハ|
      |   |/ / / /| ,ハノ| /|ノレ,ニ|ル' 
     |   |  | / / レ',二、レ′ ,ィイ|゙/   私は只の数ヲタなんかとは付き合わないわ。
.     |   \ ∠イ  ,イイ|    ,`-' |      頭が良くて数学が出来てかっこいい人。それが必要条件よ。
     |     l^,人|  ` `-'     ゝ  |        さらに Ann.of Math に論文書けば十分条件にもなるわよ。
      |      ` -'\       ー'  人          一番嫌いなのは論文数を増やすためにくだらない論文を書いて
    |        /(l     __/  ヽ、           良い論文の出版を遅らせるお馬鹿な人。
     |       (:::::`‐-、__  |::::`、     ヒニニヽ、         あなたの論文が Ann of Math に accept される確率は?
    |      / `‐-、::::::::::`‐-、::::\   /,ニニ、\            それとも最近は Inv. Math. の方が上かしら?
   |      |::::::::::::::::::|` -、:::::::,ヘ ̄|'、  ヒニ二、 \
.   |      /::::::::::::::::::|::::::::\/:::O`、::\   | '、   \
   |      /:::::::::::::::::::/:::::::::::::::::::::::::::::'、::::\ノ  ヽ、  |
  |      |:::::/:::::::::/:::::::::::::::::::::::::::::::::::'、',::::'、  /:\__/‐、
  |      |/:::::::::::/::::::::::::::::::::::::::::::::::O::| '、::| く::::::::::::: ̄|
   |     /_..-'´ ̄`ー-、:::::::::::::::::::::::::::::::::::|/:/`‐'::\;;;;;;;_|
   |    |/::::::::::::::::::::::\:::::::::::::::::::::::::::::|::/::::|::::/:::::::::::/
    |   /:::::::::::::::::::::::::::::::::|:::::::::::::::::::::O::|::|::::::|:::::::::::::::/
312132人目の素数さん:2012/05/09(水) 21:56:45.16
0<α<π 、0<β<π とする。
sin2α−cos(π−β)=1
cos2α+sinβ=1
のとき、tan(α+5β)の値を求めよ。

どう変形すればよいのかわかりません。
お願いします。
313132人目の素数さん:2012/05/09(水) 22:05:47.99
cosπひくべーたは何になる?
314132人目の素数さん:2012/05/09(水) 22:10:24.92
−cosβです
315132人目の素数さん:2012/05/09(水) 22:17:04.34
>>308ですが
どうやっても答えが合わないんですが本当に合ってますかね?
答えが何回やっても(πa^2/2)(7+5√2)
になります。
解答欄には√の前に係数が付いてないのてどこかしら間違ってると思うのですが・・・
316132人目の素数さん:2012/05/09(水) 22:26:09.86
>>315
手間かもしれないけど一度その経過の式を書いていただけるとありがたい
317132人目の素数さん:2012/05/09(水) 22:33:02.46
>>312
(sinβ)^2 + (cosβ)^2 = 1
を使う。
それでしばらく考えて見て
解らなければまた聞きに来て。
318132人目の素数さん:2012/05/09(水) 22:37:12.06
ここに居る人たちは大学生とかですか?
319132人目の素数さん:2012/05/09(水) 22:39:37.77
>>316
すいません
解決しました。
nをn-1と勘違いしてたので無限級数の和を間違えてました。
協力ありがとうございました。
320132人目の素数さん:2012/05/09(水) 22:42:48.75
素数p, q で

 1+q が 1+p^2 を割り切る

を満たす組はないでしょうか。
321132人目の素数さん:2012/05/09(水) 22:45:11.74
>>320
明後日以降答えてあげるよ^^
322132人目の素数さん:2012/05/09(水) 22:46:24.91
何かの問題なのか?
323132人目の素数さん:2012/05/09(水) 22:47:19.45
なんだ大数か
324132人目の素数さん:2012/05/09(水) 22:54:40.60
|a|<2,|b|<2のとき|ab|>2|a+b|-4を証明せよ。
の証明がうまくいきません。左辺の二乗から右辺の二乗を引いてもうまく続かず・・・
どうすればよいでしょうか
325132人目の素数さん:2012/05/09(水) 22:59:43.57
(lal-2)(lbl-2)>0
326132人目の素数さん:2012/05/09(水) 23:00:22.00
三角不等式だろ
327132人目の素数さん:2012/05/09(水) 23:09:55.61
>>326 >>325ありがとうございます。
ついでですが
|x|<1,|y|<1のとき|x+y|+|x-y|<2も三角不等式を使うのでしょうか。
328132人目の素数さん:2012/05/09(水) 23:15:59.38
>>327

左辺=|x+y|+|x-y|<|x|+|y|+|x|+|-y|
=2(|x|+|y|)<2(1+1)=4
となって右辺の2がうまく示せないのですがどうすればよいでしょうか
329132人目の素数さん:2012/05/09(水) 23:25:54.78
4人でじゃんけんをする。このとき、ちょうど2人が勝つ確率を求めよ

お願いします
330132人目の素数さん:2012/05/09(水) 23:26:39.40
>>327
(|x+y|+|x-yl)^2
=2(x^2+y^2) + 2lx^2-y^2l
=Max{4x^2 , 4y^2} < 4
331132人目の素数さん:2012/05/09(水) 23:30:56.25
>>330 ありがとうございます。
高校の数学の問題なんですが、このような解法しかないのでしょうか。
三角不等式を用いて>>328を改善して証明する方法等は
332132人目の素数さん:2012/05/09(水) 23:35:00.73
>>317
−1ですか?
333132人目の素数さん:2012/05/09(水) 23:37:34.90
場合分けのときにガウス記号のかっこを使ったら印象悪いですか?
[1] x>aのとき みたいな感じで
334132人目の素数さん:2012/05/09(水) 23:38:02.31
無問題
335132人目の素数さん:2012/05/09(水) 23:41:31.94
4人でじゃんけんをすれば必ず2人以上は同じのを出す
A,Bがグーを出したとすると
残りのC,Dの手の組み合わせは3^2=9
内条件文に合うのは
C, D=パー or チョキの2通り

よって2/9

の筈
336132人目の素数さん:2012/05/09(水) 23:42:40.93
>>332
過程はわからんけど
tan(α+5β)=−1
と言う意味ならあってる。
337333:2012/05/09(水) 23:44:10.77
>>334
ありがとうございました
338132人目の素数さん:2012/05/09(水) 23:44:50.23
>>331ですが、>>330の変形は思いつかない限り証明できないのでしょうか。
339132人目の素数さん:2012/05/09(水) 23:45:45.77
>>335
なるほど!
わかりやすくて助かりました!
ありがとうございます!
340132人目の素数さん:2012/05/09(水) 23:46:32.58
>>327
|x+y|+|x-y|≦2Max{|x+y|, |x-y|}=2Max{2Max{|x|, |y|}, Max{x-y, y-x}}≦2Max{2, 2}=4
341132人目の素数さん:2012/05/09(水) 23:48:56.31
>>340 返信ありがとうございます。
この問題は高校の問題なんですが、max記号を用いて>>340>>330のように
論述するしかないのでしょうか。高校の問題ですが答案にも使える?
342132人目の素数さん:2012/05/09(水) 23:49:22.14
>>336
ありがとうございます
過程もあってると思います。
343132人目の素数さん:2012/05/09(水) 23:50:54.67
>>341
xとy入れ替えても問題の不等式は変わらないので
x≦yとしてしまってえば問題ない
344132人目の素数さん:2012/05/09(水) 23:52:25.83
横やり申し訳ないんですけど、、
>>335のやつって、分母は4人じゃんけんしたときの結果の総数とかになるんじゃないんですか?
3人おなじ手だった場合とかのことってこの計算の中に入ってます?
気になったもので・・・
申し訳ありませんが、後学のためにお教えください
345132人目の素数さん:2012/05/10(木) 00:00:55.56
          __ノ)-'´ ̄ ̄`ー- 、_
        , '´  _. -‐'''"二ニニ=-`ヽ、
      /   /:::::; -‐''"        `ーノ
     /   /:::::/           \
     /    /::::::/          | | |  |
     |   |:::::/ /     |  | | | |  |
      |   |::/ / / |  | ||  | | ,ハ .| ,ハ|
      |   |/ / / /| ,ハノ| /|ノレ,ニ|ル' 
     |   |  | / / レ',二、レ′ ,ィイ|゙/   私は只の数ヲタなんかとは付き合わないわ。
.     |   \ ∠イ  ,イイ|    ,`-' |      頭が良くて数学が出来てかっこいい人。それが必要条件よ。
     |     l^,人|  ` `-'     ゝ  |        さらに Ann.of Math に論文書けば十分条件にもなるわよ。
      |      ` -'\       ー'  人          一番嫌いなのは論文数を増やすためにくだらない論文を書いて
    |        /(l     __/  ヽ、           良い論文の出版を遅らせるお馬鹿な人。
     |       (:::::`‐-、__  |::::`、     ヒニニヽ、         あなたの論文が Ann of Math に accept される確率は?
    |      / `‐-、::::::::::`‐-、::::\   /,ニニ、\            それとも最近は Inv. Math. の方が上かしら?
   |      |::::::::::::::::::|` -、:::::::,ヘ ̄|'、  ヒニ二、 \
.   |      /::::::::::::::::::|::::::::\/:::O`、::\   | '、   \
   |      /:::::::::::::::::::/:::::::::::::::::::::::::::::'、::::\ノ  ヽ、  |
  |      |:::::/:::::::::/:::::::::::::::::::::::::::::::::::'、',::::'、  /:\__/‐、
  |      |/:::::::::::/::::::::::::::::::::::::::::::::::O::| '、::| く::::::::::::: ̄|
   |     /_..-'´ ̄`ー-、:::::::::::::::::::::::::::::::::::|/:/`‐'::\;;;;;;;_|
   |    |/::::::::::::::::::::::\:::::::::::::::::::::::::::::|::/::::|::::/:::::::::::/
    |   /:::::::::::::::::::::::::::::::::|:::::::::::::::::::::O::|::|::::::|:::::::::::::::/
346132人目の素数さん:2012/05/10(木) 00:04:13.92
そうなんだけど3^4=81とおりになるので面倒
一般性を失わない形で一部を取り出し、後は同様、と逃げるのが普通
今回は1/9を抜き出し、以下同様とした

81通りなんでexcelがあれば表を作ってみるといい
A=B=チョキの場合、もしくはA=C=グーの場合とかを抜き出しても
まったく同じ表になるはず
ちょっと不安なので実際に検算してもらえると非常に助かる

自信がなかったら実際に何パターンか書いてみると法則性が見えてくる
347132人目の素数さん:2012/05/10(木) 00:07:20.08
          __ノ)-'´ ̄ ̄`ー- 、_
        , '´  _. -‐'''"二ニニ=-`ヽ、
      /   /:::::; -‐''"        `ーノ
     /   /:::::/           \
     /    /::::::/          | | |  |
     |   |:::::/ /     |  | | | |  |
      |   |::/ / / |  | ||  | | ,ハ .| ,ハ|
      |   |/ / / /| ,ハノ| /|ノレ,ニ|ル' 
     |   |  | / / レ',二、レ′ ,ィイ|゙/   私は只の数ヲタなんかとは付き合わないわ。
.     |   \ ∠イ  ,イイ|    ,`-' |      頭が良くて数学が出来てかっこいい人。それが必要条件よ。
     |     l^,人|  ` `-'     ゝ  |        さらに Ann.of Math に論文書けば十分条件にもなるわよ。
      |      ` -'\       ー'  人          一番嫌いなのは論文数を増やすためにくだらない論文を書いて
    |        /(l     __/  ヽ、           良い論文の出版を遅らせるお馬鹿な人。
     |       (:::::`‐-、__  |::::`、     ヒニニヽ、         あなたの論文が Ann of Math に accept される確率は?
    |      / `‐-、::::::::::`‐-、::::\   /,ニニ、\            それとも最近は Inv. Math. の方が上かしら?
   |      |::::::::::::::::::|` -、:::::::,ヘ ̄|'、  ヒニ二、 \
.   |      /::::::::::::::::::|::::::::\/:::O`、::\   | '、   \
   |      /:::::::::::::::::::/:::::::::::::::::::::::::::::'、::::\ノ  ヽ、  |
  |      |:::::/:::::::::/:::::::::::::::::::::::::::::::::::'、',::::'、  /:\__/‐、
  |      |/:::::::::::/::::::::::::::::::::::::::::::::::O::| '、::| く::::::::::::: ̄|
   |     /_..-'´ ̄`ー-、:::::::::::::::::::::::::::::::::::|/:/`‐'::\;;;;;;;_|
   |    |/::::::::::::::::::::::\:::::::::::::::::::::::::::::|::/::::|::::/:::::::::::/
    |   /:::::::::::::::::::::::::::::::::|:::::::::::::::::::::O::|::|::::::|:::::::::::::::/
348132人目の素数さん:2012/05/10(木) 00:08:52.24
銀のエンゼルの当たる確率が1/2だとすると、五枚集めるときの確率は1/2^5だと思うのですが
銀のエンゼルa,b,c,d,eを集める時の確率は1/(2・5)^5で良いのでしょうか
349132人目の素数さん:2012/05/10(木) 00:11:00.35
>>348
クーポンコレクターの亜種じゃねーの?
350132人目の素数さん:2012/05/10(木) 00:11:20.29
>>348
最初からちがうwwwwwwwwwwwww
351132人目の素数さん:2012/05/10(木) 00:29:36.51
>>329
3人が勝つ12
2人が勝つ18
1人が勝つ12
あいこ39
12+18+12+39=81
352132人目の素数さん:2012/05/10(木) 00:38:32.97
>>351
サンクス
安心したこれでグッスリ眠れる
353132人目の素数さん:2012/05/10(木) 00:41:39.92
あいこの時も考えると

(18/81)+(39/81)*{(18/81)+(39/81)*{(18/81)+......
=(2/9)+(13/27)*(2/9)+(13/27)^2*(2/9)+......
=lim[n→∞]Σ[k=0,n](2/9)(13/27)^k

になるんじゃないか?
354132人目の素数さん:2012/05/10(木) 00:47:09.54
>>353
こいつ最高にアホ
355132人目の素数さん:2012/05/10(木) 00:47:12.51
>>346 >>351
実際は分母81になるけど、結局その分二人が勝つ場合も増えるから答えは変わらず
という考えでよろしいのでしょうか?
今回の場合だと答えば2/9ですので、実際の分母が81ということは
18/81が正しく全通り計算したときの結果で、約分により2/9になる
といった形なのでしょうか?
356132人目の素数さん:2012/05/10(木) 00:51:29.96
形なのでしょうかっていうか
完璧にこの考え方ですねw
なんどもしつこく質問する形になってしまって申し訳ありません。
みなさんありがとうございました!!
357132人目の素数さん:2012/05/10(木) 00:52:13.44
>>356
樹形図かけ
そうすれば見えてくる
358132人目の素数さん:2012/05/10(木) 00:53:40.86
>>357
ありがとうございます。
樹形図書いている途中でもう一度最初から全部読み直して気づきました
丁寧に全部の通り数考えて計算しなくてもこういう計算の仕方もできるのですね!
勉強になりました!
359132人目の素数さん:2012/05/10(木) 00:57:09.11
Aがグーの場合だけ書けば27通りですむ。
後は"×3"だ
360132人目の素数さん:2012/05/10(木) 01:07:11.28
蛇足だけど

A,Bがグーを出したとすると
>抜き出すよーの合図
4人でじゃんけんをすれば必ず2人以上は同じのを出す
>一般性ちゃんと考えてるよーの言い訳

C,Dの組み合わせ…
>以下総ざらい

という思考   
361132人目の素数さん:2012/05/10(木) 01:09:37.61
>>354
アホなのはわかってるから理由も書いてくれよ
362132人目の素数さん:2012/05/10(木) 06:48:03.10
>>296
自己解答では二直線の直行条件などをもちいてB〜Dを順に求めていきました
で、そこで手詰まりになってしまいました
そこからはどう考えればよいのでしょうか?
363132人目の素数さん:2012/05/10(木) 07:38:44.31
          __ノ)-'´ ̄ ̄`ー- 、_
        , '´  _. -‐'''"二ニニ=-`ヽ、
      /   /:::::; -‐''"        `ーノ
     /   /:::::/           \
     /    /::::::/          | | |  |
     |   |:::::/ /     |  | | | |  |
      |   |::/ / / |  | ||  | | ,ハ .| ,ハ|
      |   |/ / / /| ,ハノ| /|ノレ,ニ|ル' 
     |   |  | / / レ',二、レ′ ,ィイ|゙/   私は只の数ヲタなんかとは付き合わないわ。
.     |   \ ∠イ  ,イイ|    ,`-' |      頭が良くて数学が出来てかっこいい人。それが必要条件よ。
     |     l^,人|  ` `-'     ゝ  |        さらに Ann.of Math に論文書けば十分条件にもなるわよ。
      |      ` -'\       ー'  人          一番嫌いなのは論文数を増やすためにくだらない論文を書いて
    |        /(l     __/  ヽ、           良い論文の出版を遅らせるお馬鹿な人。
     |       (:::::`‐-、__  |::::`、     ヒニニヽ、         あなたの論文が Ann of Math に accept される確率は?
    |      / `‐-、::::::::::`‐-、::::\   /,ニニ、\            それとも最近は Inv. Math. の方が上かしら?
   |      |::::::::::::::::::|` -、:::::::,ヘ ̄|'、  ヒニ二、 \
.   |      /::::::::::::::::::|::::::::\/:::O`、::\   | '、   \
   |      /:::::::::::::::::::/:::::::::::::::::::::::::::::'、::::\ノ  ヽ、  |
  |      |:::::/:::::::::/:::::::::::::::::::::::::::::::::::'、',::::'、  /:\__/‐、
  |      |/:::::::::::/::::::::::::::::::::::::::::::::::O::| '、::| く::::::::::::: ̄|
   |     /_..-'´ ̄`ー-、:::::::::::::::::::::::::::::::::::|/:/`‐'::\;;;;;;;_|
   |    |/::::::::::::::::::::::\:::::::::::::::::::::::::::::|::/::::|::::/:::::::::::/
    |   /:::::::::::::::::::::::::::::::::|:::::::::::::::::::::O::|::|::::::|:::::::::::::::/
364132人目の素数さん:2012/05/10(木) 08:26:14.08
>>362
任意の点(p,q)の直線y=kxに関して対象な点は
(2(p+kq)/(1+k^2)-a, 2k(p+kq)/(1+k^2)-q)
ここだけできてれば、あとはA→B→C→D→Aって変換していって、mの方程式といて終了じゃないの?
365132人目の素数さん:2012/05/10(木) 08:41:28.34
>>335
どこか思い違いしているかも知れないが、「5人でやってちょうど2人が勝つ確率」をその考え方でやると、
5人でじゃんけんをすれば必ず2人以上は同じのを出す
A,Bがグーを出したとすると
残りのC,D,Eの手の組み合わせは3^3=27
内条件文に合うのは
C, D,E=チョキの1通り
よって1/27
となってしまわないかな?
366132人目の素数さん:2012/05/10(木) 09:29:18.61
>>335
・勝者が最大人数グループであること
・もう一つ最大人数グループが存在するときに
扱っているA、Bが敗北側であっても
勝者がもう一つの最大人数グループなので構わないこと
という巧妙な仕掛けが含まれている気がする
367132人目の素数さん:2012/05/10(木) 10:05:33.15

x+y=0, 2x-y=3

の連立方程式ですが、
この名称は、

・2元1次連立方程式
・2元連立1次方程式
・連立2元1次方程式
・その他

正式にはどれですか?
368132人目の素数さん:2012/05/10(木) 10:12:28.22
>>367
3番目じゃないかなあ?
369132人目の素数さん:2012/05/10(木) 10:14:19.20
>>367
http://www.shinko-keirin.co.jp/keirinkan/topics/2011/data/math_keitouhyo.pdf
学校教育では3番目が採用されているらしい。
370132人目の素数さん:2012/05/10(木) 13:08:47.60
普通、2元連立1次方程式だろ
371132人目の素数さん:2012/05/10(木) 13:14:00.76
俺もそう思っていたが、啓林館に言われてはなあ。
372132人目の素数さん:2012/05/10(木) 14:27:00.73
将来、数学をも駆使して問題点を暴かなければならない。

以下問題。

真如苑では、信者の様々な悩みは霊位向上によって良くなるという。
霊位向上には、最も重要なのは"おたすけ"と呼ばれる行為で霊位向上には必須事項。

その"おたすけ"とは、一般人を真如苑に加入させ真如苑の活動に熱心に取組むよう強く動機付ける、
その結果、新信者が新たに一般人を真如苑に加入させ、真如苑の活動に熱心に取組むよう強く動機付ける様指導・・・。
と自らではなく、新規に真如苑に縁を持った者達をどれだけ熱心に真如苑の活動に取組ませるか、
判断は、加入させた信者の熱心度合いで決まる。
もちろん霊位向上にはおたすけは絶対避けて通れない。

あくまでも勧誘ではありません。真如苑に縁を持たせ積極的に取組ませるので勧誘ではありません。


数学で何が問題で、どう証明するか。漠然とした問題に明確に答えられる事が必要になる。
373132人目の素数さん:2012/05/10(木) 14:40:24.64
x+y=0 ← 2元1次方程式

x+y=0, 2x-y=3 ← 連立する2元1次方程式=連立2元1次方程式

答えを聞いた後で考えてみると、3番が自然だな
374132人目の素数さん:2012/05/10(木) 14:52:04.03
△ABCの三辺の長さがAB=√5,BC=1,AC=√10である。〜
みたいな、△ABCの問題をよく見かけます。三角形を紙に書くとき、みなさんはまず辺の比をみてから、できる限り形を似せた三角形を書いてますか?
たとえば上の三角形なら鈍角の三角形になりますが、鈍角、鋭角の三角形ぐらいの違いは与えられた辺をみて読み取るべきですよね?今まで何も考えてなかったのでそういうところで、できるできないの差がつくのかなあと思いました。
375132人目の素数さん:2012/05/10(木) 15:01:29.66
>>374
出来る限り近づけるよ。
376132人目の素数さん:2012/05/10(木) 15:19:46.90
フリーハンドで適当に書いて、適当に修正(例で言えば 1:2:3に近い形に修正)
少し真面目に書く場合は、1.0:2.2:3.1程度に書く
377132人目の素数さん:2012/05/10(木) 15:30:30.59
定規とコンパスがあれば1と√5と√10は用意できる
378132人目の素数さん:2012/05/10(木) 15:36:33.80
2元連立1次方程式の「2」は変数の個数?方程式の個数?
379132人目の素数さん:2012/05/10(木) 15:39:05.46
380132人目の素数さん:2012/05/10(木) 15:52:45.66
定規とコンパスって大学受験でも持ち込めたっけ?
381132人目の素数さん:2012/05/10(木) 15:56:49.28
定規は直線をひくため
コンパスは直角を作るためと同じ長さをとるため
目測で頑張ってください
382132人目の素数さん:2012/05/10(木) 16:01:52.37
目測なら最初から√5と√10を目測で作るわw
383132人目の素数さん:2012/05/10(木) 16:09:44.92
>>382
それは自分で長さを決めているだけだろ
1に対して2はある程度正確に作れる
384132人目の素数さん:2012/05/10(木) 16:11:18.01
行き詰まったら綺麗に書き直すけど初見で解く時に図形をかいても
下手なのもあって7より5の方が長いなんて事はザラにある。
最初はどんな法則を使うのか考えやすくする為に書いているのであって形にはあまり興味がない。
ただ角度の鈍角か鋭角かぐらいは区別してる
385132人目の素数さん:2012/05/10(木) 16:12:20.64
一応だが、1,√5,√10の三角形なら下のようにABCをとれば
AB=1,BC=√5,AC=√10になる

A B
┌┬┬┐
└┴┴┘
     C
386132人目の素数さん:2012/05/10(木) 16:15:08.44
>>383
だから、それくらいなら目測でも出来るだろ。おまえ出来ないの?
387132人目の素数さん:2012/05/10(木) 16:15:39.52
>>382
こいつ最高にアホ
388132人目の素数さん:2012/05/10(木) 16:16:09.99

ようやく正解きた
95点
389132人目の素数さん:2012/05/10(木) 16:20:55.39
>>388
なんだ?もしかしてこれ>>385を書けば正解だったのか?
それならそうと最初から言えよ
で、何がしたかったの?
390132人目の素数さん:2012/05/10(木) 16:24:34.61
√5や√10よりも2のほうが正確に作れるよ

それくらいなら目測でも出来る

??????????????
391132人目の素数さん:2012/05/10(木) 16:25:17.18
簡略したため不備が生まれたかもしれないが、とある有名な問題

・任意の2点を通る直線を引く
・任意の1点を中心とし他の任意の1点を通る円を描く
・異なる2直線、または直線と円
 または異なる2円が交わった場合、交わった場所を新たな点とみなす

いま(曲がっていない普通の)平面上にて3点A,B,Cが与えられている
上記3操作のみでAを中心とした半径BCの円を描け
いわゆるコンパスが長さの移動のために直接使えないことに注意すること
392132人目の素数さん:2012/05/10(木) 16:26:11.87
>>390
ちょっと何言ってんのかわからない
393132人目の素数さん:2012/05/10(木) 16:27:10.58
折り紙すればもっと正確に作れるよ
394132人目の素数さん:2012/05/10(木) 16:31:55.44
>>391
出題スレじゃないよ
395132人目の素数さん:2012/05/10(木) 16:37:00.30
>>391
Cを通ってABに平行な直線を引く(ひし形を利用すれば出来る)。
Aを通ってBCに平行な直線を引く。
これで、Aから距離BCの点が作れるので円を描く。
396132人目の素数さん:2012/05/10(木) 16:45:52.94
>>395
なるほど、その方法もあるな

>>394
定規とコンパスの作図の話が出てたからその関連としてね
原論では長さの移動は円の作図を介するものとして扱われてて驚いた
397132人目の素数さん:2012/05/10(木) 16:47:39.44
√aは簡単に作れないか?
√aと1 を2辺とする直角三角形の斜辺で√(a+1)
398132人目の素数さん:2012/05/10(木) 16:50:33.69
>>397
池沼
399132人目の素数さん:2012/05/10(木) 17:05:54.00
質問します。

a^3+b^3+c^3-3abc=(a+b+c)(a^2+b^2+c^2-ab-bc-ca)
という式の途中式で、
(a+b+c){(a+b)^2-(a+b)c+c^2}-3ab(a+b+c)
この後共通因数をくくり出して
(a+b+c){(a+b)^2-(a+b)c+c^2-3ab}
とするべきところを、間違えて
(a+b+c)(1-3ab){(a+b)^2-(a+b)c+c^2}
としてしまい、答えが合いませんでした。
しかし、なぜこうしてはいけないのか、しっくりきません。
どうかご教示ください。
400132人目の素数さん:2012/05/10(木) 17:14:07.62
>>399
> (a+b+c)(1-3ab){(a+b)^2-(a+b)c+c^2}
> としてしまい
なぜこうしていいと思ったのか説明してくれ
401132人目の素数さん:2012/05/10(木) 17:21:18.42
>>400
{(a+b)^2-(a+b)c+c^2}
を一旦除いて
(a+b+c)-3ab(a+b+c)
=(a+b+c)(1-3ab)
と考えたためです。
402132人目の素数さん:2012/05/10(木) 17:23:09.86
>>375
>>376
>>377
鈍角鋭角直角の三角形を間違えて書いたらかなりややこしくなりますよね。
勉強になりました。ありがとうございます
403132人目の素数さん:2012/05/10(木) 17:28:22.82
>>401
1じゃないものを勝手に1に置き換えたら正しい計算になるわけがない。
長くて書くのが面倒というなら {(a+b)^2-(a+b)c+c^2} を A とでも置いて計算する(後で戻す)。
404132人目の素数さん:2012/05/10(木) 17:33:45.40
>>399
a+b+c=A と置き換えれば
(a+b+c){(a+b)^2-(a+b)c+c^2}-3ab(a+b+c)は
A{(a+b)^2-(a+b)c+c^2}-3abAになるので、
あとは共通因数Aでくくって
A{(a+b)^2-(a+b)c+c^2-3ab}になる。
この後Aをa+b+cに戻す。

逆に(a+b+c){(a+b)^2-(a+b)c+c^2-3ab}で
a+b+c=Aと置き換えて展開すると
A{(a+b)^2-(a+b)c+c^2}-3abAになるので
この後Aをa+b+cに戻すと最初の式

>>401
>一旦除いて
何故除くのか理解不能。
405132人目の素数さん:2012/05/10(木) 17:34:57.80
>>403
(a+b+c)-3ab(a+b+c)
=(a+b+c)(1-3ab)
はそもそも間違っているということでしょうか?
(a+b+c)-3ab(a+b+c)
=1・(a+b+c)-3ab(a+b+c)
=(a+b+c)(1-3ab)
と考えました。
406132人目の素数さん:2012/05/10(木) 17:40:57.98
>>404
申し訳ありません、説明が不足していました。
正しい解き方は納得して理解しているのですが、
同時に自分の間違え方はどこがいけなかったのかがいまいちわからないという状況です。

{(a+b)^2-(a+b)c+c^2}を一旦除いたのは、
掛け算なので最後に掛けてもいいのではないかと思ったためです。
407132人目の素数さん:2012/05/10(木) 17:42:48.29
>>405
(a+b+c)=A とおく
{(a+b)^2-(a+b)c+c^2}=B とおく

与式= A(B) - 3ab(A) になるはずだのに「わざわざ Bを一旦除いて」しまったがため
A - 3ab(A) と勘違いをしている
408132人目の素数さん:2012/05/10(木) 17:51:13.30
>>401
> >>400
> {(a+b)^2-(a+b)c+c^2}
> を一旦除いて
なぜ、この操作をしてもよいと思ったのかがわからん。

ab-abの1項目のbを一旦除くとa(1-b)、bを戻してa(1-b)bを正しいと思うの?
409132人目の素数さん:2012/05/10(木) 17:53:29.18
>>406
元の(a+b+c){(a+b)^2-(a+b)c+c^2}-3ab(a+b+c)の後ろの項に{(a+b)^2-(a+b)c+c^2}はないのだが?
410132人目の素数さん:2012/05/10(木) 17:53:51.87
>>406
AX+B を「AXは掛け算なのでXを一旦取り除いて」
A+B として「Xを最後に掛けて」結局
AX+B=(A+B)X とするのが正しいと?
411132人目の素数さん:2012/05/10(木) 17:58:19.23
結局、ちゃんと算数から積み重ねないとだめですよってことだな。
順番にはいろいろ議論はあるかも知れんが、こういう順番に学習するとよいですよって言ってくれてるのに。
412132人目の素数さん:2012/05/10(木) 18:13:23.33
皆様に丁寧に解説をしていただいて、おかげ様で自分の勘違いしていた点がわかりました。
ありがとうございます。
まさに>>410の方が仰られた通りの勘違いをしていました。
式の若干の複雑さが要因であったと感じますが、
そもそもこの様な勘違いをしてしまうのは>>411の方も仰るように、基礎となる算数の分野に
怪しいまま放置してきた箇所があるのかもしれません。
一度最も基礎的な部分を洗ってみます。
回答してくださった皆様には重ねてお礼申し上げます。
では、失礼します。
413132人目の素数さん:2012/05/10(木) 19:00:52.19
追いうちかけて気分を害したら申しわけないけど

>>406
>正しい解き方は納得して理解しているのですが、
解き方を丸暗記してるだけでないかと。

>>412
>基礎となる算数の分野に
そこまで遡らなくてイイと思うけど、乗法(加法)の交換法則とか、結合法則(←大事)とか、
加法より乗法(除法)を先に計算する(→式としては乗法の結びつきが強い)とか覚えておくとイイかも。
414132人目の素数さん:2012/05/10(木) 19:55:01.35
因数定理の問題でf(x)=0となるxを1から当てはめてくのではなく、
簡単に求める方法はありますか?
415132人目の素数さん:2012/05/10(木) 19:58:53.95
>>414
ない
でも高校の問題なら大抵は暗算でパッと出るように作ってある
416132人目の素数さん:2012/05/10(木) 20:01:55.99
>>414
整数範囲でという条件があるなら、定数項の因数とか?
417132人目の素数さん:2012/05/10(木) 21:33:39.42
>>414
ないと言えばないけど、あてとしては↓

定数項の正・負の約数 または (定数項の正・負の約数)/(最高次の係数の約数)

理解しやすいシグマベスト数U・BのP56に記載されてるし色は忘れたけどチャート式にもあった。
418132人目の素数さん:2012/05/10(木) 22:01:41.97
S=2^(n-1)+2*2^(n-2)+3*2^(n-3)+・・・・・・+(n-1)*2+n

和Sを求めでください。お願いします。
419132人目の素数さん:2012/05/10(木) 22:10:49.27
>>418
等差×等比の和だからS-2S
420132人目の素数さん:2012/05/10(木) 22:21:34.34
因数定理は高次方程式の解になるのでスンナリ理解できましたけど剰余の定理はスッキリ解りません。
実際に割った時の余りと一致しますけど、皆さんはどうやって剰余の定理の証明を理解しましたか?
421132人目の素数さん:2012/05/10(木) 22:25:40.00
>>418
k*2^(-k)=(k+1)*2^(-(k-1))-(k+2)*2^(-k)

S=2^n*Σ[k=1,n]k*2^(-k)
=2^n*(1*2^(-1)-(n+2)*2^(-n))
422132人目の素数さん:2012/05/10(木) 22:26:49.72
>>420
g(x)=f(x)-f(α)
423132人目の素数さん:2012/05/10(木) 22:30:28.40
P(x)をx-aで割った商をQ(x)余りをRとした時
P(x)=(x-a)*Q(x)+R

x=aとした時
R=0なら因数定理
R≠0なら剰余定理

くらいの感覚
424132人目の素数さん:2012/05/10(木) 22:36:57.87
>>420
どこがわからんのか具体的に。
425132人目の素数さん:2012/05/10(木) 22:48:00.16
皆さん、頭の良い方なのでしょうね。

>>422
失礼ながら循環論法的な説明になっています。

因数定理が解るのであればg(x)=f(x)-f(α)=0 と言う問題に帰結するので剰余の定理も解るのでは?
と言ってるようです。
f(α)が何故、実際に割ってもいないのに余りが解るのか そこがスッキリ解らないどかしさが残ります。

>>423
証明だけは何十回も見ました。

>>424
実際に割ってもいないのに余りが解ることです。証明は高校生に難易度が高いのでは。
426132人目の素数さん:2012/05/10(木) 22:49:18.53
x)そこがスッキリ解らないどかしさが残ります。
○)そこがスッキリ解らないもどかしさが残ります。
427132人目の素数さん:2012/05/10(木) 22:53:28.83
>>425
P(x)=(x-a)*Q(x)+Rがわからんということ?
428132人目の素数さん:2012/05/10(木) 22:55:04.95
>>425
お前が池沼だから
429132人目の素数さん:2012/05/10(木) 22:56:28.75
>>425
g(x)=f(x)-f(α)
g(α)=0
(x-α)を因数に持つ
430132人目の素数さん:2012/05/10(木) 22:57:06.73
循環論法?
431132人目の素数さん:2012/05/10(木) 22:57:25.98
>>425
xの多項式P(x)を(x-a)で割ったときの商と余りの定義を書いてみて
432132人目の素数さん:2012/05/10(木) 22:58:43.60
>>425
> 実際に割ってもいないのに余りが解ることです。証明は高校生に難易度が高いのでは。
P(x)=(x-a)*Q(x)+R は式変形しているだけで、割っているわけではない
433132人目の素数さん:2012/05/10(木) 22:59:31.47
高二病か
434132人目の素数さん:2012/05/10(木) 23:06:08.92
割るっていう操作の認識の宗教上の違いみたいなもんだろ(笑)
そういう小理屈の数学のジャンルが流行った時期があるんだぞ。ゲーデルに粉々に打ち砕かれたが。
1+1が2にならない理由とか考えてドヤ顔でかましそうだな
435132人目の素数さん:2012/05/10(木) 23:08:29.72
|x-1|=-1を満たすxは存在しない


何故ですか?
436132人目の素数さん:2012/05/10(木) 23:08:52.95
するだろゴミ
437132人目の素数さん:2012/05/10(木) 23:11:39.74
皆さん、どうもありがとうございました。言えることは頭のよい人は羨ましいなと言うことです。
とりあえずスッキリは解りませんけど問題は解けますのでそのうち解ることもあるかな、と決めました。

>>427
えぇ確かにそれもあります。
因みに一次式で割ったので R は定数であるとかの説明もありますよね。

>>428
はい、自覚してます。
貴方のように頭脳明晰な人間ばかりではないことは知ってます。

>>432
そして P(a)=R となって余りになることは知ってます。スッキリ理解できないだけです。
438132人目の素数さん:2012/05/10(木) 23:13:37.73
え?キモいんだけど
439132人目の素数さん:2012/05/10(木) 23:14:30.54
>>437
理解できてないのは
P(a)=(a-a)*Q(a)+R=R
ではなく
P(x)=(x-a)*Q(x)+R
じゃないのか?
440132人目の素数さん:2012/05/10(木) 23:14:41.81
>>437
> そして P(a)=R となって余りになることは知ってます。スッキリ理解できないだけです。
実際になることを示して見せているのに理解出来ないといわれてもなあ。
441132人目の素数さん:2012/05/10(木) 23:17:20.91
感覚的な話だから
本人も書いてあるように
時間かけて理解するしかないと思う。
442132人目の素数さん:2012/05/10(木) 23:17:52.01
>>437
> えぇ確かにそれもあります。
それがわからんのなら話にならん。>>431がわからんってことだろ?
ちゃんと勉強してないだけだ。
443132人目の素数さん:2012/05/10(木) 23:18:19.60
||x-1|-2|-3=0

||x-1|-2|=3
|x-1|-1=±3
|x-1|=±3+2

|x-1|=5
x-1=±5
x=±5+1
x=6、-4

|x-1|=-1
x-1=±1
x=±1+1

x=0、2

x=6、-4、0、2


答えは6、-4なんだが
444132人目の素数さん:2012/05/10(木) 23:23:32.70
> ||x-1|-2|-3=0

> x=0、2
を入れても成り立たないからな
445132人目の素数さん:2012/05/10(木) 23:24:43.03
>>443
> |x-1|=-1
この時点で不適だが。
なんで、
> x-1=±1
なんだよw
446132人目の素数さん:2012/05/10(木) 23:28:40.78
絶対値は0以上の数だ
447132人目の素数さん:2012/05/10(木) 23:29:38.06
>>435に対して>>436と書かれたから
>>443
を書いたんだろ。(たぶん)
448132人目の素数さん:2012/05/10(木) 23:33:33.04
>>445
ならないのか

正のみ±?
449132人目の素数さん:2012/05/10(木) 23:34:49.68
>>448
絶対値の定義
450132人目の素数さん:2012/05/10(木) 23:35:12.33
> |x-1|=-1
> x-1=±1
この主張は
|±1|=-1
になるが当然間違い
|1|=1,|-1|=1
451132人目の素数さん:2012/05/10(木) 23:35:44.38
>>449
忘れてたわ


おまいらサンクス
452132人目の素数さん:2012/05/11(金) 00:13:22.50
          __ノ)-'´ ̄ ̄`ー- 、_
        , '´  _. -‐'''"二ニニ=-`ヽ、
      /   /:::::; -‐''"        `ーノ
     /   /:::::/           \
     /    /::::::/          | | |  |
     |   |:::::/ /     |  | | | |  |
      |   |::/ / / |  | ||  | | ,ハ .| ,ハ|
      |   |/ / / /| ,ハノ| /|ノレ,ニ|ル' 
     |   |  | / / レ',二、レ′ ,ィイ|゙/   私は只の数ヲタなんかとは付き合わないわ。
.     |   \ ∠イ  ,イイ|    ,`-' |      頭が良くて数学が出来てかっこいい人。それが必要条件よ。
     |     l^,人|  ` `-'     ゝ  |        さらに Ann.of Math に論文書けば十分条件にもなるわよ。
      |      ` -'\       ー'  人          一番嫌いなのは論文数を増やすためにくだらない論文を書いて
    |        /(l     __/  ヽ、           良い論文の出版を遅らせるお馬鹿な人。
     |       (:::::`‐-、__  |::::`、     ヒニニヽ、         あなたの論文が Ann of Math に accept される確率は?
    |      / `‐-、::::::::::`‐-、::::\   /,ニニ、\            それとも最近は Inv. Math. の方が上かしら?
   |      |::::::::::::::::::|` -、:::::::,ヘ ̄|'、  ヒニ二、 \
.   |      /::::::::::::::::::|::::::::\/:::O`、::\   | '、   \
   |      /:::::::::::::::::::/:::::::::::::::::::::::::::::'、::::\ノ  ヽ、  |
  |      |:::::/:::::::::/:::::::::::::::::::::::::::::::::::'、',::::'、  /:\__/‐、
  |      |/:::::::::::/::::::::::::::::::::::::::::::::::O::| '、::| く::::::::::::: ̄|
   |     /_..-'´ ̄`ー-、:::::::::::::::::::::::::::::::::::|/:/`‐'::\;;;;;;;_|
   |    |/::::::::::::::::::::::\:::::::::::::::::::::::::::::|::/::::|::::/:::::::::::/
    |   /:::::::::::::::::::::::::::::::::|:::::::::::::::::::::O::|::|::::::|:::::::::::::::/
453132人目の素数さん:2012/05/11(金) 01:32:47.81
auやっと規制解除された、、、1ヶ月以上かかった気がする。
454132人目の素数さん:2012/05/11(金) 03:20:42.71
>>365
>>366
335だけど確かにあの解答の仕方だと問題ありだった。

@ A & B = グーのとき
C & D = チョキ : 1通り
C & D = パー : 1通り
A A & C = グーのとき
B A & D = グーのとき

@は2通り、AとBは実は@と同じことをやっている
だから@だけ計算すればいいといったレトリックを脳内で保管していたようだ
組み合わせ数(2)を決定してから分母(3^2)で割るという順序


同じことを5人でやると@ABCで重複が発生するのでうまくいかない
@ A & B = グーのとき
CDE={チョキ*3} : 1通り
CDE={グー*1&パー*2)} : 3通り
A A & C = グー : 4
BDE={チョキ*3} : 1通り
BDE={グー*1&パー*2)} : 3通り (ただし@との重複1通り)
B A & D = グー : 4通り (@Aとの重複2)
C A & E = グー : 4通り (@ABとの重複3)

(4*4-1-2-3) / 3^4 = 10/81

続く
455132人目の素数さん:2012/05/11(金) 03:24:16.89
続き
素直に組み合わせを計算したほうが早かった

2人勝って2人負けたということは
勝負がついた→出た手の種類はグーチョキパーのうち2つ→ 3C2
勝ち側の手を出したのは4人中2人→ 4C2
総数 3^4
3C2 * 4C2 / 3^4 = 3 * 6 / 81 = 2 / 9

同様に5人中2人勝ちを計算すると
3C2 * 5C2 / 3^5 = 3 * 10 / 3^5 = 10 / 81
456132人目の素数さん:2012/05/11(金) 07:00:25.32
円周上にm個の赤い点とn個の青い点を任意の順序で並べる!これらの点により円周はm+n個の弧に分けられる。このとき、これらの弧のうち、両端の点の色が異なるものの数は偶数であることを示す。ただしm,n≧1とする。
457132人目の素数さん:2012/05/11(金) 07:18:39.84
>>456
イヤです
458132人目の素数さん:2012/05/11(金) 07:19:57.79
>>457
お願いします
459132人目の素数さん:2012/05/11(金) 08:34:49.05
>>456
パズル的な解き方なら出来たよ。
460132人目の素数さん:2012/05/11(金) 08:39:30.41
>>456
円周上のある一つの赤い点から時計回りに見ていくと、赤から始まり最後に赤で終わるのだから色の変化は必ず偶数回起こる
461132人目の素数さん:2012/05/11(金) 08:52:58.64
例えば、放物線Cを表す方程式y=f(x)と、直線Lを表す方程式y=g(x)があって、
CとLの位置関係は方程式f(x)-g(x)=0の判別式で判断できるんだけど、
それ以外の方法でも判断できるの?

「高校数学+α」てサイトに割り算云々と書かれていたんだが、理解できなかった…
462132人目の素数さん:2012/05/11(金) 10:02:14.52
>>461
>CとLの位置関係は方程式f(x)-g(x)=0の判別式で判断できる
まぁこれは定石。

>「高校数学+α」てサイトに割り算云々と書かれていたんだが
そのサイトのリンクか内容を貼って。内容解らないとコメントできない。

f(x)-g(x)=0 を因数分解で解く時、係数、定数項が実数の一次式でf(x)-g(x)が割り切れれば実数解となる。
当たり前だけど実数解になるのはf(x)-g(x)=0の判別式が D≧0 の時だから。 
463132人目の素数さん:2012/05/11(金) 10:48:32.68
まったくわかりません、やり方を教えてください


関数f(x)は次の条件を満たしている
(i)すべての実数xに対してf(3+x)=f(3-x)
(ii)xの値が、異なる5つの実数(a1),(a2),(a3),(a4),(a5)のときに限りf(x)=0となる
このとき(a1)+(a2)+(a3)+(a4)+(a5)の値を求めよ
464132人目の素数さん:2012/05/11(金) 11:28:41.23
>>463
1)5次の一般式を f(x) で書いて見て、まぁ5次の係数は1でも良いかと。
2)f(3+x)=f(3-x) は恒等式なので係数を見比べて等しくなるように。4次の部分だけで良い気もするけど。
3)f(x)=0 この手の方程式には解と係数の関係が存在していて
「(a1)+(a2)+(a3)+(a4)+(a5)」は 5次方程式の f(x)=0 の
4次の項の係数と絶対値は同じで符号は逆になる。
結局、恒等式 f(3+x)=f(3-x) の4次の部分だけ間違えずに計算すれば何とかなるのではと。

まぁこんな感じで予想してみた。
当方、計算嫌いだし苦手なのであとの検証はあなたの頑張りで!
465132人目の素数さん:2012/05/11(金) 11:36:38.44
追伸
2)f(3+x)=f(3-x) は恒等式なので計算しやすい任意の値をxに代入すると楽。
466132人目の素数さん:2012/05/11(金) 11:37:07.17
こんなん計算いらねぇよ
(3+0)+(3-a)+(3+a)+(3-b)+(3+b)=15
467132人目の素数さん:2012/05/11(金) 12:43:24.50
>>464>>466
わかりました、ありがとうございます
468132人目の素数さん:2012/05/11(金) 13:03:15.59
(・3・)
469132人目の素数さん:2012/05/11(金) 13:24:45.58
>>463

ホントに分かってるのかな?
この問題を特には

>関数f(x)は次の条件を満たしている
>(i)すべての実数xに対してf(3+x)=f(3-x)

からf(x)がx=3で線対称になっている事がまずわからないといけない。

>(ii)xの値が、異なる5つの実数(a1),(a2),(a3),(a4),(a5)のときに限りf(x)=0となる
>このとき(a1)+(a2)+(a3)+(a4)+(a5)の値を求めよ

グラフが線対称になっているって事は対称軸上の点以外は必ず対になるペアの相手がいる事に気付かないといけない。
異なる5つの実数だけ特定の値を満たすっていうのは、通常偶数じゃないとオカシイ特定の値を満たす点が奇数個あるから、x=3が解の一個になる事がわかる。
残りの4つの解は2ペアあって、それぞれが466の様に表せるから466の様になる。
470132人目の素数さん:2012/05/11(金) 13:33:00.96
>>469
とてもわかりやすいです
解説ありがとうございます
471132人目の素数さん:2012/05/11(金) 13:39:44.71
後、実数解が5個だから5次とか真っ赤な嘘だからな。ありえない。6次だって問題ない
というか5次なんか線対称になり得ん
そもそも条件を満たせば整式である必要さえない。
472132人目の素数さん:2012/05/11(金) 13:44:05.50
質問お願いします。
今、yが5ずつ増加する毎に、bは半減していく場合、式で表現するとどうなりますか。
473132人目の素数さん:2012/05/11(金) 13:52:28.71
半減期でググれ
474132人目の素数さん:2012/05/11(金) 13:56:13.83
半減期か?
頭の切れる人は想像がすごいね。ありがとう!!!
475132人目の素数さん:2012/05/11(金) 14:02:11.92
もう一度すまん。
yの値は、xが5ずつ減少する毎に、半減していく。
これを、式表現お願いします。自分高1です。
476132人目の素数さん:2012/05/11(金) 14:05:03.16
半減期 式でググればまんま5の例での奴が上に上がってくる。
なんでy使っているかといえばyearだから
そもそもその説明がまんま半減期の定義みたいなもん。
放射線関連だけじゃなくて、およその数を見積もる事とかにも半減期の考え形は良く使う。
477132人目の素数さん:2012/05/11(金) 14:09:03.93
高1じゃ下手したら指数計算もまともにわからないんじゃないか?
http://www7.plala.or.jp/stokida/l2h/hangenkie/node1.html
の一番下のグラフとそこに書いてある式
478132人目の素数さん:2012/05/11(金) 14:09:38.68
ありがとうございます。
がんばります。
479132人目の素数さん:2012/05/11(金) 14:11:27.79
b=(y=0の時のbの値)*(1/2)^(y/5)
480132人目の素数さん:2012/05/11(金) 15:41:57.11
質問お願いします

(3x+2)/(x^2+4x+5) 
xを実数とした場合の取りうる範囲を求めよ



481132人目の素数さん:2012/05/11(金) 15:54:26.41
>>480
>質問お願いします

お願いするのは考え方でいいんですか?

(3x+2)/(x^2+4x+5)=kとおいて
xが実数範囲で存在するkの条件を見る。
k=0とそうでない場合で場合わけするのを忘れないように
482132人目の素数さん:2012/05/11(金) 15:58:58.39
>>481
ありがとうございます おかげでわかりました
483132人目の素数さん:2012/05/11(金) 17:25:32.94
>>471
>実数解が5個だから5次とか真っ赤な嘘だからな
↑は書いてないだろ、勝手な妄想するな。ガウスは知ってるのだろ?
484132人目の素数さん:2012/05/11(金) 17:28:09.47
カウス・ボタンなら知ってるぞ
485132人目の素数さん:2012/05/11(金) 17:32:22.91
高認試験に向けて数学Tを不忍堂という無料サイトで勉強しているものです
質問があります
問題
キャンディーが63個ある、子供たちに一人3個ずつ配ると余りが10個以上になる
この時、子供の人数は最大で○○人と考えられる

という問題で不忍堂では
63−3I≧10
として
I≦17,66…
で、答、最大17人としていますが
486132人目の素数さん:2012/05/11(金) 17:36:12.91
続き
余りが10個以上になるとあるので
63−3I≦10個
になって
I≧17,66…
で、答、最大で18人じゃないかと思うのですが
どなたか詳しい方教えてください
487132人目の素数さん:2012/05/11(金) 17:36:46.32
>471
>6次だって問題ない
(x−3)^2 (x+α)(x−α)(x+β)(x-β)= 0
とすれば6次だし

>そもそも条件を満たせば整式である必要さえない。
流石に高校レベルを超えないかと。
488132人目の素数さん:2012/05/11(金) 17:39:22.34
なんで>>464他は整式の類を出してくるんだ
わけわからん
489132人目の素数さん:2012/05/11(金) 17:40:12.51
>>486
実際に17人と18人で計算すると
17人が正しいのですが…
490132人目の素数さん:2012/05/11(金) 18:07:09.61
g(a,x)=1-2^(a-x)や
g(a,x)=cos((e^-(a-x)^2)pi/2)などを用いて
f(x)=g(1,x)g(2,x)g(3,x)g(4,x)g(5,x)とでもすれば
かなり人工的ながら高校生レベルの非整式なf(x)くらい簡単につくれる
絶対値関数や定義域で分けてもいいならもっと簡単に作れるし
491132人目の素数さん:2012/05/11(金) 18:28:38.30
>>486
Aが10以上…A≧10
492132人目の素数さん:2012/05/11(金) 18:30:52.92
>>483
471がガウスの消去法を知ってるか知らないかを別にしてさ
一切の予備知識の説明なしに、その説明で463が分かると本気で思ってるのか?
そう思っているなら何も言うことはないよね
493132人目の素数さん:2012/05/11(金) 19:10:31.19
>>321 明後日になりました。宜しくお願いします。

320 名前: 132人目の素数さん 投稿日: 2012/05/09(水) 22:42:48.75
素数p, q で

 1+q が 1+p^2 を割り切る

を満たす組はないでしょうか。

494132人目の素数さん:2012/05/11(金) 19:23:04.71
三点O(0、0)、A(4、0)、B(2、2)を頂点とす三角形OABの面積を、
直線L:y=mx+m+1が二等分するとき、定数mの値を求めよ


まず直線Lが三角形OABのどの辺と交わるかを検討する必要があると思いますが、
皆さんならどのような手順によって検討しますか?
模範解答の説明がいまいち理解できなかったので…
495132人目の素数さん:2012/05/11(金) 19:29:39.95
>>494
まずは3パタンの全てを検討してみる
496132人目の素数さん:2012/05/11(金) 19:31:53.85
>>493
(179,73)とか(863,673)とか(7883,2689)とか
497493:2012/05/11(金) 19:43:30.08
>>496
どうもありがとうございます!!
あるんですね。存在しないのかと思ってました。
498132人目の素数さん:2012/05/11(金) 19:56:17.89
>>494

直線L:y=mx+m+1

まず直線Lがy-1=m(x+1)と変形できて
(-1,1)を通る傾きmの直線という事が分からない始まらない。
499132人目の素数さん:2012/05/11(金) 19:57:20.94
自分の答えが
2^(2n-3)+2^(2n-2)-2^(n-2)
と出たのですが、回答は
2^(n-2)(3*2^(n-1)-1)
となっていました。これは間違いになるのですか?

あと、変形の仕方がわかりません。
500132人目の素数さん:2012/05/11(金) 20:10:52.41
>>499
正直どっちでもいいと思う。
問題によっては文脈でベターな方がかわる。

2^(2n-3)+2^(2n-2)-2^(n-2)
=2^((n-2)+(n-1))+2^((n-2)+n)-2^(n-2)
=2^(n-2)(2^(n-1)+2^n-1)
=2^(n-2)(2^(n-1)+2*2^(n-1)-1)
=2^(n-2)(3*2^(n-1)-1)
501132人目の素数さん:2012/05/11(金) 20:21:31.17
>>491
63−3Iが余りを計算してるんですか?
502132人目の素数さん:2012/05/11(金) 20:38:27.20
>>501
何だと思ってたの?
503132人目の素数さん:2012/05/11(金) 20:45:35.76
>>502
あーーーーーーーーーーーーーーーーーーーーー分った!
分かりました!
ちょっと混乱してて
ありがとうございました
504132人目の素数さん:2012/05/11(金) 20:57:21.61
          __ノ)-'´ ̄ ̄`ー- 、_
        , '´  _. -‐'''"二ニニ=-`ヽ、
      /   /:::::; -‐''"        `ーノ
     /   /:::::/           \
     /    /::::::/          | | |  |
     |   |:::::/ /     |  | | | |  |
      |   |::/ / / |  | ||  | | ,ハ .| ,ハ|
      |   |/ / / /| ,ハノ| /|ノレ,ニ|ル' 
     |   |  | / / レ',二、レ′ ,ィイ|゙/   私は只の数ヲタなんかとは付き合わないわ。
.     |   \ ∠イ  ,イイ|    ,`-' |      頭が良くて数学が出来てかっこいい人。それが必要条件よ。
     |     l^,人|  ` `-'     ゝ  |        さらに Ann.of Math に論文書けば十分条件にもなるわよ。
      |      ` -'\       ー'  人          一番嫌いなのは論文数を増やすためにくだらない論文を書いて
    |        /(l     __/  ヽ、           良い論文の出版を遅らせるお馬鹿な人。
     |       (:::::`‐-、__  |::::`、     ヒニニヽ、         あなたの論文が Ann of Math に accept される確率は?
    |      / `‐-、::::::::::`‐-、::::\   /,ニニ、\            それとも最近は Inv. Math. の方が上かしら?
   |      |::::::::::::::::::|` -、:::::::,ヘ ̄|'、  ヒニ二、 \
.   |      /::::::::::::::::::|::::::::\/:::O`、::\   | '、   \
   |      /:::::::::::::::::::/:::::::::::::::::::::::::::::'、::::\ノ  ヽ、  |
  |      |:::::/:::::::::/:::::::::::::::::::::::::::::::::::'、',::::'、  /:\__/‐、
  |      |/:::::::::::/::::::::::::::::::::::::::::::::::O::| '、::| く::::::::::::: ̄|
   |     /_..-'´ ̄`ー-、:::::::::::::::::::::::::::::::::::|/:/`‐'::\;;;;;;;_|
   |    |/::::::::::::::::::::::\:::::::::::::::::::::::::::::|::/::::|::::/:::::::::::/
    |   /:::::::::::::::::::::::::::::::::|:::::::::::::::::::::O::|::|::::::|:::::::::::::::/
505132人目の素数さん:2012/05/11(金) 20:57:35.76
          __ノ)-'´ ̄ ̄`ー- 、_
        , '´  _. -‐'''"二ニニ=-`ヽ、
      /   /:::::; -‐''"        `ーノ
     /   /:::::/           \
     /    /::::::/          | | |  |
     |   |:::::/ /     |  | | | |  |
      |   |::/ / / |  | ||  | | ,ハ .| ,ハ|
      |   |/ / / /| ,ハノ| /|ノレ,ニ|ル' 
     |   |  | / / レ',二、レ′ ,ィイ|゙/   私は只の数ヲタなんかとは付き合わないわ。
.     |   \ ∠イ  ,イイ|    ,`-' |      頭が良くて数学が出来てかっこいい人。それが必要条件よ。
     |     l^,人|  ` `-'     ゝ  |        さらに Ann.of Math に論文書けば十分条件にもなるわよ。
      |      ` -'\       ー'  人          一番嫌いなのは論文数を増やすためにくだらない論文を書いて
    |        /(l     __/  ヽ、           良い論文の出版を遅らせるお馬鹿な人。
     |       (:::::`‐-、__  |::::`、     ヒニニヽ、         あなたの論文が Ann of Math に accept される確率は?
    |      / `‐-、::::::::::`‐-、::::\   /,ニニ、\            それとも最近は Inv. Math. の方が上かしら?
   |      |::::::::::::::::::|` -、:::::::,ヘ ̄|'、  ヒニ二、 \
.   |      /::::::::::::::::::|::::::::\/:::O`、::\   | '、   \
   |      /:::::::::::::::::::/:::::::::::::::::::::::::::::'、::::\ノ  ヽ、  |
  |      |:::::/:::::::::/:::::::::::::::::::::::::::::::::::'、',::::'、  /:\__/‐、
  |      |/:::::::::::/::::::::::::::::::::::::::::::::::O::| '、::| く::::::::::::: ̄|
   |     /_..-'´ ̄`ー-、:::::::::::::::::::::::::::::::::::|/:/`‐'::\;;;;;;;_|
   |    |/::::::::::::::::::::::\:::::::::::::::::::::::::::::|::/::::|::::/:::::::::::/
    |   /:::::::::::::::::::::::::::::::::|:::::::::::::::::::::O::|::|::::::|:::::::::::::::/
506132人目の素数さん:2012/05/11(金) 21:22:52.76
lim[x→0]sin(sinx)/sinxで
解答は
sinx=tとおいて、x→0のときt→0であるから
与式=lim[t→0]sint/t=1となっているんですが

lim[x→0]sin■/■=1のパターンで
すぐ1としてはダメですか?
507493:2012/05/11(金) 21:32:06.77
>>496
ちなみに これらの組はどのように見つけるのでしょうか。
また、このような組のうち、(179, 73)が“最小”のものでしょうか。
よかったら教えて下さい。
508132人目の素数さん:2012/05/11(金) 21:37:17.40
>>506
>lim[x→0]sin■/■=1のパターンで
>すぐ1としてはダメですか?

だめにきまってるだろw
lim[■ →0]sin■/■=1
ならいい。
509132人目の素数さん:2012/05/11(金) 21:51:47.41
>>508
あ、分かりました
ありがとうございます
510132人目の素数さん:2012/05/11(金) 22:02:55.32
>>498
そのことには気づきました
そしてmのとりうる値の範囲は直線Lが三角形OABと交点を持つ傾きの範囲として、−1<m<1/3としました

分からなくなったのはその後で、模範解答いわく
“三角形OCAの面積は、三角形OABのそれの半分に一致するため、直線Lは三角形OABの辺ABと交点を持つ”
とのことですが、これが理解できません…
ちなみに、点C(−1、1)です
511132人目の素数さん:2012/05/11(金) 22:04:34.84
          __ノ)-'´ ̄ ̄`ー- 、_
        , '´  _. -‐'''"二ニニ=-`ヽ、
      /   /:::::; -‐''"        `ーノ
     /   /:::::/           \
     /    /::::::/          | | |  |
     |   |:::::/ /     |  | | | |  |
      |   |::/ / / |  | ||  | | ,ハ .| ,ハ|
      |   |/ / / /| ,ハノ| /|ノレ,ニ|ル' 
     |   |  | / / レ',二、レ′ ,ィイ|゙/   私は只の数ヲタなんかとは付き合わないわ。
.     |   \ ∠イ  ,イイ|    ,`-' |      頭が良くて数学が出来てかっこいい人。それが必要条件よ。
     |     l^,人|  ` `-'     ゝ  |        さらに Ann.of Math に論文書けば十分条件にもなるわよ。
      |      ` -'\       ー'  人          一番嫌いなのは論文数を増やすためにくだらない論文を書いて
    |        /(l     __/  ヽ、           良い論文の出版を遅らせるお馬鹿な人。
     |       (:::::`‐-、__  |::::`、     ヒニニヽ、         あなたの論文が Ann of Math に accept される確率は?
    |      / `‐-、::::::::::`‐-、::::\   /,ニニ、\            それとも最近は Inv. Math. の方が上かしら?
   |      |::::::::::::::::::|` -、:::::::,ヘ ̄|'、  ヒニ二、 \
.   |      /::::::::::::::::::|::::::::\/:::O`、::\   | '、   \
   |      /:::::::::::::::::::/:::::::::::::::::::::::::::::'、::::\ノ  ヽ、  |
  |      |:::::/:::::::::/:::::::::::::::::::::::::::::::::::'、',::::'、  /:\__/‐、
  |      |/:::::::::::/::::::::::::::::::::::::::::::::::O::| '、::| く::::::::::::: ̄|
   |     /_..-'´ ̄`ー-、:::::::::::::::::::::::::::::::::::|/:/`‐'::\;;;;;;;_|
   |    |/::::::::::::::::::::::\:::::::::::::::::::::::::::::|::/::::|::::/:::::::::::/
    |   /:::::::::::::::::::::::::::::::::|:::::::::::::::::::::O::|::|::::::|:::::::::::::::/
512132人目の素数さん:2012/05/11(金) 22:15:52.74
>>507
計算はコンピューター任せ(何か効率的な探し方はあるのかもしれないが)。
例えばq=5がダメなのは、p^2=6k-1 (k:整数)をみたすような
(2乗を6で割った余りが5となる整数)pが存在しないことから示せる。
pが小さい順のリスト:
(31,73),(43,73),(73,409),(107,457),(151,1753),(173,409),(179,73),(191,73),
(193,1489),(239,337),(251,577),(269,193),(281,6073),(293,1009),(293,3433),
(307,3769),(313,193),(313,1009)…
面白そうだから(q≡1 mod24であるらしいこととか)暇なときにでも色々調べてみては?
513132人目の素数さん:2012/05/11(金) 22:16:12.58
          __ノ)-'´ ̄ ̄`ー- 、_
        , '´  _. -‐'''"二ニニ=-`ヽ、
      /   /:::::; -‐''"        `ーノ
     /   /:::::/           \
     /    /::::::/          | | |  |
     |   |:::::/ /     |  | | | |  |
      |   |::/ / / |  | ||  | | ,ハ .| ,ハ|
      |   |/ / / /| ,ハノ| /|ノレ,ニ|ル' 
     |   |  | / / レ',二、レ′ ,ィイ|゙/   私は只の数ヲタなんかとは付き合わないわ。
.     |   \ ∠イ  ,イイ|    ,`-' |      頭が良くて数学が出来てかっこいい人。それが必要条件よ。
     |     l^,人|  ` `-'     ゝ  |        さらに Ann.of Math に論文書けば十分条件にもなるわよ。
      |      ` -'\       ー'  人          一番嫌いなのは論文数を増やすためにくだらない論文を書いて
    |        /(l     __/  ヽ、           良い論文の出版を遅らせるお馬鹿な人。
     |       (:::::`‐-、__  |::::`、     ヒニニヽ、         あなたの論文が Ann of Math に accept される確率は?
    |      / `‐-、::::::::::`‐-、::::\   /,ニニ、\            それとも最近は Inv. Math. の方が上かしら?
   |      |::::::::::::::::::|` -、:::::::,ヘ ̄|'、  ヒニ二、 \
.   |      /::::::::::::::::::|::::::::\/:::O`、::\   | '、   \
   |      /:::::::::::::::::::/:::::::::::::::::::::::::::::'、::::\ノ  ヽ、  |
  |      |:::::/:::::::::/:::::::::::::::::::::::::::::::::::'、',::::'、  /:\__/‐、
  |      |/:::::::::::/::::::::::::::::::::::::::::::::::O::| '、::| く::::::::::::: ̄|
   |     /_..-'´ ̄`ー-、:::::::::::::::::::::::::::::::::::|/:/`‐'::\;;;;;;;_|
   |    |/::::::::::::::::::::::\:::::::::::::::::::::::::::::|::/::::|::::/:::::::::::/
    |   /:::::::::::::::::::::::::::::::::|:::::::::::::::::::::O::|::|::::::|:::::::::::::::/
514132人目の素数さん:2012/05/11(金) 22:24:09.16
>>510
△OCAは△OABと比べて底辺が同じで高さが半分なので、面積が半分であることはすぐにわかる。
ACとOBの交点をDとすると、△ODAは△OCAよりも面積が小さいから、△OABの半分より小さい。
だから、半分の面積にするためにはもっと拡げないとならない。
直線はCを通るのだから……
515132人目の素数さん:2012/05/11(金) 22:24:42.14
>>510
何も難しく考える事はなくてOCAの面積が三角形OABの二分の一なら
CAでOABを切ると当然下の三角形は1/2より小さいからOAと交点を持つ様に切ったら言うまでもなく、1/2より下の部分の面積が小さくなるよね
だからABと交点を持つはずだよね!ってのを簡略化して書いてある感じかな
516132人目の素数さん:2012/05/11(金) 22:33:33.74
>>512
重ね重ねありがとうございました。
色々考えてみます!!
517132人目の素数さん:2012/05/11(金) 22:36:12.21
409って素数なんだな。
パッと見、合成数っぽいけど。
「4」「9」につられて平方数とか一瞬思ってしまいそうだ。
518132人目の素数さん:2012/05/11(金) 23:30:51.96
3333333331は素数?
519132人目の素数さん:2012/05/11(金) 23:41:46.10
3333333331=673*4952947
520132人目の素数さん:2012/05/12(土) 00:15:52.12
xが負の整数のとき3x-5<7(x+1)+4を満たすxの値をすべて求めよ。という問題なんですが、解き方を教えて下さい
521132人目の素数さん:2012/05/12(土) 00:19:24.46
>>520
学校やめてしまえ
522132人目の素数さん:2012/05/12(土) 00:23:39.41
すいません間違えました普通に解けましたごめんなさい。
523132人目の素数さん:2012/05/12(土) 00:24:41.01
ここまで俺の自演
524 ◆27Tn7FHaVY :2012/05/12(土) 01:30:18.18
んだよ、質スレはAA連投されてないんかい。手動でやってんか?
525 ◆27Tn7FHaVY :2012/05/12(土) 01:33:07.45
って思ったら、すぐ上にあった。
さっき、設定したの忘れてた。失礼
526132人目の素数さん:2012/05/12(土) 01:56:40.14
ベクトル(または行列)の転置は,幾何学的に何を表しているのでしょうか?

例えば,右手系の直交座標系において大きさ1のx,y,z方向の,基底ベクトルをそれぞれex↑,ey↑,ez↑とします.
これらを列ベクトルe↑=[ex↑ ey↑ ez↑]^{T}と定義します.Tは転置を示します.
e↑は,図で表すと以下のようになるのは明らかです.
ttp://upup.bz/j/my35305kJZYtOQ2VjNKJbBE.jpg

この列ベクトルe↑を転置した行ベクトルe↑=([ex↑ ey↑ ez↑]^{T})^{T}=[ex↑ ey↑ ez↑]
を図で表すとどのようになりますか?
527132人目の素数さん:2012/05/12(土) 02:18:48.67
          __ノ)-'´ ̄ ̄`ー- 、_
        , '´  _. -‐'''"二ニニ=-`ヽ、
      /   /:::::; -‐''"        `ーノ
     /   /:::::/           \
     /    /::::::/          | | |  |
     |   |:::::/ /     |  | | | |  |
      |   |::/ / / |  | ||  | | ,ハ .| ,ハ|
      |   |/ / / /| ,ハノ| /|ノレ,ニ|ル' 
     |   |  | / / レ',二、レ′ ,ィイ|゙/   私は只の数ヲタなんかとは付き合わないわ。
.     |   \ ∠イ  ,イイ|    ,`-' |      頭が良くて数学が出来てかっこいい人。それが必要条件よ。
     |     l^,人|  ` `-'     ゝ  |        さらに Ann.of Math に論文書けば十分条件にもなるわよ。
      |      ` -'\       ー'  人          一番嫌いなのは論文数を増やすためにくだらない論文を書いて
    |        /(l     __/  ヽ、           良い論文の出版を遅らせるお馬鹿な人。
     |       (:::::`‐-、__  |::::`、     ヒニニヽ、         あなたの論文が Ann of Math に accept される確率は?
    |      / `‐-、::::::::::`‐-、::::\   /,ニニ、\            それとも最近は Inv. Math. の方が上かしら?
   |      |::::::::::::::::::|` -、:::::::,ヘ ̄|'、  ヒニ二、 \
.   |      /::::::::::::::::::|::::::::\/:::O`、::\   | '、   \
   |      /:::::::::::::::::::/:::::::::::::::::::::::::::::'、::::\ノ  ヽ、  |
  |      |:::::/:::::::::/:::::::::::::::::::::::::::::::::::'、',::::'、  /:\__/‐、
  |      |/:::::::::::/::::::::::::::::::::::::::::::::::O::| '、::| く::::::::::::: ̄|
   |     /_..-'´ ̄`ー-、:::::::::::::::::::::::::::::::::::|/:/`‐'::\;;;;;;;_|
   |    |/::::::::::::::::::::::\:::::::::::::::::::::::::::::|::/::::|::::/:::::::::::/
    |   /:::::::::::::::::::::::::::::::::|:::::::::::::::::::::O::|::|::::::|:::::::::::::::/
528132人目の素数さん:2012/05/12(土) 05:51:19.06
          __ノ)-'´ ̄ ̄`ー- 、_
        , '´  _. -‐'''"二ニニ=-`ヽ、
      /   /:::::; -‐''"        `ーノ
     /   /:::::/           \
     /    /::::::/          | | |  |
     |   |:::::/ /     |  | | | |  |
      |   |::/ / / |  | ||  | | ,ハ .| ,ハ|
      |   |/ / / /| ,ハノ| /|ノレ,ニ|ル' 
     |   |  | / / レ',二、レ′ ,ィイ|゙/   私は只の数ヲタなんかとは付き合わないわ。
.     |   \ ∠イ  ,イイ|    ,`-' |      頭が良くて数学が出来てかっこいい人。それが必要条件よ。
     |     l^,人|  ` `-'     ゝ  |        さらに Ann.of Math に論文書けば十分条件にもなるわよ。
      |      ` -'\       ー'  人          一番嫌いなのは論文数を増やすためにくだらない論文を書いて
    |        /(l     __/  ヽ、           良い論文の出版を遅らせるお馬鹿な人。
     |       (:::::`‐-、__  |::::`、     ヒニニヽ、         あなたの論文が Ann of Math に accept される確率は?
    |      / `‐-、::::::::::`‐-、::::\   /,ニニ、\            それとも最近は Inv. Math. の方が上かしら?
   |      |::::::::::::::::::|` -、:::::::,ヘ ̄|'、  ヒニ二、 \
.   |      /::::::::::::::::::|::::::::\/:::O`、::\   | '、   \
   |      /:::::::::::::::::::/:::::::::::::::::::::::::::::'、::::\ノ  ヽ、  |
  |      |:::::/:::::::::/:::::::::::::::::::::::::::::::::::'、',::::'、  /:\__/‐、
  |      |/:::::::::::/::::::::::::::::::::::::::::::::::O::| '、::| く::::::::::::: ̄|
   |     /_..-'´ ̄`ー-、:::::::::::::::::::::::::::::::::::|/:/`‐'::\;;;;;;;_|
   |    |/::::::::::::::::::::::\:::::::::::::::::::::::::::::|::/::::|::::/:::::::::::/
529132人目の素数さん:2012/05/12(土) 08:12:13.59
90%で0点、6%で1点、4%で2点があたるくじを10回ひいて
合計がn点以上になる確率をおしえてくらさい
530132人目の素数さん:2012/05/12(土) 08:27:02.33
>>529
ありえるnについてそれぞれ地道に計算する。
531132人目の素数さん:2012/05/12(土) 08:40:34.93
そのクジ引きは、引いたクジは戻すor毎回補充するのか。
普通クジ引きというと引いたクジは戻さないがすると引くたびに確率が変わる。
全本数が不明では計算しようがない。
532132人目の素数さん:2012/05/12(土) 11:50:05.44
>>531
> 引くたびに確率が変わる。

変わらないと仮定するのが普通だと思うが。
て言うか、馬鹿は ROM ってなよ。
533132人目の素数さん:2012/05/12(土) 12:15:57.81
>>526
>列ベクトルe↑=[ex↑ ey↑ ez↑]^{T}
   1 0 0
e = 0 1 0
   0 0 1
ベクトル並べたら行列なので、eはベクトルじゃない。
>
図は
んー、適当に言うとね?
[x, y, z]∈R^{3}


図は、ex↑、ey↑、ez↑

行列とは何か?
単純に数を並べただけともとれるし(その時の行列の積や和に意味があるかは分からんけど)、色々な場合に当て嵌めれる。
一例として、線形写像って考えることが多いかなぁ。
線形写像って?
「体上の加群としての準同型写像」(wikipedia、線形写像より)
534132人目の素数さん:2012/05/12(土) 12:28:32.76
男子5人女子4人が1列に並ぶとき、どの女子も隣り合わない並び方は何通りですか?解き方を教えて下さい。
535132人目の素数さん:2012/05/12(土) 12:32:04.97
先に男子を並べます
○○○○○○

その間に女子を入れていきます
∧○∧○∧○∧○∧○∧○∧

∧のところから4つ選べばいい
536132人目の素数さん:2012/05/12(土) 12:34:23.40
>>535
解説に、6P4って書いてあるんですが、それだと7P4になりませんか?
537132人目の素数さん:2012/05/12(土) 12:36:15.32
ごめん
男子○が一つ多かった

先に男子を並べます
○○○○○

その間に女子を入れていきます
∧○∧○∧○∧○∧○∧

∧のところから4つ選べばいい

に訂正
538132人目の素数さん:2012/05/12(土) 12:37:15.52
>>535の○がひとつ多いだけ。
考え方を読め。
539132人目の素数さん:2012/05/12(土) 12:37:35.00
バカみたいに鵜呑みにするなよ。
まず男を並べるって時点で丸が六個並んでるから、男子5人なのに勢いあまって6人にしちゃったってのが分かるじゃん
540132人目の素数さん:2012/05/12(土) 12:38:09.01
ごめんなさい皆さんありがとうございます
541533:2012/05/12(土) 12:44:31.25
途中で投稿してしまったorz

>列ベクトルe↑=[ex↑ ey↑ ez↑]^{T}
   1 0 0
e = 0 1 0
   0 0 1
ベクトル並べたら行列なので、eはベクトルじゃない。
因みに、
>e↑は,図で表すと以下のようになるのは明らかです.
ttp://upup.bz/j/my35305kJZYtOQ2VjNKJbBE.jpg
この図は、ex↑、ey↑、ez↑の3つのベクトルを書いてあるだけでeを書いているわけではない。

んー、適当に言うとね
なんだかよく分からにけど変数x, y, zを並べてみたものがベクトル r↑=[x, y, z]^{T}
さらにこのr↑をあえて、とあるベクトルex↑, ey↑, ez↑を数倍して足したもので表現すると
r↑ = x*ex↑+ y*ey↑ + z*ez↑
こんなかんじに書くことができる。
因みに、ex↑、ey↑,ez↑を使ったけどex↑=[1, 0, 0]^{T}とかを使わなくてもいい。a1↑=[1, 1, 0]^{T}とか使って
r↑ = k1*a1↑+ k2*a2↑+ k3*a3↑
としてもいい。ここで、∀k1, k2, k3∈R(実数)。
で、この時使ったベクトル(ex↑、ey↑、ez↑とかa1↑、a2↑、a3↑)を基底ベクトルって呼ぶ。
係数(x、y、zとかk1、k2、k3)を並べたベクトル[x, y, z]^{T}を数ベクトルとか言ったりする。
542132人目の素数さん:2012/05/12(土) 12:46:12.14
ピタゴラス数についての質問です
原始的ピタゴラス数は
a=m^2-n^2
b=2mn
c=m^2+n^2
(m,nは互いに素でどちらかが偶数の自然数)
であらわされるとおもうんですけど
aとcが互いに素であることが証明できません
助けてください
543132人目の素数さん:2012/05/12(土) 12:54:12.33
>>542
aとcに共通因数があると仮定するとbも巻き添えで同じ因数を持つが
それは原始的ピタゴラス数の定義に反する
544132人目の素数さん:2012/05/12(土) 12:55:41.17
生徒2人と先生6人が円卓のまわりに座るとき、生徒が向かい合うような並び方は何通りですか?解き方を教えて下さい。
545132人目の素数さん:2012/05/12(土) 13:04:42.65
なんか6!になるらしいのですが、なぜそうなるかがわかりません…
546132人目の素数さん:2012/05/12(土) 13:06:43.88
生徒2名は固定
 ○
□ □
□ □
□ □
 ●

□に先生いれるだけ
547132人目の素数さん:2012/05/12(土) 13:09:58.17
>>546
生徒が入れ替わった場合とかかけなくていいんですか?そこがよくわからないんです…
548132人目の素数さん:2012/05/12(土) 13:15:05.45
 ○
A F
B E
C D
 ●

 ●
D C
E B
F A
 ○

いれかわったとして
円卓で線対称的に並んでるから数えられてる

円順列と言うものをもう一度復習してください
549132人目の素数さん:2012/05/12(土) 13:16:48.35
>>547
白丸君を自分と思え!
自分から数えて右から何番目にナントカ先生がいる場合って考えたらいいだろ。
550548:2012/05/12(土) 13:17:39.73
説明が下手だった
雰囲気で察してくれ
551132人目の素数さん:2012/05/12(土) 13:20:34.20
>>549
わかりました!ありがとうございます

>>548
ありがとうございます

549を見るまで、なぜ隣り合うだと×2なのにこっちはしないのかがわかりませんでした

二人ともありがとうございます
552132人目の素数さん:2012/05/12(土) 13:24:37.66
>>551
549みたいにリアルなイメージで考えるの大切。
一般でnのケースで考える問題だったらnが1のとき2のとき3のときを考えて見る
みたいに、実感が出来るレベルで想像して拡張するクセをつけよう。
553132人目の素数さん:2012/05/12(土) 13:42:27.32
>>552
ありがとうございます!
554132人目の素数さん:2012/05/12(土) 13:48:56.91
平和やな〜
555132人目の素数さん:2012/05/12(土) 16:02:51.02
以下の問で疑問点があります

f(x) は x≠0 である実数 x について定義された連続関数であり x≠0, y≠0 であるすべての実数 x, y について関係式 f(x)-f(y)=(x-y)f(x)f(y) を満たす
f(x) は x≠0 である x について微分可能であることを示し,f'(x) を f(x) を用いて表せ

方針はなんとなく y=x+h とおいて h→0 の微分係数の式に帰着させるのだとわかりますが
そうすると f(x+h)-f(x)=hf(x)f(x+h) となり両辺を h で割る際に0かどうかの場合分けをする必要があると思うんですがよくわかりません
それとも x, y はともに変数ですから y=x+h とおいた際にすでに x≠yのものとする ということが暗黙裡に了解されているのでしょうか?
お願いします
556132人目の素数さん:2012/05/12(土) 16:10:27.91
>>555
hは0に近づくが0ではない
557132人目の素数さん:2012/05/12(土) 16:13:13.80
>>555
> そうすると f(x+h)-f(x)=hf(x)f(x+h) となり両辺を h で割る際に0かどうかの場合分けをする必要があると思うんですがよくわかりません
h→0はh≠0のもとでhを0に近づける操作

というか基本的な整式を微分する場合でも分母分子をhで割ることはやっているはずだが?
558132人目の素数さん:2012/05/12(土) 16:17:47.99
>>556
>>557
それは定義に依る
559132人目の素数さん:2012/05/12(土) 16:21:02.81
>>556
0に近づくという表現はh≠0つまりx≠yだからこそできると思うのですが
xとyがたまたま等しかった場合はhは0に近づくという表現自体できずh=0だと思うんです

http://homepage2.nifty.com/skimp-studio/htm/crawl2/img/2_1_diff1_4.png
こういう図だと明らかにh≠0なるhをおいているので納得なのですが
関係式 f(x)-f(y)=(x-y)f(x)f(y)においてh≠0つまりx≠yとは読み取れないと思うんです

なんか根本的に間違ってますかね?
560132人目の素数さん:2012/05/12(土) 16:24:02.57
>>558
横からすまんけど
微分の定義とか(弱微分とか)そういう話?
561132人目の素数さん:2012/05/12(土) 16:28:53.51
>>559
>xとyがたまたま等しかった場合

意味わからん、h ≠ 0 の場合に、なぜ x = y なんてなるんだ?
562132人目の素数さん:2012/05/12(土) 16:29:43.98
>>559
> 関係式 f(x)-f(y)=(x-y)f(x)f(y)においてh≠0つまりx≠yとは読み取れないと思うんです

> f(x)-f(y)=(x-y)f(x)f(y)
これを見てどうやって
> h≠0つまりx≠y
が出てくるんだよ

h→0 があるから h≠0 が言える
h→0 がどこから来るかだったら微分の定義
563132人目の素数さん:2012/05/12(土) 16:32:11.35
>>559
多分根本的に間違ってる。
>関係式 f(x)-f(y)=(x-y)f(x)f(y)においてh≠0つまりx≠yとは読み取れない
それは当然で、今は計算のためにh→0となる場合を考えてる。
564132人目の素数さん:2012/05/12(土) 16:40:08.07
>y=x+h とおいた際にすでに x≠yのものとする ということが暗黙裡に了解されているのでしょうか?
了解されていません。
x, yは、x≠0, y≠0 を満たす範囲で任意なので、x=y を取ることは可能です。
また、y=x+hと定義し、h=0を取ることも可能です。

微分の定義は

lim_[h→0] (f(x+h)-f(x))/h

が存在することです。
因みに、h→0にはh≠0という意味が含まれます。
(h≠0が含まれないというのは、ε-δ論法とかで調べると納得できるかも。
ここで"近づく"とか"収束"とか極限を、厳密に数式で定義している)

なので、h=0はとれるけど、今考える必要がない、といったところだろうか
565132人目の素数さん:2012/05/12(土) 16:40:10.99
>>563
わかった気がします
つまり解答には y=x+h (以下h≠0として議論をすすめる) と書くべきなのでしょうが
微分の定義からそれは明らかなので書くまでもなくhで割って良いということですね
h→0 の微分係数の式においてhが0かどうかの吟味は今後100%要らないのでしょうか
566132人目の素数さん:2012/05/12(土) 16:44:26.18
>>564
すっきりしました!
高校の範囲では極限の定義は曖昧というのはよく聞く話なので
大学でがんばろうと思います

皆さんありがとうございましたm(_ _)m
567132人目の素数さん:2012/05/12(土) 17:01:36.37
>>564
杉浦解析入門1巻
1章セクション6定義2および
2章セクション1定義1を見よ
568132人目の素数さん:2012/05/12(土) 17:05:59.80
> 因みに、h→0にはh≠0という意味が含まれます。
> (h≠0が含まれないというのは、ε-δ論法とかで調べると納得できるかも。
> ここで"近づく"とか"収束"とか極限を、厳密に数式で定義している)
569132人目の素数さん:2012/05/12(土) 17:24:00.03
昨日も質問しました
高認数学独学中の者です
昨日はありがとうございました

6分の-3±√21って
分母の6と分子の-3は約分できない?しないみたい?
なんででしょうか?
570132人目の素数さん:2012/05/12(土) 17:29:09.75
(-3±√21)/6

-1/2±√21/6

どちらが見やすい?
571132人目の素数さん:2012/05/12(土) 17:30:55.38
>>569
まず、>>1を読め。

(3+a)/3=(1+a)/3は正しいのか?っていうこと?正しくないよ。
小学校レベル。
572132人目の素数さん:2012/05/12(土) 17:32:07.74
>>570
(-3±√21)/6 を(-1±√21)の2
にはできないのでしょうか?
573132人目の素数さん:2012/05/12(土) 17:32:52.21
>>572
できない
574132人目の素数さん:2012/05/12(土) 17:34:11.51
>>573
なんで?なんで?
出来ないで覚えろでおkでしょうか?
575132人目の素数さん:2012/05/12(土) 17:35:08.64
>>569
まずは>>1-3を読んで記法を覚える

>>572
できない
(-3±√21)/6=-1/2±√21/6
(-1±√21)/2=-1/2±√21/2
576132人目の素数さん:2012/05/12(土) 17:38:12.40
(-3±√21)/6=-3/6±√21/6=-1/2±√21/6
(-1±√21)/2=-1/2±√21/2
577132人目の素数さん:2012/05/12(土) 17:41:09.89
中身微分てどういう意味?
578132人目の素数さん:2012/05/12(土) 17:43:46.63
(-3±√21)/6は-3と±√21の両方を割る6してるので
(-3±√21)/6≠(-1±√21))/2になるのね
579132人目の素数さん:2012/05/12(土) 18:04:26.42
>>574
(12+18)/3は書けば括弧は要らないけど意味は(12+18)÷3 ってこと。
計算すれば答えが10になるのは明らかだけど
12か18のどちらかを3で割って、そのあと「4+18」とか「12+6」だと結果違うだろ?
つまり分子が足す、引くの形式になってる場合、分子の一部とは約分できない。
無理数や複素数になっても同じ事。

多分(12×18)/3と混同してるのだろうな、この時は12か18のどちらかと約分しても結果は同一。
580132人目の素数さん:2012/05/12(土) 18:51:23.11
赤、青、黄の3箱があり、無作為に一箱を選び、そこに腐ったミカンを1つ入れた。
さらに普通のミカンを3つの箱に入れていき、いま赤箱には100個、青箱には80個、黄箱には20個のミカンが入っている。
A、B、Cの3人がそれぞれ赤箱、青箱、黄箱のミカンを1個ずつ取り出して調べ、腐ったミカンを見つけようとしている。
3人とも、1個のミカンを調べるのにかかる時間は等しい。
また、調べている箱のミカンをすべて調べ尽くしたら、
自分が見つける確率が多くなるように他の箱に移り(2人ないし3人共同で)調査を続行する。
なお箱のミカンを調べ尽くさない限り他の箱には移れない。
このとき、Cが腐ったミカンを見つける確率はいくらか。

この問題で、Cが黄箱を調べ尽くしたら、次は青箱に移るべき、というのは明らかでしょうか。
解答には特に断りなく青箱に移るものとして書いてあったのですが・・・
黄→赤→青と黄→青→赤を両方計算すると確かに後者の方が確率は高かったのですが。
581132人目の素数さん:2012/05/12(土) 18:55:01.10
>>324

i) 0≦a<2 , 0≦b<2 のとき
   (左辺)−(右辺)=ab−2a−2b+4=(a−2)(b−2)>0 ,
ii) -2<a≦0 , -2<b≦0 のとき
   (左辺)−(右辺)=ab+2a+2b+4=(a+2)(b+2)>0 ,
iii) -4<ab<0 , 0≦a+b<2 のとき
   (左辺)−(右辺)=−ab−2a−2b+4>0−4+4=0 ,
iv) -4<ab<0 , -2<a+b≦0 のとき
   (左辺)−(右辺)=−ab+2a+2b+4>0−4+4=0 .
多少の重複もあるが、稚拙にすべての場合を調べて、該不等式が真なることが示せた。
582132人目の素数さん:2012/05/12(土) 18:57:50.19
>>579
分かりやすいです
ありがとうございました
583132人目の素数さん:2012/05/12(土) 19:30:27.88
>>580
確率の場合は直感に反する問題が多いんで
>>580みたいにパタンを確かめて答えを出すほうがいい
その本だか参考書だかの解答の仕方はちょっとおかしいんで、あんまり信じない方が良い
584132人目の素数さん:2012/05/12(土) 19:46:44.51
>>331

>>330
(|x+y|+|x-yl)^2
=2(x^2+y^2) + 2lx^2-y^2l
=Max{4x^2 , 4y^2} < 4

は、たんに、

(|x+y|+|x-yl)^2
=2(x^2+y^2)+2lx^2−y^2l

4x^2 (|x|≧|y|)
={
4y^2 (|x|<|y|)

<4

だよ。
585132人目の素数さん:2012/05/12(土) 19:49:09.00
>>584
4x^2 (|x|≧|y|)
={
4y^2 (|x|<|y|)
は、
   4x^2 (|x|≧|y|)
={
   4y^2 (|x|<|y|)
586132人目の素数さん:2012/05/12(土) 19:58:59.31
致違いで申し訳ございません。
どこにもこれから質問するものに関連するものがなかったのでここで質問させていただきます。

今現在研究で(大学生4年です)解析をしています。
解析手法としては線形加速度法という陰解法を使用していますが、
この手法の欠点としては、ステップ間隔を短くしなければなりません。

実際研究で使う地震波は、100秒もあり、扱う対象のものは
固有円振動数だけで330個、その中の最小のものより1/60程度にしなければ
なりません。そうしなければ、解は拡散してしまうからです。

そこで、0.0005秒間隔で100秒までとなると20万ステップしなければなりません。
構造解析のソフトウェアではそれにどれだけいったいかかるのか
図ったことはありませんが、最適化ですのでこれを少なくとも100回は回していかなければ
なりません。線形加速度法で扱う行列は、330行列。。。
この前0.0005秒間隔で20000ステップやるのですら、
1時間かかていました。。 20万ステップですから単純計算で
一回終えるのに10時間。これが最低でも100回回さなければならないため、
1000時間かかってしまいます。
587132人目の素数さん:2012/05/12(土) 19:59:32.93
そこで、解析手法を変えようと思っているのですが、
どの方法が適切でしょうか?何卒ご協力お願い申し上げます。
588132人目の素数さん:2012/05/12(土) 20:06:43.06
>>587
こういうスレもある

分からない問題はここに書いてね369
http://uni.2ch.net/test/read.cgi/math/1336490850/

あとはプログラム板で
アルゴリズム漁るとか
並列処理がんばるとか
スパコン買うとか
589132人目の素数さん:2012/05/12(土) 20:09:59.21
>>588
プログラム板ですね。。。お手数をおかけしまして誠に申し訳ございません。
大変ありがとうございます。失礼致します。
590132人目の素数さん:2012/05/12(土) 21:09:00.30
さてと>>586がプログラム板に行ってどう言う扱い受けるか偵察に行くとするか。

ラジアン って何? 聞く奴とか そんなもん中学校の教科書で調べろ、とか言う奴いるし。
591132人目の素数さん:2012/05/12(土) 21:11:27.36
スレタイに高校生のためって書いてあんのに
592132人目の素数さん:2012/05/12(土) 21:12:06.76
ごるちだし
593132人目の素数さん:2012/05/12(土) 21:16:17.47
>>590
れぽよろ
594132人目の素数さん:2012/05/12(土) 21:18:31.73
まぁしかし時には〜♪(古いな)小学校に近いレベルもあれば大学レベルでの言い争いもあったりするし。
595132人目の素数さん:2012/05/12(土) 21:19:35.18
2次関数 f=−x^2+6x+a (1≦x≦4)
の最小値が−2であるように、定数aの値を定めよ。

どなたか教えてくださいm(_ _)m
596132人目の素数さん:2012/05/12(土) 21:21:43.66
>>595
学校を辞めてしまえ
597132人目の素数さん:2012/05/12(土) 21:30:53.39
>>595
f(x)=-(x-3)^2+9+a
軸x=3は[1,4]の中にあるんだからf(3)が最大
するとf(1)が最小になるのが分かる

つまりf(1)=a+5=-2
598132人目の素数さん:2012/05/12(土) 21:32:46.86
599580:2012/05/12(土) 21:41:51.83
>>583 どうもです。

ところで>580の問題では、個数を一般化しても、
赤箱>青箱>黄箱であればCは黄箱を調べ尽くしたら青箱に移るべき、という結論になるでしょうか
600132人目の素数さん:2012/05/12(土) 21:43:15.24
>>597
ありがとうございます
601132人目の素数さん:2012/05/12(土) 23:54:53.57
n を自然数とするとき、
  p^n+q^n=1  
を満たす正の有理数の組 (p,q) が存在するための必要かつ十分な条件は
  n=1 or 2  
である、ということを証明し、それを解説してください。

602132人目の素数さん:2012/05/13(日) 00:00:44.45
>>593
知ったかが現れて詳しくと言うと「いやだよーん 」と逃げて行って
恥の上塗りだけが残って the end でした。
http://uni.2ch.net/test/read.cgi/math/1336490850/
603132人目の素数さん:2012/05/13(日) 00:02:27.90
>>602
本人登場
604132人目の素数さん:2012/05/13(日) 00:14:17.56
>>603
まるで自己紹介のようですね。
605132人目の素数さん:2012/05/13(日) 00:20:30.15
>>601
ふぇるーまたんはぁはぁ
606132人目の素数さん:2012/05/13(日) 00:25:46.63
>>601って
実際数学界のどんなところに役にたったんですか?
607甜菜:2012/05/13(日) 00:32:08.19
>>586
知らねーよ
教授でも先輩でも聞ける相手は近くにいんだろが
608132人目の素数さん:2012/05/13(日) 00:32:47.49
それを解く過程で色んな道具が考案されたこと。
609甜菜2:2012/05/13(日) 00:40:34.04
>>586
議論は研究室でやるものだよ
相談できる教授や先輩がいないなら、その研究室は残念ながらよい研究室ではない

それから、他の人に相談する場合は、必ず自分だったらこういう方法が考えられる
と思っていますが、他に良い方法などはありますか?的な聞き方をするのが鉄則

単にわかんないからおせーてでは、怒り狂う人もいる
610132人目の素数さん:2012/05/13(日) 01:50:55.97
ベクトル(マトリックス)がいまいち理解できません.
右手系の直交座標系において大きさ1のx,y,z方向の,基底ベクトルをそれぞれex↑,ey↑,ez↑としたとき,
これらをマトリックスe↑=[ex↑ ey↑ ez↑]^{T}とする.Tは転置を示す.

このときe↑=[ex↑ ey↑ ez↑]^{T}={[ex↑ ey↑ ez↑]^{T}}^{T}が成り立ちますか?

また,
   1 0 0
e = 0 1 0
   0 0 1
だとしたとき,これは3本の列ベクトルとして直行座標系で考えられますか?
これを図示する場合は,3本の列ベクトルで表すのはおかしいですか?
ttp://upup.bz/j/my35305kJZYtOQ2VjNKJbBE.jpg

3本の列ベクトル(もしくは写像?)で表せないとすると原点Oから座標(1,1,1)に向かう一本のベクトルと考えればいいんですか?
とにかく3次元なのでビジュアライズできると勝手に考えているんですが・・・どうなんでしょう
611132人目の素数さん:2012/05/13(日) 05:03:24.50
マトリックスと言うからにはex↑,ey↑,ez↑は数ベクトルでe↑は単位行列だから転置で変わらん。
図示に問題は無いが、単位行列でない場合はどう考えるのかね?
(1,1,1)はダメ。
612132人目の素数さん:2012/05/13(日) 07:36:49.01
東大の学生かなんかが 「なんちゃらビジュアライゼーション」っていう
ソフト開発してなかったっけ?
ウォーラルヴィジュアライゼーションだっけ?
613132人目の素数さん:2012/05/13(日) 07:37:23.24
あれ使えば視覚化できるんだけどな。
614甜菜3:2012/05/13(日) 07:56:01.31
>>586
eriのeic借りたら?
共同利用機関だから教員経由で使えるはずだよ
いま使ってるPCのスペックは知らんが、8cpu借りるだけでも
それよりずっと速いっしょ。8倍速以上になるから1週間で終わる。

つか、ここでヒント貰ったら参考文献どうすんのよw
615132人目の素数さん:2012/05/13(日) 09:59:40.24
参考文献:2ちゃんねる 数学板 「高校生のための数学の質問スレPART331」 (2012,5月)

突っ込みどころが多すぎる
616132人目の素数さん:2012/05/13(日) 11:16:50.10
>>615
くっそわろた
617132人目の素数さん:2012/05/13(日) 11:27:30.03
別に参考したもの全てに文献付けないといけないわけじゃないし、いいんじゃねぇの?
アドバイスぐらいなら誰しも色々な奴から貰うだろ。書くとしたら卒論の謝辞になるな。修論でもなきゃネタとしてもありじゃん(笑)
糞みたいな卒論沢山あるし。
618132人目の素数さん:2012/05/13(日) 12:28:24.68
これがこうなる理由を教えて下さい
http://beebee2see.appspot.com/i/azuY1fSpBgw.jpg
619132人目の素数さん:2012/05/13(日) 12:33:50.26
>>618
(sinθ)^2で約分しただけ。
620132人目の素数さん:2012/05/13(日) 12:36:01.33
難しく考えすぎてると予想
これは分母をsin^2θでくくるだけ

分母 = sin^2θ((1/cos^2θ)-1)
621132人目の素数さん:2012/05/13(日) 12:36:04.31
>>611
細かいことですが,基底ベクトルex↑,ex↑,ez↑は大きさと向きを持つから数ベクトルではなく幾何ベクトルですよね?
単位行列でなくても1列目,2列目,3列目で3本のベクトルと考えられるので,それぞれx,y,z軸に図示できると考えています
もちろんこれは数ベクトルではなく,幾何ベクトルでの話です
622132人目の素数さん:2012/05/13(日) 12:40:19.23
>>619>>620
ありがとうございます!
623132人目の素数さん:2012/05/13(日) 12:51:18.62
>>621
指摘しても思い込みから抜け出せんのは放置。
624132人目の素数さん:2012/05/13(日) 13:06:10.83
>>623
指摘とはどの部分についてでしょうか,文章が抽象的すぎて分からないです.
ともかく,基底ベクトルは数ベクトルではなく,幾何ベクトルですよね?理由は向きと大きさを持つからです.
625132人目の素数さん:2012/05/13(日) 13:06:37.58
>>610
e↑は何行何列と思っている?
626132人目の素数さん:2012/05/13(日) 13:34:31.89
>>624
基底ベクトルは幾何ベクトルで合ってるよ
627甜菜4,5:2012/05/13(日) 13:56:26.10
>>586
建築板で聞けよとおもうがw

1000時間で研究が終わるのならそれで十分でしょ
628132人目の素数さん:2012/05/13(日) 14:33:10.79
>>610
数学専門じゃないんで、違ったらすまん。

>とにかく3次元なのでビジュアライズできると勝手に考えているんですが・・・どうなんでしょう
ベクトルを図示することはできるけど、行列を図示するのはできないと思う

>マトリックスがいまいち理解できません
m次元からn次元への写像f

f : R^[m] → R^[n]
x↑∈R^[m] → f(x↑)∈R^[n]

の内、線型性(f(x↑+y↑) = f(x↑) + f(y↑), f(cx↑) = cf(x↑))を持つものはn×m行列Aを用いて表現できる。
つまり、行列というのは sin(x)とか、関数(写像)の仲間。

例えば、物体(3次元空間上の点)をカメラで撮影したら写真上(2次元上)のどの点に写るか、を2×3行列Bを使って

[x', y']^[T] = B [x, y, z]^[T]

と表現できる。
これは、3次元空間の点を、2次元空間(写真の上)の点に変換してるわけだが、もうちょっと一般化すると、
点(元)をとあるm次元ベクトル空間から別のn次元ベクトル空間へ対応させるのが行列、と言える。
(まあ、"別の"n次元ベクトル空間って考える必要もないのかなぁ。
小難しく考えず、固有ベクトル方向に固有値倍してるだけ、とかの方がいいのか?)
629132人目の素数さん:2012/05/13(日) 14:48:26.78
実数成分の行列、ベクトルとベクトルを成分とするベクトル、行列の間の関係を
幾何的に説明したい、と思ってるんじゃないのか?
>>610 で書かれている e↑ を文字通り何行何列と考えているかに返事がもらえればな。
630132人目の素数さん:2012/05/13(日) 14:55:55.33
>>629
e↑は3行3列の単位ベクトルと捉えるのが一般的だと思います.

が,質問の場合は3行1列の基底列ベクトル(実際にはベクトルとは呼べないが!),
つまり基底ベクトルex↑,ex↑,ez↑の列ベクトルe↑=[ex↑ ex↑ ez↑]^{T}と考えています.
631132人目の素数さん:2012/05/13(日) 14:56:41.52
>>514-515
詳しい説明ありがとうございます
あの後体調を崩してしまい、寝込んでいました
返事が遅れてしまってすいません

さて、直線Lが三角形OABの二辺OA、ABと交点をもつことが分かったため、その後は、
まず二つの交点の座標を求め、その二点を結んでつくられる線分の長さを求め、
点Aから直線Lまでの距離を求め、三角形OABの面積の半分になることを利用してmの二次方程式をつくる方針をとったんですが、うまくいきません…
632132人目の素数さん:2012/05/13(日) 14:59:05.93
誤解を招くので訂正

>e↑は3行3列の単位ベクトルと捉えるのが一般的だと思います.
単位ベクトルではなく,単位行列です
633132人目の素数さん:2012/05/13(日) 15:06:12.77
>>631
どこで詰まったのかわからないから、実際の立式や交点がどうなったのか書いてくれなきゃ

ところで514や515にあるような考え方でmが負の値になるって事は気付いてる?
634132人目の素数さん:2012/05/13(日) 15:20:36.58
>>630

> が,質問の場合は3行1列の基底列ベクトル(実際にはベクトルとは呼べないが!),
> つまり基底ベクトルex↑,ex↑,ez↑の列ベクトルe↑=[ex↑ ex↑ ez↑]^{T}と考えています.
ならば、
[ex↑ ey↑ ez↑]^{T}≠{[ex↑ ey↑ ez↑]^{T}}^{T}
は明らかだね。前者は3×1、後者は1×3なのだから。
一般に形だけの話ならn行n列の行列Aに対して A=A^T を考えるのは無意味ではないが、
行列Aが表す幾何的な機能に対し、A^Tが表す機能は何かということになると、それほど単純な話ではなくなる。

一次形式とか双対空間、ということをキーワードにして考えて見るとよいと思う。
(受験の数学として直接には登場しないけれど、知っていれば、問題の出所がハハ〜ン、と分かって
解答作りが楽になる可能性はある。)
635132人目の素数さん:2012/05/13(日) 15:29:47.92
>ならば、
>[ex↑ ey↑ ez↑]^{T}≠{[ex↑ ey↑ ez↑]^{T}}^{T}
>は明らかだね。前者は3×1、後者は1×3なのだから。

やはり,そうですか.
代数ベクトルの場合は向きを持たないので,
[ax ay az]^{T}={[ax ay az]^{T}}^{T}
が成り立ちますよね.(ただしax,ay,azはスカラー)

基底列ベクトル[ex↑ ey↑ ez↑]^{T}を考えたとき,これを転置すると一体どういう状態になるのか想像できないです
転置前はただ3本の基底ベクトルがあるだけのはずなのですが・・
636甜菜6:2012/05/13(日) 15:40:16.09
>>586
>最適化ですので
何をどう最適化したいのか、エスパーしようにも
どうしたものやら。
地盤?それとも構造物?

真面目に変位量の時間履歴を取りたいのなら
愚直に解くしかないんじゃないの?
それをどうやったら他の手法で置き換えられるのか、
それはむしろ君自身が研究テーマとして取り組むべき
問題なんじゃないかと思うのだが。

いま京の研究公募してるから、それに応募してみたら?
今の手法でも結構頑張れちゃうかもよ?
637132人目の素数さん:2012/05/13(日) 15:43:34.08
高校生のための・・・?
638132人目の素数さん:2012/05/13(日) 15:52:02.50
白玉が4個、黒玉が3個、赤玉が1個あるとする。
これらの玉にひもを通し、輪をつくる方法は何通りあるか

という問題で、ある軸について対称で裏返しても同じ順になるものは
3!/2!1!=3通り
と書いてあるのですがこれは一体何を計算しているのでしょうか
よろしくお願いします。
639132人目の素数さん:2012/05/13(日) 16:01:45.29
>>638
>という問題で、ある軸について対称で裏返しても同じ順になるものは

赤、黒が奇数個あるから対称軸上に赤と黒がくる。
残りの白4個と黒2個を対称になる様に並べるには
対称軸上の赤と黒の間の片側三個に
白2個と黒1個を並べればいいので

>3!/2!1!=3通り
640132人目の素数さん:2012/05/13(日) 16:03:19.65
>>639
ありがとうございました!
よくわかりまいした^^
641132人目の素数さん:2012/05/13(日) 16:05:22.85
裏返したのと裏返してないのでかぶるので÷2する必要がある
ただし
 黒
黒 黒
白 白
白 白
 赤
のように裏返しても同じものは除いて、÷2しなければいけない。
642132人目の素数さん:2012/05/13(日) 16:06:59.64
2X^3−X^2−5X+3の因数分解って
どうやってやればいいのですか?
よろしくお願いします。
643132人目の素数さん:2012/05/13(日) 16:22:17.07
標準偏差について質問です。
上式を使って下ニ式が等価であることを証明したいのですがやり方がわかりません。
http://iup.2ch-library.com/i/i0634115-1336893702.jpg
644132人目の素数さん:2012/05/13(日) 16:25:14.25
645132人目の素数さん:2012/05/13(日) 16:29:00.80
>643
普通に展開しろ
Σμ^2 = Nμ^2
μ(Σxi) = μ(Nμ) = Nμ^2
646甜菜:2012/05/13(日) 16:58:16.18
素人の方が解はいっぱいもってるな

最近の状況からするとD論もちょろそう
647132人目の素数さん:2012/05/13(日) 17:10:06.54
高校になったらこんな難しいことやるの?
648132人目の素数さん:2012/05/13(日) 17:22:12.87
>>642
定数項があるからx=0は解の可能性から除外される
最高次係数が2、定数項が3なので
x=±1/2、±1、±3/2、±3…あたりを中心に調べる
奇数次のf(x)=2x^3-5x、偶数次のg(x)=-x^2+3に分ける
f(-x)=-f(x)、g(-x)=g(x)に気をつけると少し計算が楽になるから

f(1/2)とg(1/2)を計算してふたつの和と差を調べる
f(1)とg(1)を計算してふたつの和と差を調べる
f(3/2)とg(3/2)を計算してふたつの和と差を調べる
f(3)とg(3)を計算してふたつの和と差を調べる

このあたりで0になる組み合わせが見つからなかったら
たいてい計算機の出番だが、この問題では解が見つかる
649643:2012/05/13(日) 18:01:16.09
>>645
Σμ^2 = Nμ^2
これの証明教えてください
650132人目の素数さん:2012/05/13(日) 18:03:47.33
>>635
> やはり,そうですか.
> 代数ベクトルの場合は向きを持たないので,
> [ax ay az]^{T}={[ax ay az]^{T}}^{T}
> が成り立ちますよね.(ただしax,ay,azはスカラー)
成り立たない。
一方は縦、もう一方は横。
651132人目の素数さん:2012/05/13(日) 18:12:42.39
>>648
ありがとうございました
652132人目の素数さん:2012/05/13(日) 18:25:16.20
とある点について、
両側極限の値と実際の値が異なるような関数って存在するんでしょうか?
653132人目の素数さん:2012/05/13(日) 18:30:20.93
関数は1つの値xに対し1つの値f(x)をとればいいだけなので
人工的に作ろうと思えばいくらでも作れる
654132人目の素数さん:2012/05/13(日) 19:13:34.79
>>652
デルタ関数とか
655132人目の素数さん:2012/05/13(日) 19:46:51.50
>>654
ちなみにとある関数がδ関数であることを示せって問題が与えられたら
・積分したら1である
・x=0で∞
・x→0の両側極限が∞
のどれを示せばいいんでしょうか?
全部?
656132人目の素数さん:2012/05/13(日) 19:48:12.62
>>652のような関数が存在するなら
関数の連続性、微分可能性を示すには両側極限が同じ値を取ることを示すだけでは足りないんでしょうか?
657132人目の素数さん:2012/05/13(日) 19:56:08.74
×示すだけでは足りない
○連続や微分の定義を新設した
658132人目の素数さん:2012/05/13(日) 19:57:10.13
>>635
通常の3次元ベクトル を a↑、b↑ とし
これらのベクトルの内積を (a↑,b↑)と書くことにする。
Aを3×3行列とするとき  (a,Ab)=(A^{T}a,b) が成り立つことを確認せよ。
659132人目の素数さん:2012/05/13(日) 20:17:26.29
教科書にはa>0,b>0のときa≦b⇔a^2≦b^2は証明されてるが、
途中から0≦a, 0≦b でもこの同値性は成り立つ。と書いてあるが証明はない。
aとbが同時に0にならない場合は、a+b>0 より 0≦b-a⇔0≦(b-a)(b+a)
で証明できるが、同時に0のときはこんな感じで一発に証明できますか?
a=0, b=0→a^2=0, b^2=0 逆に a^2=0, b^2=0→a=0, b=0
だから、aとbが同時に0になるとき, a=b⇔a^2=b^2
なら証明できますが、なんと言うかパッとできる方法ないかなあ?
660132人目の素数さん:2012/05/13(日) 20:29:36.06
a=b=0なんだからあきらか。
661甜菜7,8:2012/05/13(日) 20:43:01.19
>>586
時間が限られてるなら最適化の項目を減らしなさい。

どうしてもやる必要があるなら数年掛かろうがやりなさい。
662132人目の素数さん:2012/05/13(日) 21:05:00.03
>>654
デルタ関数は高校で通常習う意味での「関数」ではない
663132人目の素数さん:2012/05/13(日) 21:14:16.52
>>660
明らかでいいのかなあ..
664132人目の素数さん:2012/05/13(日) 21:38:23.64
>>663
同時に0の時なんだろ?
その時点でa=b=a^2=b^2=0だろ。
665132人目の素数さん:2012/05/13(日) 21:45:24.50
馬鹿には何を言っても無駄
666132人目の素数さん:2012/05/13(日) 22:15:34.44
今高1何ですが本気で悩んでます。
僕は勉強(特に数学)が完璧に納特しながらでないと先に進めないタチです。
中学の時は塾の先生にある程度は覚えた方が早いと言われつつもその先生が大変優しい方で、
とことん納得するまで質問させて頂きました。今でも感謝しているしであっていなかったら今頃どうなっているかぞっとします。
そのかいあって地域でいちばん(田舎ですが)の高校に進学できました。
しかし高校では授業のスピードがケタ違いでなかなかついていけません。あっという間に取り残されました。
そこで質問です。
ズバリ言いますと、このやり方は変えるべきでしょうか?ある程度納得出来なくても公式やパターンさえ覚えればテストではそこそこの点が取れると思います。

大学は恐らく文系に進むと思います。読みにくい文でごめんなさい動揺しておりますので・・・
先の事まで考えた先駆者としての回答お願い致します。
667132人目の素数さん:2012/05/13(日) 22:22:07.03
>>666
例えばどんなことに納得できてないんだ?
授業の進度は?
668132人目の素数さん:2012/05/13(日) 22:30:24.51
丸暗記を悪いとは言わないけど
しっかり理解してないとちょっと捻った問題にやられる可能性はある

基礎だけ拾って無難にセンター7割取るだけなら丸暗記でもいいかもね

そうじゃなくてしっかり納得して進みたいなら授業後に質問しに行くなり自習するなりすればいい
授業でやったことはメモ書き程度にまとめておいて後から清書用ノートにまとめ直せば理解も深まる
もちろんそれなりの手間はかかるけど


個人的には後者をすすめる
669132人目の素数さん:2012/05/13(日) 22:36:11.87
>>666
教科書読んでる?
人より頭が悪くて納得し難いって自覚があるなら、学校でやる範囲あらかじめ読んでおいて、授業で分からない所質問してる?
少なくともどこが納得できなくて、注意して説明を聞くべき点なのか程度は事前に明らかにしている?

まぁそこまでやっても田舎の公立高校の先生なんざ、自分自身も分かってないようなクズも多いから分からないかもしれないけどね。
670132人目の素数さん:2012/05/13(日) 22:46:08.13
特に数学って、学校でやる範囲の事にいちいち納得できる様に書いてあんのは数学だけだろ。
他の教科はどう納得してるんだよ(笑)
671132人目の素数さん:2012/05/13(日) 22:49:58.84
>>667
授業は「普通」の学校の1.5倍です。根っからの文系なので式で証明されても素直に頷けないというか・・・
たとえばかなり前の話になりますが-1×-1=+1というのにかなり悩みました。覚えればすぐなんですが・・・
>>668
清書ノートと普通ノートいいかもしれませんやって見ます。
>>669
先生は馬鹿ではないと思いますがあまり説明する気がないようです・・・

672132人目の素数さん:2012/05/13(日) 22:50:04.75
加法定理の導き方で質問です
教科書には
単位円で、動径OP,OQの表す角をα,βとすると
P(cosα,sinα),Q(cosβ,sinβ)より
PQ^2=(cosα-cosβ)^2+(sinα-sinβ)^2

と書いてあるのですがよく分かりません PQ^2がなぜこのような式になったのでしょうか
丸暗記すればいい話だとは思いますが、誰か詳しくお願いします
673132人目の素数さん:2012/05/13(日) 22:51:56.86
初歩的な質問ですいません
次の関数を微分せよ
@√X
AX^2/(X+1)
B1/(2X+3)
この微分の仕方がほんとに分かりません・・・
674132人目の素数さん:2012/05/13(日) 22:52:20.36
>>672
ピタゴラスの定理
675132人目の素数さん:2012/05/13(日) 22:53:09.07
>>672
間違っても丸暗記するような話ではないぞ
676132人目の素数さん:2012/05/13(日) 22:58:25.89
>>671

>授業は「普通」の学校の1.5倍です。根っからの文系なので式で証明されても素直に頷けないというか・・・

意味が分からない。何処に疑問点があるんだよ。AならばB、BならばC、CならばD
よってAならばDって作業やってるだけじゃん。式だと信用できないとか(^_^;)

>先生は馬鹿ではないと思いますがあまり説明する気がないようです・・・

そっちより教科書しっかり読んでるのか知りたい。
677132人目の素数さん:2012/05/13(日) 23:00:30.45
>>671
>式で証明されても素直に頷けない
式だけだとイメージが湧かないってこと?
関数は常にグラフを描く(思い浮かべる)癖をつけとくといいぞ
678132人目の素数さん:2012/05/13(日) 23:00:53.15
>>671
意味づけが出来ないとダメな気がするってことか?
そういうことなら、それはスルーしろ。
そうやるとうまくいくからそうやってるだけって意味しかないものもいっぱいある。
679132人目の素数さん:2012/05/13(日) 23:02:21.72
>>673
教科書よめ
>@√X
これはx^(1/2)と同じこと、これ微分してみ
>AX^2/(X+1)
これは商の微分
>B1/(2X+3)
これは(2x+3)^(1/2)と同じこと
680132人目の素数さん:2012/05/13(日) 23:04:21.93
>>679
大嘘書いた
>>B1/(2X+3)
>これは(2x+3)^(1/2)と同じこと
ちげぇ
(2x+3)^(-1)と同じことだった
681132人目の素数さん:2012/05/13(日) 23:08:13.83
皆さんありがとうございます
特に>>677・678さんグラフで思い浮かべるのはもう点でした(突っ込まれると思いますが)
頭デッカチになっていたようです
もうすこし気楽?に頑張ります
あと教科書は何回も読んでます
682132人目の素数さん:2012/05/13(日) 23:15:39.56
>>672
>P(cosα,sinα),Q(cosβ,sinβ)より
↑は納得してるか?

>PQ^2=(cosα-cosβ)^2+(sinα-sinβ)^2
三平方の定理

ひょっとして教科書は啓林館(p116)か?

こちらからの質問
お前さんの学校は教科書の最初から始めてもう「三角関数」に行ったのか?
683132人目の素数さん:2012/05/13(日) 23:19:23.94
>>681

>特に>>677・678さんグラフで思い浮かべるのはもう点でした(突っ込まれると思いますが)

>あと教科書は何回も読んでます

ホントかよ、二次関数とか教科書にうぜぇぐらいグラフ書いてあるだろ。分かってないって自覚あるのに字面だけ見てグラフガン無視か?
684132人目の素数さん:2012/05/13(日) 23:49:58.87
外積や内積の線形変換って線形変換の外積や内積になりますか?
式で書くと、
 f(a・b)=f(a)・f(b) (a, bはベクトル)
などは成り立ちますか?
685132人目の素数さん:2012/05/13(日) 23:53:14.96
>>683
質問てのは応用問題ですね
中学の教科書なんてリアルに10回ぐらいしかまともにみてないです
686132人目の素数さん:2012/05/13(日) 23:53:51.69
間違えました、内積じゃなくて、外積のみについてお願いします。
内積じゃ意味不明ですね…
687132人目の素数さん:2012/05/14(月) 01:02:06.36
(Ma+v)x(Mb+v)=MaxMb
M(axb)+v=Mc+v
688132人目の素数さん:2012/05/14(月) 01:03:39.99
ならん
689132人目の素数さん:2012/05/14(月) 05:51:22.41
>>664
a^2=0→a=0 は実数なら常に 0≦a^2 等号はa=0のときのみ成り立つ
ってやつの等号の性質使っているんですよね。
690132人目の素数さん:2012/05/14(月) 06:13:11.84
使ってないよ
691132人目の素数さん:2012/05/14(月) 07:54:52.77
xyz空間において、3点(0,1,-3),(2,3,-1), (3,-1,-1)を通る円の中心の座標を求めよ。

これはどう考えればいいですか。
xy平面で3点を通る円だったらx^2+y^2+ax+by+c=0とおいて連立方程式で解けるのですが・・・
692132人目の素数さん:2012/05/14(月) 07:57:06.56
>>689
なんだよ、それ。
それじゃあ、a^2=0→a=0が成り立つのはa^2=0→a=0だからって言ってるのと同じだろ。
693132人目の素数さん:2012/05/14(月) 07:58:52.39
>>691
それら3点と距離が同じで、同じ平面上にある。
694132人目の素数さん:2012/05/14(月) 11:22:07.38
P_(n)=2^n・sin(π/2^n)
q_(n)=2^n・tan(π/2^n) のとき
q_(n+1)=2p_(n)q_(n)/(p_(n)+q_(n))
p_(n+1)=√(p_(n)q_(n+1)) を示せ


それぞれ半角の公式と倍角の公式でできるそうなのですが、式変形が間違ってるのか解けませんでした
よろしくお願いします
695132人目の素数さん:2012/05/14(月) 12:00:01.72
>>689
横からすみません
実数以外(複素数?)だとa^2=0→a=0とならないこともあるんですか?
ちょっと気になったもので…
696132人目の素数さん:2012/05/14(月) 12:20:29.80
>>694
やった式変形かきなよ。
697132人目の素数さん:2012/05/14(月) 12:31:38.96
>>685
中学の教科書じゃなくて、高校の教科書の話をしている。中学の範囲は一応大丈夫なんでしょ?
たぶん、大丈夫じゃなさそうだけど、自分が大丈夫だと思っているならわざわざ中学の教科書まで読み込む必要ないよ。

応用問題ったって、モノによるからなぁ
数と式なんかは、そうやると上手くいくって経験則が多いしほぼ丸暗記だけど
二次関数なんか、ほぼ全て理詰めでわかるような事。
後、文系でもそれなりの学校行きたいなら数学結構出来ないとまずい上に、理系との範囲の違いは3Cだけど、3Cが一番丸暗記に近い分野。私文なら数学なんかいらんけどね。
698132人目の素数さん:2012/05/14(月) 12:52:13.19
>私文なら数学なんかいらんけどね。
バカの拡大再生産か
699132人目の素数さん:2012/05/14(月) 13:27:37.81
>>692
実数の性質で、a<0, a=0, a>0のいずれかひとつのみ成り立つ。
a<0のとき、両辺に負の数aをかけて, a^2>0
a=0のとき、両辺にaをかけて(いいのかな?)a^2=0
a>0のとき、両辺に正の数aをかけて, a^2>0
転換法よりa^2=0→a=0
ということを言いたかったのですが...
700132人目の素数さん:2012/05/14(月) 13:36:54.50
>a=0のとき、両辺にaをかけて(いいのかな?)a^2=0
それで、この式単独なら、a^2=0のとき、両辺をaで割ってa=0
なんてことしてはいけませんよね。そのあたりの証明は
転換法しかないのでしょうか??

>>695
複素数でもa≠0→a^2≠0…(1), a=0→a^2=0から
この場合は背理法でa^2=0→a=0は出ますね。
ただ、(1)はどう証明する(または定義から導くのだろうか??)
701132人目の素数さん:2012/05/14(月) 13:40:59.84
a>0→a^2>0 はどう証明するの?
702132人目の素数さん:2012/05/14(月) 13:40:59.92
>>699
a^2=0→a=0 を背理法で示す
a≠0と仮定すると、aの逆数a^(-1)が存在し
a=a^(-1)×a^2=a^(-1)×0=0 となり、仮定に反する

この証明からわかるように、実数の大小関係や符号といった概念とは無関係に成り立つ事実です
703132人目の素数さん:2012/05/14(月) 13:43:48.49
>>695
行列だとある。
704132人目の素数さん:2012/05/14(月) 13:47:40.61
>>633
一旦僕が計算の途中で詰まってしまったことは忘れていただいて、
自らが解かれるならどのような手順でおこなうかを教えていただけませんか?
705132人目の素数さん:2012/05/14(月) 14:04:16.95
>>700
それも背理法でいいだろ。
706132人目の素数さん:2012/05/14(月) 14:18:03.02
>>704いやです
707132人目の素数さん:2012/05/14(月) 14:28:55.96
-90°<x<90°の範囲でtanx=√2cosxを満たすxをもとめよ。
答えがx=45°と書いているのですが-45°は違うのですか?sinx=1/√2としてもとめました。
708132人目の素数さん:2012/05/14(月) 14:32:45.35
>>707
自己解決しました。
709132人目の素数さん:2012/05/14(月) 14:38:40.05
>>707
cosの式にしてたら式変形の過程で-45度も、なり立っちゃう様に感じるだろうけどねぇ。sinで出してるみたいだから単なる勘違いだと思うけど、いい機会だからcosの式にしても解いてみてごらん。
710132人目の素数さん:2012/05/14(月) 14:50:39.43
>>709
値なので単位円の範囲から45°としかなり得ませんね。

0≦θ≦180°の範囲で√3cosθ=sinθのθの値は?という問題もわかりません。
答えが60°となっているのですが120°は違うのですか?
3cos^2θ=sin^2θ
3cos^2θ+cos^2θ=1
cos^2θ=1/4
cosθ=±1/2と考えたのですが。
別の方法で、√3=sinθ/cosθとすれば、答えは60°のみとなりますが、前者は何が間違っているのでしょうか。
711132人目の素数さん:2012/05/14(月) 14:59:51.05
>>710

>0≦θ≦180°の範囲で√3cosθ=sinθのθの値は?という問題もわかりません。
>答えが60°となっているのですが120°は違うのですか?

そうそう、こういう事。たぶんさっきの問題もcosになおして計算しようとすると同じような事になったはず。


>3cos^2θ=sin^2θ

ここでやらかしてる。
三角関数に限らず、絶対に覚えておいた方がいい事として、二乗をする時には気を付けること。
大雑把に言えば二乗をすると、符号の情報が消し飛ぶんだ。

√3cosθ=sinθの段階だと、cosとsinが同符号なのが情報として含まれているんだけど
二乗して3cos^2θ=sin^2θにするとcosとsinが異符号でも成り立っちゃうの
二乗は同値変形(⇔)じゃないから気をつけろ!ともよく言われる。
712132人目の素数さん:2012/05/14(月) 15:00:43.79
>>710
3cos^2θ=sin^2θとしたところが同値変形じゃないから、元で成り立つかどうかを確かめる必要がある。
713132人目の素数さん:2012/05/14(月) 15:26:19.81
>>711
>>712
実際に-1/2を代入すれば違うのは明らかですね。
あくまでも条件√3cosθ=sinθを満たすことを考え、cosとsinどちらも正なので答えは第一象限のどこかの角度っていうことがわかるって感じですかね?
714132人目の素数さん:2012/05/14(月) 15:34:32.63
>>713
>実際に-1/2を代入すれば違うのは明らかですね。

んーこの問題の場合は最後に最初にもどって代入してみるその考えでも問題ないけど、同値変形になれてかないと他でヤバイからそれだけじゃあとあと問題かな

条件√3cosθ=sinθは
√3cosθとsinθの正負が一致しますよ!っていう条件が隠れている式なの
ところが二乗しちゃうと
条件3cos^2θ=sin^2θには
3cos^2θとsin^2θが正ですよっていう情報になって、情報量が減っちゃってるの。

基本的に、なるべく情報量が減らない変形(これを同値変形という)を心がけて
情報量が減る変形をする時には、あっ自分今情報量が減る変形したぞ‼って自覚して式を扱う必要がある。
715132人目の素数さん:2012/05/14(月) 15:54:09.28
>>699
そもそも考えるべき問題が違う。
問題は「a=b=0のとき、a=b⇔a^2=b^2はどう示すのか」だろ?
この場合、a=b=0とすでに仮定されてるんだからa=bもa^2=b^2も真
よってa=b⇔a^2=b^2も真
というだけ。

a^2=0⇒a=0の証明は、厳密に言えば、実数の定義を知らなければならない。
もしかしたら大小関係を定義するためにa^2=0⇒a=0が必要かもしれないし、
>>702だってもしかしたら「0でない元に逆元が存在する」を証明するためにa^2=0⇒a=0が必要かもしれない。
(実際は両方ともそんなことは無いからどちらも正しい)
こういうのは高校数学じゃ限界がある。とりあえず今は>>699>>702の証明で納得しとくのがいいと思う。
716132人目の素数さん:2012/05/14(月) 16:07:46.66
教科書の問題です

半径10の円に内接する正五角形の1辺の長さを求めよ。
また、円の中心Oから正五角形の1辺に下した垂線の長さを求めよ。

解説を見ると、円に内接する五角形の中にできる三角形の角度が
いきなり∠AOH=36度と書かれているのですが、どのように求めるのですか
717132人目の素数さん:2012/05/14(月) 16:09:37.23
>>716
なぜ突然AとかHが出てくる?
718132人目の素数さん:2012/05/14(月) 16:11:16.73
>>713
必要条件、十分条件、必要十分条件(同値)に気をつけろってこと

x=-1 ・・・@

両辺二乗すると

x^2=1 ・・・A

気をつけるのは、@ならばAは真だが、Aならば@は偽である点(@⇒A)。
Aを解いても(x=±1)それが@を満たすかは分からない。
Aを解いた後で、実際に@に代入して、解を吟味する必要がある。

両辺に同じ数字を足したり、掛けたりした場合は
"同値"な変形(@⇔A)なので、代入して吟味する必要はない。
719132人目の素数さん:2012/05/14(月) 16:14:40.22
>>716
正五角形だからAOBが360/5で72度
円の中心から弦に下ろす垂線は垂直二等分線で、出来る二つの三角形AOHとBOHが合同だからAOBは72の半分の36度
720132人目の素数さん:2012/05/14(月) 16:42:52.50
みなさまありがとうございます。

>>701
>>699

>>702
ありがとうございます。教科書の記述の流れだと
実数の条件を利用していたので、それで何とかならないか、
なおかつ背理法とか転換法とか使わなくてどうにかならないか、
y=x^2のグラフでごまかすのはやだ。
と思っていました。

>>715
a=bとa^2=b^2が両方真だとしても、
「a^2=b^2→a=b」とは別命題なので何かそれでいい感じがしないんです。

>>718
その理屈の理論的背景が結局 a=b⇔a^2=b^2 に行き着くので
何とかお助けくださいまし。
721132人目の素数さん:2012/05/14(月) 16:47:52.73
>>720
大学行ってから好きなだけやってくれよ。
722132人目の素数さん:2012/05/14(月) 16:50:40.44
>>720
> 何かそれでいい感じがしないんです。
俺には「a=b=0のとき」って言ってんのに「a=bならば」って考えるほうが不思議だわ。
723132人目の素数さん:2012/05/14(月) 17:09:03.08
695です
横からなのにありがとうございました

>>700
感覚的にはやはり複素数を二乗しても、虚部が残りそうで0にはならない感じはしてました
でも、証明はちょっとわかりませんでした…

>>703
あー行列だと確かに!
ちょっと計算してみたら、((0,0),(1,0))あたりがそうですね
AB=BAが必ずしも成り立たないことなどもあるので、やはりちょっと特殊な感じがしますね
724132人目の素数さん:2012/05/14(月) 17:11:39.63
円周率をπとする。
このとき、π<3.17を示せ。
725132人目の素数さん:2012/05/14(月) 17:35:29.42
三角形の辺の比についてです。http://beebee2see.appspot.com/i/azuYmKasBgw.jpg
画像の三角形OABがあって、ABの中点がP,CDの中点がQのとき、O,Q,Rは一直線上にあるのでしょうか?
証明の仕方がわかりません(_ _)
726132人目の素数さん:2012/05/14(月) 17:37:00.35
>>720
>a=bとa^2=b^2が両方真だとしても、
>「a^2=b^2→a=b」とは別命題なので何かそれでいい感じがしないんです。
数学(というか論理学)では、A⇒Bという命題は、Bが真ならAがどうなっていようと真である、と「約束」する。
例えば「どんな三角形も内角の和が180゚」⇔「どんな正の実数a,bに対してもa+b≧2√(ab)」も真。
まあ違和感あるかもしれんが、そんなもんだ。
727132人目の素数さん:2012/05/14(月) 17:40:39.16
>>725
CDは自由にうごいちゃうの?CとDに対する条件が無きゃなりたたないよ
728132人目の素数さん:2012/05/14(月) 17:44:16.55
>>725
> O,Q,Rは一直線上に
図にRはないが
729132人目の素数さん:2012/05/14(月) 17:57:17.76
>>728
図にはあるぞ。全然定義されてないけど。
730132人目の素数さん:2012/05/14(月) 18:00:12.62
お前目大丈夫か?
731132人目の素数さん:2012/05/14(月) 18:56:25.85
>>727
>>728
>>729
大事なところ間違えてました。PというのをRに置き換えてください。OQRが一直線上にある証明です。
732132人目の素数さん:2012/05/14(月) 19:02:04.66
>>731
だから、条件不足。そんなの成り立たねえよ。
733132人目の素数さん:2012/05/14(月) 19:12:12.31
>>731
CがBのすぐ近く、DがOのすぐ近くにそれぞれあったら、QはBCの中点のすぐ近く
になる。このときOQRが一直線上にあるわけがない。
734132人目の素数さん:2012/05/14(月) 19:20:21.44
領域図示の問題なのかな?
735132人目の素数さん:2012/05/14(月) 19:26:30.76
>>733訂正
× QはBCの中点のすぐ近く
○ QはBOの中点のすぐ近く
736132人目の素数さん:2012/05/14(月) 19:31:21.28
xが2つの値をとる時
2x+a≦1 の場合のaのとる範囲は何か。

お願いします。

737132人目の素数さん:2012/05/14(月) 19:36:35.77
>>732
>>733
確かにそうですね。
よくわからないので、問題を載せます(_ _)

平面に四角形ABCDがあり、頂点Cは
AC↑=(4/5)AB↑+(3/5)AD↑を満たすものとする。
(1)直線ABとDCの交点をE,直線ADとBCの交点をFとする。AE↑とAF↑をAB↑,AD↑であらわせ。
(2)線分BDの中点をQ,線分EFの中点をRとするとき、QR↑をAB↑,AD↑を用いてあらわせ。
(3)線分ACの中点をPとするとき、3点P,Q,Rは同一直線上にあることを証明せよ。
(2)なんですが、QR↑を求めるのに解答ではAR↑-AQ↑としてます。これはA,Q,Rが同一直線上にあるということですよね?
この問題の場合なぜ同一直線上にあるとわかるのか教えてください。
738132人目の素数さん:2012/05/14(月) 19:53:08.42
想像の斜め上を超える問題の上に、
>>737
>(2)なんですが、QR↑を求めるのに解答ではAR↑-AQ↑としてます。これはA,Q,Rが同一直線上にあるということですよね?

のキチガイ発言っぷりにワロタ
悪い事は言わんから教科書のベクトルの最初から読み直した方がいいよキミ。マジで
739132人目の素数さん:2012/05/14(月) 20:09:44.97
>>726
ありがとうございます。確かに、a=b=1のときも
a=b⇔a^2=b^2ではあるんですよね。違和感ありまくりですけどね。
納得しました。
740132人目の素数さん:2012/05/14(月) 20:19:02.54
>>737
その問題とは別に、
△ABCにAB↑、AC↑、BC↑を書いてBC↑をAB↑とAC↑で表してみて
741132人目の素数さん:2012/05/14(月) 20:32:38.97


>>740さんに従えまず。
742132人目の素数さん:2012/05/14(月) 20:40:01.42
というか、それ分からなくて今までベクトルのどんな問題を解いてきたのか逆に興味ある。
743132人目の素数さん:2012/05/14(月) 20:41:46.71
>>740
AC-AB
>>741
アクセス規制かかってるからレスにじかんかかる
744132人目の素数さん:2012/05/14(月) 20:44:01.70
a(b^2-c^2)+b(c^2-a^2)+c(a^2-b^2)が解けません。誰かお助けを
745132人目の素数さん:2012/05/14(月) 20:45:23.97
746132人目の素数さん:2012/05/14(月) 21:00:05.74
>>745
それ見たんですけど、この(b-c)で括る辺りがイマイチ納得できないんです
= a(b^2−c^2)-bc(b−c)-a^2(b-c)
= -(b-c){a^2 - (b+c) + bc)}
747132人目の素数さん:2012/05/14(月) 21:04:49.68
タイプミスだろうね
×-(b-c){a^2 - (b+c) + bc)}
○-(b-c){a^2 - (b+c)a + bc)}
748132人目の素数さん:2012/05/14(月) 21:14:30.73
>>736
他にも条件あっただろ
そのままだとすべての実数になるぞ
749132人目の素数さん:2012/05/14(月) 21:29:42.17
>>747
今わかりました。ありがとうございます
750132人目の素数さん:2012/05/14(月) 21:47:50.98
>>743
>>740がBC↑=AC↑-AB↑と分かるなら
> QR↑を求めるのに解答ではAR↑-AQ↑としてます。これはA,Q,Rが同一直線上にあるということですよね?
こういう発想は出てこないはずなんだが。
751132人目の素数さん:2012/05/14(月) 21:53:52.00
>>737
AC-AB
752132人目の素数さん:2012/05/14(月) 22:02:18.99
多項式P(x)を(x-1)^2で割ると余りが4x-5,x+2で割ると余りが-4である。このとき、P(x)を(x-1)^2(x+2)で割ったときの余りを求めよ、という問題がわかりません。
詳しい解答お願いします
753132人目の素数さん:2012/05/14(月) 22:07:04.39
>>750
AQ↑=(1/2)AB↑+(1/2)AD↑
QR↑=2AB↑+3AD↑
AB↑とAD↑の係数を足して1になるのでBD上にQがあるといのはわかります。
QRの場合係数の和は5になるからQR↑=5AQ↑ってことでしょうか(T T)
AQRは一直線上になりますよね...?
754132人目の素数さん:2012/05/14(月) 22:09:19.56
>>753
AR↑=AB↑+(3/2)AD↑の間違いです。
AR↑=2AQ↑?
755132人目の素数さん:2012/05/14(月) 22:31:21.76
>>752
3次式で割るのだから余りは2次以下の次数の多項式になるので
それを ax^2+bx+c とすると
P(x)=Q(x)(x-1)^2(x+2)+ax^2+bx+c
とおける。
そこで、P(x)を(x-1)^2で割った余り、x+2 で割った余りは
ax^2+bx+c を (x-1)^2で割った余り、x+2 で割った余りになるから、
a,b,cの値が求まる。

まず、ax^2+bx+c を (x-1)^2、x+2 で割って余りを求めてみな。

洒落た解き方は、それが出来てからだ。
756132人目の素数さん:2012/05/14(月) 22:41:50.01
>>755
P(x)を(x-1)^2で割った余り、x+2 で割った余りはax^2+bx+c を (x-1)^2で割った余り、x+2 で割った余りになるんですか?
757132人目の素数さん:2012/05/14(月) 22:43:28.80
>>755
なんでP(x)を(x-1)^2で割った余り、x+2 で割った余りは
ax^2+bx+c を (x-1)^2で割った余り、x+2 で割った余りになるんですか?
758132人目の素数さん:2012/05/14(月) 22:52:09.82
>>757
P(x)=Q(x)(x-1)^2(x+2)+ax^2+bx+c
759132人目の素数さん:2012/05/14(月) 22:53:01.99
>>753
> AQRは一直線上に
ならない
760132人目の素数さん:2012/05/14(月) 22:54:57.09
>>753
冗談抜きでベクトル全然分かって無いから最初からやれって(笑)
761132人目の素数さん:2012/05/14(月) 23:01:46.48
>>758
P(x)/(x-1)^2=Q(x)(x+2)+ax^2+bx+c/(x-1)^2
になるんじゃないですか?
762132人目の素数さん:2012/05/14(月) 23:03:16.77
多項式の割り算の定理くらい、証明付きで教科書に載せればいいのにな
763132人目の素数さん:2012/05/14(月) 23:03:40.65
何でこのAA最近いっぱいあるの?
数学板って専門的な話で盛り上がるんだし、荒らさないでほしいわ。
何か意図でもあるんかな?数学板過疎らしたいのか?
こういうAAがあると荒らされてるスレって認定されるんだが。
764132人目の素数さん:2012/05/14(月) 23:04:55.41
>>761
523を10^2で割ったら余りはいくつだ?
523=500+23=5×10^2+23だから 
523/10^2=5+23/10^2か?
765132人目の素数さん:2012/05/14(月) 23:18:53.96
只の数学者なんちゃらとは付き合わないわ
みたいなAA

うざすぎるんだが.....
このAAがあるスレは荒らされてるんだなって
印象が植え付けられる。
まじで嫌なんだが。
766132人目の素数さん:2012/05/14(月) 23:22:34.40
lim[x→∞]cosx/xを求める問題で

[振動するものがあるのではさみうちの原理・・・@]を使って
いくのですが
0≦|cosx|≦1としますよね
そして、|1/x|をかけて...といくわけですが、[そもそも場合わけする理由が不等式の両辺に文字をかけるので、その正負で場合わけが生じるから面倒...って理由ででした・・・A]これは、今xが∞の近くでの話なので必ずx>0ではないのでしょうか、
また、@,Aの考え方はあってますか?
767132人目の素数さん:2012/05/14(月) 23:26:43.27
>>650
成り立ちます.代数ベクトルの話です.教科書に載っています.

>>658
基底列行列の内積e↑=[ex↑ ey↑ ez↑]を考えると,内積は

e↑^{T}・e↑=3

      1 0 0
e↑・e↑^{T} = 0 1 0
   0 0 1

となります.
このときe↑^{T}はデカルト座標系でどのように表現できると考えられますか?

私が望むのはただ一点だけで,e↑^{T}をデカルト座標系で表してほしいです.
1000字の言葉で説明されるより,一発で理解できるので.
768132人目の素数さん:2012/05/14(月) 23:32:14.52
>>767
^{T} は転置の記号じゃなかったのか?

>>767
> >>650
> 成り立ちます.代数ベクトルの話です.教科書に載っています.

[ax ay az]={[ax ay az]^{T}}^{T}
なら成り立つけどな。

ただの記号だよ。
769132人目の素数さん:2012/05/14(月) 23:36:51.21
>>767
>基底列行列の内積e↑=[ex↑ ey↑ ez↑]

意味不明
770132人目の素数さん:2012/05/14(月) 23:43:37.74
>>768
転置の記号です.代数ベクトルなので縦でも横でも同じ.
なぜなら代数ベクトルは大きさだけで方向を持たないから.

>[ax ay az]={[ax ay az]^{T}}^{T}
>なら成り立つけどな。
これは2回転置して元に戻ってるだけですね

>ただの記号だよ。
では,e↑^{T}・e↑≠e↑・e↑^{T} になる理由を述べてください.


>>769
基底ベクトルは分かる?基底ベクトルx,y,z方向の列ベクトルです.
771132人目の素数さん:2012/05/14(月) 23:52:00.83
>>770
「代数ベクトル」の定義を聞かせてもらおうか。
772132人目の素数さん:2012/05/14(月) 23:56:52.12
数ベクトルは分かる?
773132人目の素数さん:2012/05/14(月) 23:58:29.75
>>766
お願いします
774132人目の素数さん:2012/05/15(火) 00:01:04.55
お湯400mlって重さに直すと何gですか?
775132人目の素数さん:2012/05/15(火) 00:01:44.80
776132人目の素数さん:2012/05/15(火) 00:02:05.56
n≦x≦n+1(nは自然数)が(1+1/(n+1))^n≦(1+1/x)^x≦(1+1/n)^(n+1)にどうしたら、変形できるのですか?
教科書にはこの過程が書いていないのでわかりません、教えてください
777132人目の素数さん:2012/05/15(火) 00:02:39.98
>>772
どうやらベクトル空間を自分勝手に解釈しているようだ。
778132人目の素数さん:2012/05/15(火) 00:07:37.78
>>776
(1+1/x)^n
(1+1/x)^(n+1)
(1+1/n)^x
(1+1/(n+1))^x
あたりと片っ端から大小関係を比べてみる
779132人目の素数さん:2012/05/15(火) 00:08:00.28
>>777

教科書に載っていることについて文句言われても困ります
780132人目の素数さん:2012/05/15(火) 00:28:46.62
limx→-∞ cos2x/xについて教えてください
781132人目の素数さん:2012/05/15(火) 00:38:55.71
>>779
何も難しいことは聞いていない。
「代数ベクトル」はその教科書でどう定義されているのかを聞きたいだけ。
縦ベクトル、横ベクトルに違いはない、と言っているからには、
ベクトル空間の同型を超越したなにか不思議な「同等」の定義があるのだろうから。
782132人目の素数さん:2012/05/15(火) 00:48:39.09
>>770
> >>768
> 転置の記号です.代数ベクトルなので縦でも横でも同じ.

なら
[ax ay az]=[ax ay az]^{T}
で終わりだな。
要するに、転置に意味はないということになるな。
はて?
783132人目の素数さん:2012/05/15(火) 00:57:07.06
cos(180°-θ)=-cosθ

θが90°より小さい場合は-cosθ だと言うのはしっくりくるのですが
θが90°≦θ≦180° だった場合も-cosθになるというのがよくわかりません
この場合の180°-θはcosθ だと思ったのですが、、、
誰か詳しくお願いします
784132人目の素数さん:2012/05/15(火) 01:10:37.80
>>783
θ=120° のとき cos(180°-θ) と cosθ はそれぞれいくつ?
785132人目の素数さん:2012/05/15(火) 01:11:01.48
>>783
実際に135度とか入れて見なよ
786132人目の素数さん:2012/05/15(火) 01:15:16.20
代数ベクトルって言葉は聞いた事ないなー
ググっても見つからないし
よっぽどレアな教科書だな
787132人目の素数さん:2012/05/15(火) 01:16:01.58
>>783
180°からθを時計回りに動かしてcosの値の変化を見てみろ
788132人目の素数さん:2012/05/15(火) 01:18:20.25
>>759
数直線上に左からA,B,Cって順番にあるとき、BC=AC-ABですよね?
三角形ABCのBCをACとABで表した場合、ABCは確かに一直線上にないですが、>>737の問題の場合、図でみると視覚的に一直線上にあるAQRからAR-AQと引いてるような気がします。http://beebee2see.appspot.com/i/azuYl5arBgw.jpg
789132人目の素数さん:2012/05/15(火) 01:21:07.82
>>786
代数ベクトル=数ベクトルです.書き方が悪かったかな.

数ベクトルは,大きさだけで向きをもたないベクトル.
間違って書いているサイトがあるので注意しておくと,数ベクトルに対して外積,内積は定義できない.向きを持たないから.
基底列ベクトルと組み合わせてはじめて内積・外積を定義できる.
790132人目の素数さん:2012/05/15(火) 01:21:30.45
将来は数学科を目差そうと思っています
数論幾何学というのは、何を研究する分野なのですか?
また、数論幾何学が21世紀の数学を開拓していくと
聞いたのですが、それはどういう意味ですか?
791132人目の素数さん:2012/05/15(火) 01:22:43.15
>>788
定規使ってその図に直線QRを書き込んでみ
792132人目の素数さん:2012/05/15(火) 01:34:31.33
>>789
そのように書いてある教科書のISBNを教えてくれ。
793132人目の素数さん:2012/05/15(火) 01:37:27.29
>>790
まずはそのあたりを自力で調べることが
数学者への第一歩な。
794132人目の素数さん:2012/05/15(火) 01:47:33.46
>>783
紙に半径1の円を書いて下の文章読んでってね。
-θが指す半径1の円上の位置と、θが指す半径1の円上の位置は、それぞれx軸に対称だよね。
つまりx軸にそって平面をパタンと折りたたむとお互いピッタリ一致する。
よって、θから原点に引いた線分とx軸の負の部分のなす角(この角をAとする)は等しい。
180°-θ=-θ+180°が指す半径1の円上の位置は、-θから原点をまっすぐ突っ切った先にある半径1の円上の点だよね。
この点から原点に引いた線分と、x軸の正の部分とのなす角は、Aと一致するね。(いわゆる対頂角の性質)
θが指す円上の位置からまっすぐx軸に線を引いてみよう。そのx座標はまさにcosθだね。
同様に、180°-θが指す円上の位置からもまっすぐx軸に線を引いてみよう。そのx座標はcos(180°-θ)だね。
この二つの三角形は合同で、しかもy軸にそって平面をパタンと折りたたむとお互いピッタリ一致するね。
これより、θが90°≦θ≦180°だった場合もcos(180°-θ) = -cosθとなるのが分かる。
795132人目の素数さん:2012/05/15(火) 01:51:23.07
あ、ごめん3行目訂正させてw
×よって、θから原点に引いた線分とx軸の負の部分のなす角(この角をAとする)は等しい。
○よって、θから原点に引いた線分とx軸の負の部分のなす角(この角をAとする)と、
 -θから原点に引いた線分とx軸の負の部分のなす角はともに等しい。
796132人目の素数さん:2012/05/15(火) 01:51:23.80
>>790
残念ながら、学問の話を聞く事が好きなのと、自分でやるのは似て非なるものなんだよ。
そんな事ここで聞いているような奴は未来が無いから、諦めて普通に就職する道考えた方がいいよ。
797132人目の素数さん:2012/05/15(火) 01:52:37.25
796が将来のライバルを蹴落とそうとしているようです
798132人目の素数さん:2012/05/15(火) 01:55:54.07
無邪気な子供への苛立ち
という感情ではないか
799132人目の素数さん:2012/05/15(火) 01:57:04.52
数学者を目差すとは言っていない件
800132人目の素数さん:2012/05/15(火) 02:01:41.35
>>799
それならなおさら数学科なんて行かない方が良い
ホンキでやんなきゃ
死ぬぞ
801132人目の素数さん:2012/05/15(火) 02:06:46.75
てか大学入ったら、高校までと違って誰かが分かり易く教えてくれるとかないから(笑)
せいぜい友達とか先輩ぐらい。
教える事に力入れてる日本の大学教授とか激レア。いないとは言わんけどね。
特に教養時代とか、嫌々やってる感隠さない奴のが多いぐらいだわ
802132人目の素数さん:2012/05/15(火) 02:15:39.01
で、誰も数論幾何の今日的な意義を
説明できへんのか
803132人目の素数さん:2012/05/15(火) 02:48:45.58
>>789
そういう数ベクトルは知らんな。
名古屋で教育実習に行った高校でも使ってなかったし、どの地方?
804132人目の素数さん:2012/05/15(火) 08:21:51.47
>>788
視覚なんぞ当てにならない。実際間違ってる。
問題の条件によっては一直線上に並んでいる場合もあり得るが、
一直線上にあることを示さずに(「一直線上にあるように見える」では示したことにならない)、
一直線上にあるものとして論証したら間違い。
805132人目の素数さん:2012/05/15(火) 08:25:40.20
>>791
それ教える側としては微妙
物理の問題なんかわざとおかしくしてたりするから図で調べるのは信用しない方がいい
806132人目の素数さん:2012/05/15(火) 08:42:00.46
>>804
図をみて一直線上にあるからQRをAR-AQとしてるかと勘違いしてました。
ベクトルの始点をAに変形してるだけなんすね。
807132人目の素数さん:2012/05/15(火) 08:48:30.63
>>806
基本的にベクトルがわかってないってみんな何度も言ってただろう。
△ABCを例に出したり。
聞く耳持たないと結局遠回りだよ。
808132人目の素数さん:2012/05/15(火) 08:49:47.61
>>806
> AR-AQ
その矢印を省いた表記をやめろ。そういうことをするから頭の中でも混同するんじゃないか?
809132人目の素数さん:2012/05/15(火) 08:51:22.59
頂点が(50,1)で
y=0のときxは0または100
となるような二次関数はどうなりますか?
810132人目の素数さん:2012/05/15(火) 08:56:07.51
>>807
>>808
ありがとうございました
811132人目の素数さん:2012/05/15(火) 09:02:19.81
>>809
> 頂点が(50,1)で
そういう二次関数の一般形を作る。
> y=0のときxは0または100
代入して確定させる。
812809:2012/05/15(火) 09:14:15.88
>>811
y=a(x-50)^2 + 1
0=a(0-50)^2+1=2500a+1
a=-1/2500
y=(-1/2500)(x-50)^2+1
で合ってますか?
813132人目の素数さん:2012/05/15(火) 11:19:54.24
>>806
もうその発想自体がマジキチ
ベクトルの問題なんか、99%最初に始点揃える所から始めるのに、それに気づかないって時点で今までまともに頭つかって無かった事が露呈している。
814132人目の素数さん:2012/05/15(火) 11:37:55.14
θsin(θ)の解き方がわからないです><おしえてください
815132人目の素数さん:2012/05/15(火) 11:41:52.44
また意味不明な
816132人目の素数さん:2012/05/15(火) 11:42:01.39
↑訂正します
θsin(θ)=aの式をθ=の形になおしたいんですが、なんの公式を使えばいいですか?
817132人目の素数さん:2012/05/15(火) 11:58:39.83
無理じゃね?
818132人目の素数さん:2012/05/15(火) 12:31:03.02
>789
スカラーという言葉があるんやで
819132人目の素数さん:2012/05/15(火) 13:04:36.85
>>816
あえて言えばテーラー展開かな?
でもまともな近似値出る前に計算不可能になりそうだが…
820132人目の素数さん:2012/05/15(火) 13:14:36.39
too many solutions
821132人目の素数さん:2012/05/15(火) 15:58:49.07
「図形と式」と「ベクトル」は同時平行で学んだ方が前者の理解が容易、と聞いたんだけど、マジか?
まあ、正確には前者の基本事項をざっと学んだ直後に後者の基本事項を学んで、改めて前者を学ぶて方法らしいけど
822132人目の素数さん:2012/05/15(火) 16:36:04.00
先にベクトルやるだけで良くね?
823132人目の素数さん:2012/05/15(火) 17:43:42.20
>>738
>>742
>>813
完全に無視ワロタ
824132人目の素数さん:2012/05/15(火) 17:53:55.86
>>822
先にベクトルの基本事項を学んで、後から図形と式て順番ですか?
825132人目の素数さん:2012/05/15(火) 18:04:18.64
初めて書き込みます、至らないところがあると思いますがよろしくおねがいします

12,16,18の最小公倍数を素因数分解で求めたら、
12=2^2×3
16=2^4
18=2×3^2 となりました。

それで最小公倍数は指数が大きいものをかけると書いてあったので、
2^2(4)×2^4(16)×3^2(9)をかけたら576となり、正答の144と大きく違ってしまいました。

最小公倍数の求めかたが合っていないのでしょうか?
826132人目の素数さん:2012/05/15(火) 18:05:57.71
>>819
返答あざっす^^
827132人目の素数さん:2012/05/15(火) 18:09:37.22
>>825
>指数が大きいもの
>12=2^2×3
>16=2^4
>18=2×3^2
2の累乗で最大は2^4
3の累乗で最大は3^2
(他の素数についての累乗は現れない)
これらの積は2^4×3^2=144

最小公倍数の「定義」と「簡単な求め方」は別物だからね
何故この求め方で定義通りの値になるのか、ちっとは考えようね
828132人目の素数さん:2012/05/15(火) 18:16:26.41
>>824
それこそベクトルの基本的な考えなんて、まともな頭の持ち主なら教科書読めばわかる。
逆に分かりにくくなるんじゃないかって言うぐらい低レベルから説明してあるから、先に内容さらっとくぐらいわけ無い。
たださ、ベクトルを後にやろうが先にやろうが、図形と方程式の考えにベクトルが使われている事は気付く奴は気付くし、気付かない奴は気付かないと思うよ。
829132人目の素数さん:2012/05/15(火) 18:17:56.02
lim[x→∞]cosx/xを求める問題で

[振動するものがあるのではさみうちの原理・・・@]を使って
いくのですが
0≦|cosx|≦1としますよね
そして、|1/x|をかけて...といくわけですが、[そもそも場合わけする理由が不等式の両辺に文字をかけるので、その正負で場合わけが生じるから面倒...って理由ででした・・・A]これは、今xが∞の近くでの話なので必ずx>0ではないのでしょうか、
また、@,Aの考え方はあってますか?
830132人目の素数さん:2012/05/15(火) 18:20:31.24
831132人目の素数さん:2012/05/15(火) 18:40:33.82
>>827
返信ありがとうございました!
理解できました、素数ごとに指数が大きいのをかけるんですね
丁寧に書いていただいて助かりました
832132人目の素数さん:2012/05/15(火) 18:48:39.31
>>830
どういうことでしょうか?
お願いします
833132人目の素数さん:2012/05/15(火) 18:58:20.68
>>832
この程度の英語出来ないなら
中学からやり直せ
834132人目の素数さん:2012/05/15(火) 19:14:14.32
>>833
@,Aの考え方はあっているのかということ。
絶対値はxが無限大の近くでの話なので常に正、ゆえに絶対値つけなくても場合わけが生じないのでは?ということ。
この2点についてです。お願いします。
835132人目の素数さん:2012/05/15(火) 19:23:18.55
>>829
>lim[x→∞]cosx/xを求める問題で

>[振動するものがあるのではさみうちの原理・・・@]を使っていくのですが

振動するものがあるからってわけじゃ…
発想の取っ掛かりとしてはいいかもしれないけど、言葉にすると変だな。振動する事もあるんだし発散する事もあるから振動する事からハサミうちとはならん。
ただ高校数学では実質上収束するものしかで無さそうだから問題ないのでは?

どっちかって言うと、式変形する前から0に行くのが分かっているから適当なもので挟んで証明しようって感覚なんだが

>0≦|cosx|≦1としますよね

別に無理に絶対値使う必要性ないし
この問題だけならx>0だけ考えればよい
836132人目の素数さん:2012/05/15(火) 19:28:07.56
>>829
合ってるよ。ていうか不等式に正の値をかける時は場合分けは生じないよ?
|1/x|という値はですね、数直線上の原点から、同じく数直線上の1/xまでの「長さ」なんですよ。
長さっていうのは必ず正の値で表わされるものなんですよ。例えば1センチとか42.195キロメートルとかね。
特にメートルとかの単位が数直線に指定されていない場合、距離は単に正の値だけで表わすものだから無理してつけなくていいよ。

要するに、|1/x|をかけちゃっても場合分けは生じないわけ。
かけたら後はxを無限大まで飛ばして不等式の真ん中がどんどん0に追いやられていくのを示せばいい
837132人目の素数さん:2012/05/15(火) 19:33:28.97
>>829
その考え方でいい
lim[x→−∞]cosx/xも同じようにして解くことができる
838132人目の素数さん:2012/05/15(火) 19:34:45.59
>>835-836
ありがとうございます!理解しました!
839あのこうちやんは始皇帝だった:2012/05/15(火) 19:41:30.38

 お前たちは、定職に就くのが先決だろがあああああああ!!!!!!!!!!

 ニート・無職の、ゴミ・クズ・カスのクソガキどもがあああああ!!!!!!!!!!!!
840132人目の素数さん:2012/05/15(火) 19:43:27.61
数Aの範囲ですが、教えてください。お願いします

集合A,Bは全体集合U={x|50≦x≦200, xは実数}
A={x|xは2の倍数}
B={x|xは3の倍数}

上記の条件で次の集合のっ要素の個数を求めよ

   ________
問  AПB

   ___
問  AUB   

記号の読みがわからないので
似たような記号を使用しました。
尚、A=76(個) B=50(個)というのは出ています

よろしくお願いします
841132人目の素数さん:2012/05/15(火) 19:49:20.05
記号の意味は分かってるの?
842132人目の素数さん:2012/05/15(火) 19:50:14.80
>>841
わかってます

Пが「AとBの共通部分」で
Uが「AとBの和集合」です
843132人目の素数さん:2012/05/15(火) 19:51:30.91
xは実数…要素の個数…
844132人目の素数さん:2012/05/15(火) 19:57:10.12
f(x)=x^2 (x≠0)
   1  (x=0)

このような関数f(x)は、x=0で極限を持ちますか?
845132人目の素数さん:2012/05/15(火) 20:00:18.44

相加相乗の関係 の問題です。 

(a+1/b)(b+4/a)≧9 を証明せよ。


展開すれば、すんなり証明できるのですが、

(a+1/b)≧2√a×1/b
(b+4/a)≧2√b×4/a

として、辺々を掛け合わせると

(a+1/b)(b+4/a)≧8 となって、
どこが間違えているのかが、分かりません。


もし、分かる方がいたら教えてください。
よろしくお願いします。

846132人目の素数さん:2012/05/15(火) 20:05:23.97
>>845
バカは相加相乗使う資格ないって事だな。
847132人目の素数さん:2012/05/15(火) 20:06:07.71
>>842
A∩Bが何の集合か分かれば個数を数えて補集合は求まるだろ
A∪Bも同様
848132人目の素数さん:2012/05/15(火) 20:09:39.50
>>845

(a+1/b)≧2√a×1/b
(b+4/a)≧2√b×4/a

2式それぞれの等号条件は
849132人目の素数さん:2012/05/15(火) 20:11:09.96
>>816
aは定実数として
(1)θsin(θ)=a
両辺をθで微分して
sin(θ)+θcos(θ)=0
これより
(2)θcos(θ)=-a/θ
(1)、(2)の両辺を2乗して加えると
θ^2=a^2+a^2/θ^2
これから
θ^4-a^2θ^2-a^2=0
よって
θ=±√{(a^2+√(a^4+4a^2))/2}
850132人目の素数さん:2012/05/15(火) 20:13:37.74
>>840
困ったな、Uの個数が連続体濃度だ。「xは整数」と直します。
A∩Bは6の倍数となるので、最小は54、最大は198と適当にみつける。
したがってA∩Bの個数は(198-48)÷6=25個となる。
これの補集合なので、(あとは略)
2番目はn(A∪B)=n(A)+n(B)-n(A∩B)を利用する。(あとは略)
851132人目の素数さん:2012/05/15(火) 20:38:23.18
http://i.imgur.com/vY7lp.jpg
(b-c)(a-b)(a-c)という答えは出たのですが何故
-(a-b)(b-c)(c-a)となるのかが理解できません
マイナスを付けたら全ての符号が逆になるのではないのですか?
なぜ(a-b)と(b-c)はそのままで(a-c)だけが(c-a)となっているのでしょうか
852132人目の素数さん:2012/05/15(火) 20:39:41.22
えっ…
最近とんでもなく低レベルな質問者多いな
853132人目の素数さん:2012/05/15(火) 20:40:22.97
>>851
-2×3=(-2)×(-3) ?
854132人目の素数さん:2012/05/15(火) 20:41:43.46
>>816
一般には解けません。a=0,(n+π/2),π/12,などは思いつきます。
855132人目の素数さん:2012/05/15(火) 20:43:34.17
>>848

等号成立に矛盾が生じる…。

ので、辺々掛け合せの相加相乗は使えないんですね!


ありがとうございます。
856132人目の素数さん:2012/05/15(火) 20:48:42.55
1個20円のみかんを何個か買って500円玉を出したらおつりは80円みかんはいくら?
という問題で500ー20×X=80にでなぜ21になるのかわかりません

逆算したらわかりますが誰教えてください
857132人目の素数さん:2012/05/15(火) 20:49:30.29
真偽判定が正しく行えているか、思考過程は正しいかチェックお願いします。。

x>3⇒x≧3(真)
x≧3⇒x>3(偽)
数直線上に書いて包含関係を調べれば明らか。
このやり方で問題無いですか?
858132人目の素数さん:2012/05/15(火) 20:51:02.45
>>853
ありがとうございます!
モヤモヤが消えました
>>852
LDレベルで数学が出来ないのです
すみません
859132人目の素数さん:2012/05/15(火) 20:54:07.96
>>855
普段は意識してないのだろうけど、
等号成立条件の明記および吟味のしてない相加相乗の不等式を解答に書いたら
まるまるそこの部分に点は来ないと認識してくれ。
ようはその間違えをしちゃうような奴は普段から相加相乗で解答書いてる時すら受験レベルでは点がなかったと言う事だ。
860132人目の素数さん:2012/05/15(火) 21:08:40.26
>>849
861132人目の素数さん:2012/05/15(火) 21:11:04.92
>>851
この答えを書いた人が(a-c)を-(a-c)と書き換えたかったから。(ちなみに(a-c)=-(a-c)だからこの書き換えは正しい)
だってその方が答えが美しいじゃないか。
それに文字がab,bc,caとまるでチェーンみたいで覚えやすいだろ?
862132人目の素数さん:2012/05/15(火) 21:24:21.61
>>856
式のたてかたですが、等価交換のほうがまちがいが少ない。
20x + 80 = 500
20x = 500 - 80
20x = 420
x = 420÷20
x = 21
863132人目の素数さん:2012/05/15(火) 21:31:11.13
>>849
θ=a=π/2 で試してみる
864132人目の素数さん:2012/05/15(火) 21:35:15.53
>>857
むしろ包含関係を考えないほうが頭おかしいだろう。
865132人目の素数さん:2012/05/15(火) 21:41:12.83
ほ・・・包茎関係?
866132人目の素数さん:2012/05/15(火) 21:44:24.35
↑アホ
867132人目の素数さん:2012/05/15(火) 21:48:42.95
>>861
すごいな
868132人目の素数さん:2012/05/15(火) 21:58:49.33
>>849
2乗する時は両辺が正じゃないとダメだって約束したじゃないですかーっ
869132人目の素数さん:2012/05/15(火) 22:00:14.30
両辺負でもいいよ。
870132人目の素数さん:2012/05/15(火) 22:00:45.49
>>862
金本位性ですか
871132人目の素数さん:2012/05/15(火) 22:02:26.28
>>816
無理無理、代数では表現不能。sinθ=aを解くようなもの。
周期的に値をとる必要がある。
だから、代数で答えてる奴は嘘。
872132人目の素数さん:2012/05/15(火) 22:31:00.81
>>870
釣り銭を暗算で計算する国なんて、日本だけとは言わないが、少ないだろう。
(得たもの)=(失ったもの)
このようにしないと、複雑な問題では符号をまちがえがちだ。
873132人目の素数さん:2012/05/15(火) 22:34:53.70
>>861
a-c=-(c-a)だろw
874132人目の素数さん:2012/05/15(火) 22:36:18.22
>>861
お前みたいな馬鹿が美しさを語るな
875132人目の素数さん:2012/05/15(火) 22:53:17.99
ここまで馬鹿だと逆に美しい
876132人目の素数さん:2012/05/15(火) 22:57:55.05
質問いいですか?
x^2+4y^2=1,y>0を満たすとき
z={(x+1)^2+y^2}/(x+1)yの最小値を求めよ
という問題です

どうやってやるのでしょう?
xとyのそれぞれの条件求めて精いっぱいです
877132人目の素数さん:2012/05/15(火) 22:58:25.28
>816>871
けどθの範囲を0≦θ≦π/2とかに決めたら解けそうじゃね?
面白そうだからやってみようかな
878132人目の素数さん:2012/05/15(火) 22:58:56.31
よくいるよね、美しいって言いたいだけの馬鹿
879132人目の素数さん:2012/05/15(火) 23:04:07.31
極限で途中
sin(π+0)とか書いていいですか?
[+0] ガウス記号

+0とかのことです
880132人目の素数さん:2012/05/15(火) 23:11:46.78
881仙石18:2012/05/15(火) 23:27:10.63
f(x,y,lambda)={(x+1)^2+y^2}/(x+1)y-lambda(x^2+4y^2-1)
fx=fy=flambda=0-->x=-3/5,y=2/5,lambda=0

z= 2
882132人目の素数さん:2012/05/15(火) 23:34:15.02
>>877
傾きを度数で表現するとすれば、俺は円を多角形にするようにしか思い浮かばないな、
正確な値を求めるのは俺には無理だけど、近似値くらいなら求めれるんじゃないかな。
360角形にすれば、整数の範囲ならいけるね。
360角形が作図できればの話だけど。
まあ解けたら報告してくれよ。
883132人目の素数さん:2012/05/15(火) 23:36:24.64
なんかノルアドレナリン全開っぽいやつが住み着いてるな
いいぞおもしろいからもっとやれ、のんびり観戦してるから
884132人目の素数さん:2012/05/15(火) 23:50:20.21
θsin(θ)=a
sint=a/t
sint/t=a/t^2->1
a=t^2
885132人目の素数さん:2012/05/15(火) 23:50:27.46
300の約数の総積ってどうやって求めるんですか。
886132人目の素数さん:2012/05/15(火) 23:55:34.74
まず素因数分解いたします
887132人目の素数さん:2012/05/15(火) 23:55:59.58
>>876 入試問題での常套手段
y>0なので -1<x<1、よってx+1>0
相加平均と相乗平均の関係より
z=(x+1)/y + y/(x+1) ≧2
等号はx+1=y, つまり x=-3/5, y=2/5 のとき成り立つ。
888132人目の素数さん:2012/05/15(火) 23:56:18.29
>>885
300^(約数の個数/2)

または

300の約数を小さい順に書いて全部掛けるだけ
889132人目の素数さん:2012/05/15(火) 23:56:51.86
>>863
面白いだろ?
890132人目の素数さん:2012/05/16(水) 00:01:11.28
>>879
お願い申しあげます!!!!
891132人目の素数さん:2012/05/16(水) 00:01:15.99
>>888
参考書に載ってないんですがどこで勉強したんですか
892132人目の素数さん:2012/05/16(水) 00:03:19.76
>>885
約数のペアを考える1と300、2と150、3と100などいずれも掛けると300になる。
あとは約数の個数を計算して...
893132人目の素数さん:2012/05/16(水) 00:05:10.71
>>891
よ び こ う
894132人目の素数さん:2012/05/16(水) 00:05:40.93
>>892
わからなょ〜 わかったらパンツうpする!
895132人目の素数さん:2012/05/16(水) 00:07:44.98
>>891
それを考えさせる問題なのに、参考書で勉強してることを前提にしてどうすんの…(^^;
896132人目の素数さん:2012/05/16(水) 00:15:26.90
4桁の整数が3で割り切れるには各位の数について
どんなことが成り立っていればよいか という問題を教えてください
897132人目の素数さん:2012/05/16(水) 00:18:20.15
>>894
約数の個数は300=2^2・3^1・5^2より3・2・3 = 18です。
約数は1, 2, 3, 4, 5, 6, 10, 12, 15, 20, 25, 30, 50, 60, 75, 100, 150, 300
約数の積は (1・300)(2・150)(3・100)(4・75)(5・60)(6・50)(10・30)(12・25)・(15・20)
ペアの個数は約数の個数の半分となるので、約数の積は 300^9
完全平方数の場合は約数の個数が奇数となり少し変化。
898132人目の素数さん:2012/05/16(水) 00:19:30.59
X=約数×約数×約数×・・・・・・・・×約数(コウベキ)

X=約数×約数×約数×・・・・・・・・×約数(ショウベキ)

両辺かけてX^2=300^(約数の個数)
両辺1/2乗して、X=300^(約数の個数/2)

さぁパンツ見せろ。
899132人目の素数さん:2012/05/16(水) 00:22:16.85
極限で解答の途中
sin(π+0)とか書いていいですか?
[+0] ガウス記号

+0とかのことです
900132人目の素数さん:2012/05/16(水) 00:23:42.88
>>896
10=9+1, 100=99+1, 1000=999+1を使う
901132人目の素数さん:2012/05/16(水) 00:32:32.38
>>900
1000a+100b+10c+d
=(999+1)a+(99+1)b(9+1)c+d

3で割りきるには各桁の数字を足して3で割り切れる が正解ですか?
902132人目の素数さん:2012/05/16(水) 00:37:47.49
分数の割り算って分母どうし分子同士割ってもいけるんだ…
その代わり3,33333とかになるけど
903132人目の素数さん:2012/05/16(水) 00:38:34.21
>>899
何が聞きたいか全くわからんから
なにか例題とその解答を一通り書いてみてくれ
904132人目の素数さん:2012/05/16(水) 00:38:34.80
>>901 はい
905132人目の素数さん:2012/05/16(水) 00:38:42.19
>>902
906132人目の素数さん:2012/05/16(水) 00:41:14.01
>[+0] ガウス記号

>+0とかのことです

この三行が何を言いたいのかよく分からない。
もっと日本語使って。
907132人目の素数さん:2012/05/16(水) 00:41:27.37
a+b10+c100+d1000=0 mod3
a+b+c+d=0 mod 3
908132人目の素数さん:2012/05/16(水) 00:43:19.74
>>898
エレガントな解説ですね
909132人目の素数さん:2012/05/16(水) 00:45:15.94
>>899
ちゃんと説明して使うか、あるいは減点覚悟で
910132人目の素数さん:2012/05/16(水) 01:50:45.88
次の不等式を満たすxの値の範囲を求めよ。

2x-3/x-2>x

√ ̄ ̄
 5-x ≦3

以上の2題の解法が分かりません。どなたかご教授お願いします。
911132人目の素数さん:2012/05/16(水) 01:54:19.52
>>910
>>1のテンプラみて書きなおせ
912132人目の素数さん:2012/05/16(水) 02:01:24.51
>>910
(2x-3)(x-2)>x(x-2)^2を解く、(x-2)が共通因数に注意
5-x≧0、両辺を2乗して5-x≦9
913132人目の素数さん:2012/05/16(水) 02:02:06.90
A−A×0.9×15/21=7500

この問題の答えはA=21000なんですが分数が割り切れないので計算できません。どなたか解き方を教えて下さい。アホな質問ですみません。
914132人目の素数さん:2012/05/16(水) 02:59:17.46
>>910
a>bという不等式があるとする。
不等式の両サイドの数に正の数c(>0)をかけると、ac>bcが成り立つ。
(例: 3>2の両辺に5(>0)をかけると15>10となり不等式は成り立つ)
一方、両サイドに負の数d(<0)をかけるとad<bdが成り立つ。
(例: 3>2の両辺に-5(<0)をかけると-15>-10、ではなく -15<-10が成り立つ。不等号の向きが逆になるのである。)
この性質はa,b,cがどんな実数でも成り立つ。
また、正の数であれ”負の数であれ”、どんな実数もその二乗は必ず0以上となる。(例:2^2=4(>0)、(-3)^2=9(>0) )
これらの性質を決して忘れてはならない。
(2x-3)/(x-2)>xの両辺に(x-2)を掛けて(2x-3)>x(x-2)としたくなる気持ちは分かる。
だがちょっと待ってほしい。
x-2は正の数か負の数か分からないではないか!問題文に「x-2は正です」などの情報は書かれてないであろう?(x≠2である事は問答無用で分かるが。)
なので、(x-2)×{(2x-3)/(x-2)}=2x-3という数と、(x-2)×(x)=x(x-2)という数の大小関係が、最初の不等式と同じだと断言することができない。
だが、両辺に正の数をかければ数の大小関係は不変である。
そして、どんな数も二乗すれば正の数である。この性質を利用するのである。
そう、(x-2)^2を両辺に掛ければよいのだ!
そうすることで不等号の向きは間違いなく定まり、嫌な分数式も消える。
後は右辺を左辺に移行してそれを積の形にし、その積が0より大きいという条件を導く。
この条件からxの範囲をしぼり出すことができるのである。やってみよ。
915132人目の素数さん:2012/05/16(水) 03:34:16.51
ゾウのふんって肥料として使えますか?
916132人目の素数さん:2012/05/16(水) 06:16:17.53
関数f(θ)=acos^2θ+(a-b)sinθcosθ+bcos^2θの最大値が3+√7、最小値が3-√7
となるような実数定数a,bの値を求めよ

お願いします!
917132人目の素数さん:2012/05/16(水) 06:18:24.54
>>906
f(x)=[sinx]はx=0で連続か?[ ]はガウス記号
この問題で答案の中に
lim[sinx] =[sin(+0)]=[+0]=
x→+0

などという風にして書いてもいいですか?
918132人目の素数さん:2012/05/16(水) 06:30:00.53
連投すいません

a,bを実数の定数とするとき
lim{√(x^2+ax+b)-αx-β}でxを∞に飛ばしたとき(左の式)=0となる定数α、βの値を求めよ


{1/(x-1)}log{(x-a)/(1-a)}をx→1
としたときの極限を求めよ。ただしaは0<a<1を満たす定数とする
919132人目の素数さん:2012/05/16(水) 06:34:56.06
ヒント。漸近線の式の導き方。
920132人目の素数さん:2012/05/16(水) 07:32:41.03
>>917
ダメ
921132人目の素数さん:2012/05/16(水) 07:37:08.18
>>916>>918
これ河合塾の添削課題だよな
他人様の解答かいて提出して何になると思ってんの?
このレベルもわからんようならコースのレベル下げろって
922132人目の素数さん:2012/05/16(水) 09:37:13.50
>>917
だめ
+0を使いたいならこう書くのは?

x→+0のときsinx→+0
ゆえにf(x)→0

x→-0のときsinx→-0
ゆえにf(x)→-1

当然連続でない
923132人目の素数さん:2012/05/16(水) 11:08:33.49
>>916
加法定理から派生する倍角・半角の公式で「cos2θ、sin2θ」が出てくるから後は「合成」かな。

通信添削に費用かけるより定評のある参考書と教科書をきっちりした方が安上がりで着実。
924132人目の素数さん:2012/05/16(水) 11:20:08.73
x+y+z=9
x>0, y>0,z>0
x<y<z
(>は全部=つきです)
をみたすx,y,zの組の総数をもとめよという問題で、手元の解答では全部書き出してあるのですが計算で解く方法があったような気がするのですが思い出せません・・・
そのような方法はありますか
925132人目の素数さん:2012/05/16(水) 11:26:12.67
>>923
河合の添削課題って多分本科(浪人)の連中がプログラムの一環で渡される課題。
ぶっちゃけ白紙でだしても問題ないようなものだと思うぞ
出さない事を咎めるような組織でもないしな
926132人目の素数さん:2012/05/16(水) 11:44:46.78
>>924
>x+y+z=9
>x>0, y>0,z>0
>x<y<z
>(>は全部=つきです)
>をみたすx,y,zの組の総数をもとめよ

問題は正確に書こうよ。x,y,zは整数なんだよね?
x,y,zに大小関係がないなら、重複組み合わせと言う奴で9個のモノと2個の仕切りを並び換える問題になるけど
大小関係があると、二つが同じ場合は三倍、三つが違う場合は六倍で計算してるからそれを補正する必要がある。
様は結局バラで考えるのとあまり手間が変わらないと思うよ。
927132人目の素数さん:2012/05/16(水) 11:48:44.15
>924
9=x+y+z>x+x+x=3x
x<3
928132人目の素数さん:2012/05/16(水) 11:51:48.92
>>926
ありがとうございます。その通り整数です。すいません・・・
大小関係があるときはめんどくさいんですね。

>>927
そのやり方は頭の片隅にもありませんでした!やってみます
ありがとうございました
929923:2012/05/16(水) 13:29:26.51
>>925
アンカを自分につけるな。
自分は解法を提案してるだけ。>>916に言ってくれ。

料金払っているお客様に企業体はどうのこうのの文句は言わないだろうし
問題に不備があるなら「ここまでできたけど後どうすんの?」って>>916が聞けばイイだけだろうし。
930132人目の素数さん:2012/05/16(水) 13:39:13.33
>>929

何切れてるの?短気すぎない?

>自分は解法を提案してるだけ。

>>923
>通信添削に費用かけるより定評のある参考書と教科書をきっちりした方が安上がりで着実。

ってかいてあるからレス付けられたんじゃないの?解法だけじゃないからレス呼んだんだろ?
931132人目の素数さん:2012/05/16(水) 13:49:42.93
>>922
ですね!ありがとうございます
932132人目の素数さん:2012/05/16(水) 13:49:52.58
>>930
自分は添削指導受けてないから添削出すも出さなないもかんけーねーから、アホ過ぎ。
お前の寝言なのかアドバイスなのかは>>916宛が適切なので>>916言ってくれ。

>通信添削に費用かけるより定評のある参考書と教科書をきっちりした方が安上がりで着実。
↑にコメントだすのは自由だけどモット適切なコメントよこせ、トンチンカン野郎。
933132人目の素数さん:2012/05/16(水) 15:53:48.97
>>913
教科書を読みなさい。
そして、それは中学生の範囲である。
934132人目の素数さん:2012/05/16(水) 16:32:59.20
>>933
15/21を見て「あ、これ3で割れんじゃん」って事くらい気づこうぜ^^;
935132人目の素数さん:2012/05/16(水) 16:33:55.85
>>933さんすみませんアンカー間違えました^^;

>>913
15/21を見て「あ、これ3で割れんじゃん」って事くらい気づこうぜ^^;
936132人目の素数さん:2012/05/16(水) 16:54:15.52
>>935
お前もおかしい
3で割れるんじゃなく「3で約分できる」だろ
>>933
両辺を21回足してみ
937132人目の素数さん:2012/05/16(水) 16:57:34.36
両辺+両辺+両辺+両辺+両辺+両辺+両辺+両辺+両辺+両辺+
両辺+両辺+両辺+両辺+両辺+両辺+両辺+両辺+両辺+両辺+
両辺+両辺
938132人目の素数さん:2012/05/16(水) 17:10:34.77
確率の問題なんですが
20000個の部品からなるPCがあり
そのうち1個の部品が不良品である確率は100万分の1の時に
少なくとも1個は不良品がある確率を求めたいです

この場合は空集合の考え方でいいのでしょうか?
自分では解けなかったので解法と考え方を教えて欲しいですm(_ _)m
939132人目の素数さん:2012/05/16(水) 17:13:47.54
> 1個は不良品
てのは PC1台が不良で動かない ということか?
940132人目の素数さん:2012/05/16(水) 17:17:39.58
>>938
任意の2つの部品が不良品である事象は独立でしょうね
941132人目の素数さん:2012/05/16(水) 17:19:01.54
少なくとも1個は不良品がある確率
= 1 - 全てが不良品でない確率

全てが不良品でない確率
=部品1が不良品でない確率 × ・・・ × 部品20000が不良品でない確率
942938:2012/05/16(水) 17:21:48.81
>>939さん
1台のPCを構成する20000個のうちの1個の部品が、という意味でお願いします。

>>940さん
独立ですか・・・ちょっと調べてみます!
943132人目の素数さん:2012/05/16(水) 17:21:58.58
人生掛けてもだめなら足してみな
944132人目の素数さん:2012/05/16(水) 17:23:42.49
>938
単純な間違いだろうが
空集合ではなく余事象
945132人目の素数さん:2012/05/16(水) 18:14:52.77
数学の微分の問題

関数f(x)=x+a/x+alogx について次の問いに答えよ。

(1)f(x)が極大値と極小値をもつaの値の範囲
(2)f(x)が極大値をもたず、極小値をもつaの値の範囲

わからなくて困ってるんだが誰かいるかな?
946132人目の素数さん:2012/05/16(水) 18:16:42.80
いるけど、なにか
947132人目の素数さん:2012/05/16(水) 18:17:53.86
質問丸投げ大杉だろ
>>1読んだのかよ
948132人目の素数さん:2012/05/16(水) 18:18:27.11
分からないんじゃあ無くて
微分ができないだけじゃあないのか?
949132人目の素数さん:2012/05/16(水) 18:43:21.35
a[2]^2=√2
a[k+1]^2=2ー√(4ーa[k])
l[k]=2^(kー1)a[k]
のとき、l[k]、またはl[k]^2についての漸化式を導き、それを用いてl[k]を求めよ。

l[k+1]を求めて、l[k+1]ーl[k]=2^k(a[k+1]ーa[k])
という式を作ったのですがここから先どうすればいいのか、またそもそも合っているのかわかりません。
よろしくお願いします
950132人目の素数さん:2012/05/16(水) 19:41:23.59
>>949 問題がおかしい。aの漸化式が変だ。解けるはずがない。
951132人目の素数さん:2012/05/16(水) 19:58:01.00
>>945
グラフを書こうとすると、自然に極大と極小を求めたくなるので、できてしまうだろう
952132人目の素数さん:2012/05/16(水) 20:23:46.18
>>891
ちなみに、「マスターオブ整数」という参考書に
ずばり載ってるけどね。
953132人目の素数さん:2012/05/16(水) 20:57:51.05
>>949
a[2]=√2 でした
すみません
954132人目の素数さん:2012/05/16(水) 21:08:55.51
2点(1,1),(1,5)を通り、頂点が直線2x+y=3上にある放物線の方程式を求めよ。
平方完成使うみたいなんですけど、そもそも平方完成すらよくわかんないです。
955132人目の素数さん:2012/05/16(水) 21:09:43.12
すいません(1,1),(-1,5)です
956132人目の素数さん:2012/05/16(水) 21:26:45.29
>>954
勉強し直せよ
957132人目の素数さん:2012/05/16(水) 21:28:32.88
平方は二乗の別名
3^2は3の平方、x^2はxの平方、(a^3+b^2+4)^4は(a^3+b^2+4)^2の平方

平方完成がよく使われる場面は
ax^2+bx+c=0
という二次方程式。これに対し

[x+{b/(2a)}]^2=(b^2-4ac)/(4a^2)
とxを含む式全体を二乗の形にすると
いろいろわかりやすくなることが多い
958132人目の素数さん:2012/05/16(水) 21:53:29.36
>>954
平方完成は必要ないよ。2次関数のグラフの頂点の座標を知ってれば解ける。
頂点のx座標をtとすると、2t+y=3よりy座標は-2t+3となる。
y=a(x-t)^2+(-2t+3)のグラフがが(1,1),(-1,5)を通る。あとは連立方程式。
959132人目の素数さん:2012/05/16(水) 22:01:15.69
>>953
a[1]でなくa[2]なのはどうして?
a[k+1]^2= ... の式は合ってますか? 左辺の2乗は不自然なので
960132人目の素数さん:2012/05/16(水) 23:14:24.44
lim[f(x,y)]が存在するための条件ってなんですか?
961132人目の素数さん:2012/05/16(水) 23:16:02.54
なんでしょう?
962ε-δ:2012/05/16(水) 23:17:32.74
963132人目の素数さん:2012/05/17(木) 01:29:53.04
ドラゴン桜
964132人目の素数さん:2012/05/17(木) 01:32:51.71
P(x)=x^3+ax^2+bx+c
x+3で割ると余りが2であり x+2+√3で割り切れるとする
このときのa b cの値を求めよ。
965132人目の素数さん:2012/05/17(木) 01:34:39.73
条件が足りないか、一意に決まるためには。
966132人目の素数さん:2012/05/17(木) 01:35:15.11
967132人目の素数さん:2012/05/17(木) 01:36:52.00
なけなしの知恵を絞った自作問題なんだろうな、大目に見てやろうよ。
968132人目の素数さん:2012/05/17(木) 01:42:53.73
整数a,b,cを係数とする三次の正式

P(x)=x^3+ax^2+bx+c

は,x+3で割ると余りが2であり x+2+√3で割り切れるとする
このとき a=ア b=イ c=ウ,
 P(x)=(x+エ)(x^2+オx+カ)

っていう問題なんだ
友達から画像送られてきたからわからんが
模試かなんかの過去問っぽい
969132人目の素数さん:2012/05/17(木) 01:45:15.38
いや、そういう嘘はいいから。
970132人目の素数さん:2012/05/17(木) 01:48:54.29

いや 嘘とか意味わからん
実際に送られてきたまんまだ

自作とかじゃないぞ
971132人目の素数さん:2012/05/17(木) 02:00:54.31
x+2-√3でも割り切れんじゃね?って思わないのか?
972132人目の素数さん:2012/05/17(木) 02:02:26.84

>>971
やってみる
973132人目の素数さん:2012/05/17(木) 02:13:11.90
>>971の言うとおり確かにx+2-√3でも割り切れるけど、
無理数の相当とか用いると3つの式出てくるから一意に決まると思う。
974132人目の素数さん:2012/05/17(木) 02:19:00.23
>>973
無理数の√3の項の係数が0になるからこそ、-√3も係数が0になるんだし、まぁおんなじようなもんじゃん?
975132人目の素数さん:2012/05/17(木) 02:21:51.29
最初にレス付けた人の名誉の為に言っとくと、最初はabcの整数条件明記してなかったし、964の段階ではとけないよ。
976132人目の素数さん:2012/05/17(木) 02:32:27.14
出来ない奴って総じて叩かれるまで問題に書いてある整数や有理数の条件書かないよね。
文字の条件とか何よりも大切なのに飾りとでも思っているんだろうな。
977132人目の素数さん:2012/05/17(木) 02:34:09.28
k^2<-4を解け。と言う問題を教えてください。Kは複素数とする。
978132人目の素数さん:2012/05/17(木) 02:41:19.95
bi (b<-2,2<b)
979977:2012/05/17(木) 02:47:36.83
>>978
b < -2i,2i < b とすると駄目ですか?
複素数の大小関係を定義した場合ですが・・・
980132人目の素数さん:2012/05/17(木) 02:49:44.33
どうやって複素数の大小定義するんだよ(笑)
すすんで泥沼に足突っ込んでまでその書き方したいのか?
981132人目の素数さん:2012/05/17(木) 02:51:05.33
>>979
別にいいけど、自力できちんと定義できるか?
982132人目の素数さん:2012/05/17(木) 03:17:13.27
k=x+iy
k^2=(x^2)-(y^2)+2xyi<-4
x=0 or y=0
y=0のときx^2≧0なので不適
∴x=0でy^2<4⇔-2<y<2
っていいの?
983132人目の素数さん:2012/05/17(木) 03:24:03.74
>>982
>(x^2)-(y^2)+2xyi<-4
この式を書いたら負けだろ
書かずに言葉で先に場合分けした方が無難

>∴x=0でy^2<4⇔-2<y<2
不等号の向き逆な(笑)
984132人目の素数さん:2012/05/17(木) 03:24:07.41
惜しいなぁ〜何でソコ間違えるんだよおい
985132人目の素数さん:2012/05/17(木) 04:07:24.59
>>849のθは関数sinθと関数a/θの値が等しくかつ接線の傾きが等しいという2式から求まった値だからこの2曲線の接点ということでいいんでしょうか?
986132人目の素数さん:2012/05/17(木) 04:37:58.05
大したことではないのだが、これの証明がいまひとつうまくできないんだ。

f:X→Y を局所Noetherスキームの間の固有射であるとし、
X , Y の構造層 ? , ? の間には f*(?)=? が成り立つと仮定する。
このとき、任意の y∈Y に対し、 f^-1(y) は、空でなく、連結である。
987132人目の素数さん:2012/05/17(木) 06:50:57.77
ほーそれはそれは
988132人目の素数さん:2012/05/17(木) 11:02:35.44
>>986 どこからのコピーなのだろうか?。原典をを見つけるのも面白い。
「X , Y の構造層 ? , ? の間には f*(?)=? 」は「X , Y の構造層 Ⓧ , Ⓨ の間には f*(Ⓧ)=Ⓨ」でしょ
989132人目の素数さん:2012/05/17(木) 12:49:34.47
     π  13
65°=65× ─= ─π
180 36

途中式が解りません
990132人目の素数さん:2012/05/17(木) 12:51:51.98
>>989
>>1のテンプレくらい見ろよアホ
書きなおせバカ
そんな無能だからわかんねーーてなことなんだろ
991132人目の素数さん:2012/05/17(木) 12:52:12.22
>>989
>>1

度数法の65°を弧度法に直すなら1°=π/180 [rad]を利用
992132人目の素数さん:2012/05/17(木) 13:13:37.70
a<x<b...@
c<x<d...A
不等式@を満たすすべてのxが不等式Aを満たすとき@はAであるための十分条件ですか?
c<a<b<dでいいのか教えたください。
993132人目の素数さん:2012/05/17(木) 13:20:10.19
-3±√21/3を
-1±√21に約分?出来ませんかと質問した
高認数学独学中も者です
その節はお世話になりました
これが約分?できないとが教わりましたが
問題を解いているうちに
@、2±√12/2
    ↓
A、2±2√3/2
    ↓
B1±√3
となるようですが
Aまでは理解できるのですが
何故、Bに約分?出来るのでしょうか?

どなたか解説お願いします
994993:2012/05/17(木) 13:22:33.29
>>993
文章がめちゃくちゃになってしまっている…
問題を解いているうちに
@、2±√12/2
    ↓
A、2±2√3/2
    ↓
B1±√3
になる問題が出てきたという意味です
訂正いたします
995132人目の素数さん:2012/05/17(木) 13:30:31.91
(2±√12)/2=1±√3にはなるが
2±√12/2=2±√3にしかならん。
996132人目の素数さん:2012/05/17(木) 13:39:13.31
>>994
(2±2√3)/2 = 2/2 ± 2√3/2 = 1±√3
約分ならば (2±2√3)/2 =2(1±√3)/2 のようにすると、分子が1つの掛算になるので
2で約せる。
997993:2012/05/17(木) 13:52:22.48
レスありがとうございます

(2±2√3)/2 = 2/2 ± 2√3/2 = 1±√3
2で約分すると
2と±2√3の両方を2で割らないとないといけない
@、2/2 =1
A、±2√3/2=±1√3→1は省略して→=±√3
@、Aで 1±√3 になるという事でしょうか?
998132人目の素数さん:2012/05/17(木) 13:54:55.79
>>997
相当分かってないから、
あなたは中学校の文字の式からやり直した方が良い
今のままで高校の課程をやるのは厳しい
999993:2012/05/17(木) 13:57:55.01
>>998
そうですね
中学数学に、場合によっては小学校の算数に一部戻ってみます
ありがとうございます
1000132人目の素数さん:2012/05/17(木) 14:01:54.72
>>992
合ってる
AならばBの時(A→B)、AがBの十分条件、BがAの必要条件

「(十分)→(必要)」になるから、
「十分な人から必要な人へ」と覚えさせられた記憶がある
10011001
このスレッドは1000を超えました。
もう書けないので、新しいスレッドを立ててくださいです。。。